FISIKA TPB

176

Transcript of FISIKA TPB

Page 1: FISIKA TPB
Page 2: FISIKA TPB

6 -

8 -

4 -

6 -

0 -

2 -

5 -

7 -

2 -

4 -

8 -

9 -

3 -

5 -

9 -

0 -

3 -

5 -

8 -

0 -

3 -

5 -

9 -

1 -

6 -

DAFTAR ISI

Ujian Tengah Semester I Fisika Dasar I ................................................................... - 5 -

Soal Ujian Tengah Semester I Tahun 2002/2003 .............................................................. -

Solusi Ujian Tengah Semester I Tahun 2002/2003 ........................................................... -

Soal Ujian Tengah Semester I Tahun 2003/2004 ............................................................ - 1

Solusi Ujian Tengah Semester I Tahun 2003/2004 ......................................................... - 1

Soal Ujian Tengah Semester I Tahun 2004/2005 ............................................................ - 2

Solusi Ujian Tengah Semester I Tahun 2004/2005 ......................................................... - 2

Soal Ujian Tengah Semester I Tahun 2005/2006 ............................................................ - 2

Solusi Ujian Tengah Semester I Tahun 2005/2006 ......................................................... - 2

Soal Ujian Tengah Semester I Tahun 2006/2007 ............................................................ - 3

Solusi Ujian Tengah Semester I Tahun 2006/2007 ......................................................... - 3

Soal Ujian Tengah Semester I Tahun 2007/2008 ............................................................ - 3

Solusi Ujian Tengah Semester I Tahun 2007/2008 ......................................................... - 3

Soal Ujian Tengah Semester I Tahun 2008/2009 ............................................................ - 4

Solusi Ujian Tengah Semester I Tahun 2008/2009 ......................................................... - 4

Ujian Tengah Semester II Fisika Dasar I ............................................................... - 48 -

Soal Ujian Tengah Semester II Tahun 2002/2003........................................................... - 4

Solusi Ujian Tengah Semester II Tahun 2002/2003 ........................................................ - 5

Soal Ujian Tengah Semester II Tahun 2003/2004........................................................... - 5

Solusi Ujian Tengah Semester II Tahun 2003/2004 ........................................................ - 5

Soal Ujian Tengah Semester II Tahun 2004/2005........................................................... - 5

Solusi Ujian Tengah Semester II Tahun 2004/2005 ........................................................ - 6

Soal Ujian Tengah Semester II Tahun 2005/2006........................................................... - 6

Solusi Ujian Tengah Semester II Tahun 2005/2006 ........................................................ - 6

Soal Ujian Tengah Semester II Tahun 2006/2007........................................................... - 6

Solusi Ujian Tengah Semester II Tahun 2006/2007 ........................................................ - 7

Soal Ujian Tengah Semester II Tahun 2007/2008........................................................... - 7

Page 3: FISIKA TPB

Solusi Ujian Tengah Semester II Tahun 2007/2008 ........................................................ - 78 -

Soal Ujian Tengah Semester II Tahun 2008/2009........................................................... - 81 -

Solusi Ujian Tengah Semester II Tahun 2008/2009 ........................................................ - 83 -

Ujian Akhir Semester I Fisika Dasar I .................................................................... - 85 -

Soal Ujian Akhir Semester I Tahun 2003/2004............................................................... - 86 -

Solusi Ujian Akhir Semester I Tahun 2003/2004 ............................................................ - 88 -

Soal Ujian Akhir Semester I Tahun 2004/2005............................................................... - 90 -

Solusi Ujian Akhir Semester I Tahun 2004/2005 ............................................................ - 91 -

Soal Ujian Akhir Semester I Tahun 2007/2008............................................................... - 93 -

Solusi Ujian Akhir Semester I Tahun 2007/2008 ............................................................ - 94 -

Ujian Tengah Semester I Fisika Dasar II ............................................................... - 85 -

Soal Ujian Tengah Semester I Tahun 2002/2003 ............................................................ - 97 -

Solusi Ujian Tengah Semester I Tahun 2002/2003 ......................................................... - 98 -

Soal Ujian Tengah Semester I Tahun 2003/2004 .......................................................... - 101 -

Solusi Ujian Tengah Semester I Tahun 2003/2004 ....................................................... - 102 -

Soal Ujian Tengah Semester I Tahun 2005/2006 .......................................................... - 106 -

Solusi Ujian Tengah Semester I Tahun 2005/2006 ....................................................... - 108 -

Soal Ujian Tengah Semester I Tahun 2006/2007 .......................................................... - 111 -

Solusi Ujian Tengah Semester I Tahun 2006/2007 ....................................................... - 113 -

Soal Ujian Tengah Semester I Tahun 2007/2008 .......................................................... - 118 -

Solusi Ujian Tengah Semester I Tahun 2007/2008 ....................................................... - 119 -

Soal Ujian Tengah Semester I Tahun 2008/2009 .......................................................... - 118 -

Solusi Ujian Tengah Semester I Tahun 2008/2009 ....................................................... - 126 -

Ujian Tengah Semester II Fisika Dasar II ............................................................ - 128 -

Soal Ujian Tengah Semester II Tahun 2002/2003......................................................... - 129 -

Solusi Ujian Tengah Semester II Tahun 2002/2003 ...................................................... - 130 -

Soal Ujian Tengah Semester II Tahun 2003/2004......................................................... - 135 -

Solusi Ujian Tengah Semester II Tahun 2003/2004 ...................................................... - 137 -

Soal Ujian Tengah Semester II Tahun 2004/2005......................................................... - 141 -

Page 4: FISIKA TPB

Solusi Ujian Tengah Semester II Tahun 2004/2005 ...................................................... - 142 -

Soal Ujian Tengah Semester II Tahun 2005/2006 ...................................................... - 146 -

Solusi Ujian Tengah Semester II Tahun 2005/2006 ...................................................... - 148 -

Soal Ujian Tengah Semester II Tahun 2006/2007......................................................... - 153 -

Solusi Ujian Tengah Semester II Tahun 2006/2007 ...................................................... - 154 -

Soal Ujian Tengah Semester II Tahun 2007/2008......................................................... - 158 -

Solusi Ujian Tengah Semester II Tahun 2007/2008 ...................................................... - 160 -

Soal Ujian Tengah Semester II Tahun 2008/2009......................................................... - 164 -

Solusi Ujian Tengah Semester II Tahun 2008/2009 ...................................................... - 165 -

Ujian Akhir Semester II Fisika Dasar II................................................................ - 167 -

Soal Ujian Akhir Semester II Tahun 2003/2004 ........................................................... - 168 -

Solusi Ujian Akhir Semester II Tahun 2003/2004 ........................................................ - 169 -

Soal Ujian Akhir Semester II Tahun 2007/2008 ........................................................... - 172 -

Solusi Ujian Akhir Semester II Tahun 2007/2008 ........................................................ - 173 -

Page 5: FISIKA TPB

UJIAN TENGAH SEMESTER I

FISIKA DASAR I

Page 6: FISIKA TPB

SOAL UJIAN TENGAH SEMESTER I

FISIKA DASAR I

TAHUN 2002/2003

1. Sebuah balok m = 1 kg diikatkan pada sebuah pegas dengan k = 50 N/m, kemudian ditarik sejauh

10 cm dan dilepaskan. Jika pada t = 0 s diambil saat pegas dalam keadaan setimbang (gambar 1), a. Tentukan fungsi osilator harmonik yang diperoleh b. Hitunglah kecepatan balok pada saat simpangan pegas setengah dari simpangan maksimum 2. Dua buah benda (lihat gambar 2) m1 = 2 kg, m2 = 1 kg dihubungkan dengan tali ideal, tak lentur dan

tanpa massa di atas bidang miring ( θ = 37o ) yang memiliki µs = 0,2 dan µk = 0,04. a. Gambarkan diagram benda bebas untuk m1 dan m2 b. Berapa tegangan tali masing-masing benda saat tepat akan bergerak? c. Jika tali yang menghubungkan benda 1 dan dinding diputus, tentukanlah percepatan masing-

masing benda d. Berdasarkan hasil c, apakah m1 akan menabrak benda m2? Jelaskan jawaban saudara 3. Sebuah perahu bergerak menyeberangi sungai ke arah utara dengan kecepatan awal v = 2 m/s. Di

saat yang sama hujan turun dengan kecepatan air jatuh terhadap tanah rata-rata 1 m/s. Air hujan akan mengakibatkan air sungai mengalir sedemikian rupa sehingga kecepatan aliran air terhadap tanah akan berubah dengan kecepatan 3 1 m/s ke arah timur.

a. Jika lebar sungai 10 meter, tentukanlah posisi perahu di seberang sungai. Anggaplah posisi awal perahu di titik (0,0)

b. Hitung kecepatan rata-rata aliran sungai antara 0 sampai 2 detik c. Hitunglah kecepatan perahu sampai di seberang d. Tentukanlah kecepatan air hujan yang dirasakan oleh orang yang berada di perahu 4. Benda kecil yang massanya m meluncur sepanjang lintasan licin yang bentuknya seperti pada

gambar 3. Benda dilepaskan dari titik P. a. Tentukan ketinggian minimal P agar benda dapat mencapai satu putaran penuh dalam lintasan

lingkaran b. Dimanakah benda akan berhenti dari titik A pada bidang datar AC, jika bidang tersebut cukup

kasar dengan koefisien gesek kinetik µk? 5. Sebuah balok m = 1 kg diikatkan pada sebuah pegas seperti pada gambar 4, sehingga pegas

bertambah panjang sejauh x(0) = 5 cm dari keadaan awalnya. a. Gambarkan diagram benda bebas dari balok tersebut di titik seimbang b. Tentukan konstanta pegas c. Jika balok ditarik ke bawah sejauh 20 cm dari titik seimbang, tentukanlah kecepatan benda saar

melewati titik seimbang

Page 7: FISIKA TPB
Page 8: FISIKA TPB

SOLUSI UJIAN TENGAH SEMESTER I

FISIKA DASAR I

TAHUN 2002/2003

1. Diketahui : m = 1 kg k = 50 N/m A = 10 cm = 0,1 m

a. Ditanyakan : Fungsi osilasi harmonik Jawab : Perhatikan gambar berikut

Dari gambar di samping dapat terlihat bahwa setelah ujung pegas diberi beban maka pegas akan meregang sejauh y0. Pada keadaan ini sistem berada pada keadaan setimbang sehingga jumlah gaya yang bekerja pada benda sama dengan nol. Dari hukum Newton dapat diperoleh ∑ 0 0

0,2

Apabila pegas diberi simpangan lagi (dengan

cara ditarik ke bawah), maka benda tidak lagi berada dalam keadaan kesetimbangan sehingga benda tersebut akan mengalami gerak harmonik terhadap titik kesetimbangannya. Gerak harmonik tersebut dapat dimodelkan dengan persamaan

cos 0,1 cos√50 Untuk mendapatkan harga , nilai y pada saat 0 dimasukkan ke dalam persamaan di

atas. Kemudian didapatkan 0,1 cos 0 , 0 # 0 0 cos 0 = π/2 Sehingga diperoleh fungsi osilasi harmonik balok adalah 0,1 cos√50 π/2

b. Ditanyakan : kecepatan benda saat y = A/2 Jawab : Fungsi kecepatan balok dapat diperoleh dari turunan pertama fungsi osilasi harmonik

terhadap waktu

&'&( 0,1√50 sin√50 π/2

Simpangan pegas setengah dari simpangan maksimum terjadi pada saat

0,1 cos +√50 ,- 0,05

Cos +√50 ,- 0,5

√50 , ,

/

,0√ s

Page 9: FISIKA TPB

Dengan mensubstitusikan hasil ini ke dalam fungsi kecepatan balok maka akan didapatkan besarnya kecepatan sesaat balok ketika simpangannya mencapai setengah dari simpangan maksimum

0,1√50 sin√50 π/2 , ,0√ s

+ ,0√- 0,1√50 sin +√50 ,

0√ ,-

+ ,0√- 0,1√50 sin +,

0 ,- 0,1√50 sin +,

/ -

Sehingga akan didapatkan kecepatan balok sebesar 0,25√6 m/s 2. Diketahui : m1 = 2 kg µs = 0,2 θ = 37o m2 = 1 kg µk = 0,04

a. Ditanyakan : Diagram benda bebas Jawab :

b. Ditanyakan : Tegangan tali pada saat benda tepat akan bergerak

Jawab : Hukum Newton untuk benda m1 pada arah sumbu-x ∑ 2 0 3 4567 3 89 0 3 3 :9 cos 7 sin 7 Hukum Newton untuk benda m2 pada arah sumbu-x ∑ 2 0 4567 3 89 0 3 :9 cos 7 sin 7 Dengan mensubstitusikan nilai-nilai sesuai dengan yang telah diketahui dari soal, maka didapatkan harga tegangan tali T2 sebesar 4,4 N. Setelah mendapatkan nilai tegangan tali T2, maka nilai tegangan T1 dapat dicari melalui persamaan 3 3 :9 cos 7 sin 7 Sehingga akan didapatkan T1 = 13,2 N

c. Ditanyakan : Percepatan masing-masing benda, jika tali T1 terputus Jawab : Asumsi : Tali T2 yang menghubungkan balok m1 dan m2 tetap tegang sehingga kedua balok akan bergerak dengan percepatan yang sama, ; ; ; Perhatikan diagram benda bebas berikut

Page 10: FISIKA TPB

Hukum Newton untuk balok 1 pada arah sumbu-x menghasilkan ∑ 2 ; 3 4567 8 ; 3 456 7 : cos 7 ; Hukum Newton untuk balok 2 pada arah sumbu-x menghasilkan ∑ 2 ; 4567 3 8 ; 4567 3 : cos 7 ; Dengan mengeliminasi T2 dari kedua persamaan itu maka akan didapatkan 456 7 : cos 7 4567 3 : cos 7 ; ; Sehingga didapatkan ; 4567 :<=47 = 5,68 m/s2 Kemudian perlu dicek kembali apakah asumsi yang digunakan memang benar atau tidak. Untuk menguji hal tersebut akan dihitung percepatan masing-masing balok apabila tali penghubung keduanya tidak tegang (T2=0). Dengan mensubstitusikan nilai ini didapatkan 456 7 : cos 7 ; 4567 : cos 7 ; Sehingga didapatkan ; ; 4567 : cos 7 5,68 m/s2, yang membuktikan asumsi di atas.

d. Ditanyakan : Apakah m1 akan menabrak m2 Kedua balok bergerak dengan kecepatan awal dan percepatan yang sama, sehingga kedua balok tidak akan bertabrakan.

3. Diketahui : Besar kecepatan perahu relatif terhadap air sungai (?@) 2 m/s Besar kecepatan air hujan relatif terhadap tanah (A() 1 m/s @( 3 1 /4 B 10 CDDDE 0

a. Ditanyakan : Posisi perahu di seberang sungai Jawab : Perhatikan gambar berikut

Page 11: FISIKA TPB

Kecepatan perahu relatif terhadap tanah dapat dituliskan ?(DDDDDDE ?@DDDDDDE @(DDDDDDE ?@F @( H 3 1H 2F m/s Waktu yang diperlukan untuk menyeberang adalah

&IJK = 5 s

Posisi perahu setelah sampai di seberang sungai adalah

CE CDDDE L ?(DDDDDDE B 0 L 3 1H 2F

B CE 130H 10F m

b. Ditanyakan : Kecepatan rata-rata aliran sungai Jawab : Kecepatan rata-rata merupakan hasil perpindahan per satuan waktu. Untuk mendapatkan perpindahan aliran sungai dapat dicari dengan mengintegralkan fungsi kecepatan sesaat aliran sungai

@( &2&( 3 1 /4

BM 3 1B # L BM2N2O L 3 1B((O

M( M ∆M / / Untuk selang waktu dari t = 0 s sampai t = 2 s didapatkan ∆M 2/ 2 10 m Sehingga akan didapatkan kecepatan rata-rata aliran sungai

∆2E∆( Q

5H m/s

c. Ditanyakan : Kecepatan perahu ketika sampai di seberang

Jawab : Setelah 5 detik perahu akan sampai di seberang. Pada saat itu perahu memiliki kecepatan ?(DDDDDDE ?@DDDDDDE @(DDDDDDE 3 1H 2F 35 1H 2F 76H 2F m/s

d. Ditanyakan : Kecepatan air hujan yang dirasakan oleh orang yang berada di perahu Jawab : Kecepatan air hujan yang dirasakan oleh orang yang berada di perahu adalah sama dengan kecepatan relatif air hujan terhadap perahu, A?DDDDDDE, yang diberikan oleh A?DDDDDDE A(DDDDDDE ?(DDDDDDE Ambil arah ke atas sebagai sumbu-z positif maka A(DDDDDDE dapat dituliskan A(DDDDDDE A(DDDDDDE STU T Sehingga kecepatan air hujan yang dirasakan oleh orang yang berada di perahu adalah A?DDDDDDE A(DDDDDDE ?(DDDDDDE T 3 1H 2F m/s

4. Diketahui : Benda bermassa m meluncur sepanjang lintasan licin

Page 12: FISIKA TPB

a. Ditanyakan : Ketinggian minimal P agar benda dapat mencapai satu putaran penuh Jawab :

Agar benda tersebut dapat mencapai satu

putaran penuh maka salah satu syarat yang harus dipenuhi adalah benda tersebut memiliki gaya normal minimal yang besarnya nol pada saat benda tersebut berada di titik B (VW X 0 V)

Gaya sentripetal yang bekerja pada benda di titik B adalah

9? V .

Gunakan tinjauan Newton untuk benda yang bergerak melingkar ∑ 9? IYZ sehingga pada

titik B akan diperoleh hubungan V I[YZ

Dengan memasukkan nilai N sebagai syarat (dalam hal ini N=0) diperoleh

I[YZ # W C

Kecepatan yang dimiliki benda berasal dari energi potensial benda di titik awal dilepaskan.

Untuk mencari ketinggian awal minimum kita tinjau persamaan energi sistem dari kedua keadaan sistem yang berbeda

\(](@^ _ \(](@^ W

`ab 2c W ab

`ab 2c c

Sehingga akan kita dapatkan `ab c

b. Ditanyakan : Dimana benda akan berhenti jika terdapat gaya gesek pada bidang datar Jawab : Pada saat benda memasuki lintasan kasar (AC), akan ada usaha oleh gaya gesek sehingga

akhirnya benda kehilangan kecepatannya. Dalam kasus ini berlaku d ∆\

84@2 0

Catatan : usaha akibat gaya gesek kinetik memiliki nilai negatif karena memiliki arah gaya yang

berlawanan dengan arah perpindahan

Nilai energi kinetik berasal dari energi potensial sebesar c sehingga akan didapatkan

c 84@2 : 4@2 atau 4@2 e

fg

5. Diketahui : m = 1 kg x(0) = 5cm = 0,05 m

a. Ditanyakan : Diagram benda bebas pada keadaan setimbang Jawab :

Page 13: FISIKA TPB

b. Ditanyakan : Konstanta pegas

Jawab : Pada keadaan setimbang berlaku ∑ 0 sehingga akan diperoleh M 4567 0

9abh2

Dengan memasukkan nilai-nilai yang telah diketahui akan diperoleh k = 100 N/m

c. Ditanyakan : Kecepatan balok saat melewati titik seimbang Jawab : Karena bidang licin, maka tidak ada energi yang hilang sebagai akibat gesekan dengan permukaan bidang. Untuk seterusnya kita dapat menggunakan prinsip konservasi energi mekanik \i \j?k@9 \jZ@Ia(@9a \i \j?k@9 \jZ@Ia(@9a

0 M 0

M4567

l M 2M4567

Dengan memasukkan nilai-nilai yang telah diketahui akan diperoleh √2 m/s

Page 14: FISIKA TPB

SOAL UJIAN TENGAH SEMESTER I

FISIKA DASAR I

TAHUN 2003/2004 1. Kecepatan sebuah kereta mainan dalam arah sumbu-x

dapat dilihat pada gambar dan dalam arah sumbu-y diberikan oleh fungsi ' 0,5 6m/s.

Posisinya pada 0 adalah CDDDE 2H 6Fm. a. Gambarkan kurva percepatan dalam arah sumbu-x

terhadap waktu b. Hitung besar perpindahannya dari 0s hingga 12s c. Tentukan besar posisinya saat 124 d. Tentukan besar kecepatan rata-rata totalnya dari 0s

hingga 12s

2. Sebuah benda 1 diikat pada sebuah pegas 100mV/ berosilasi harmonik sederhana (dalam bentuk cosines) di atas bidang datar. Diketahui pada saat 04, simpangannya adalah 0,032 m dan kecepatannya 0,24π m/s ke arah sumbu-x negatif. Tentukanlah

a. Simpangan maksimum dan tetapan fase b. Fungsi simpangan benda terhadap waktu c. Kecepatan pada saat simpangan -0.016m 3. Dua buah balok diletakkan pada bidang miring dan

dihubungkan melalui seutas tali melewati sebuah katrol (lihat gambar). Massa balok pertama adalah 10kg dan massa balok kedua 15kg. Koefisien gesekan statik dan kinetik antara balok dan bidang berturut-turut adalah 0,3 dan 0,2. Gesekan antara benda bermassa, katrol dengan tali, kemudian massa tali dan massa katrol dapat diabaikan

a. Tentukan apakah kedua benda tersebut akan diam atau bergerak. Jika bergerak kemana arahnya?

b. Gambarkan diagram gaya untuk masing-masing balok dan berapa besar resultan gayanya saat bergerak?

c. Tentukan tegangan talinya 4. Benda bermassa 0,5 dilepas dari keadaan diam di titik A pada bidang miring kasar :9 0,4 dan : 0,2 seperti pada gambar. Apabila konstanta pegas 200V/ dan massa

pegas diabaikan, tentukanlah : a. Laju benda sesaat sebelum menyentuh pegas b. Simpangan maksimum pegas c. Jarak maksimum benda setelah terpental dari pegas

Page 15: FISIKA TPB

5. Dua buah benda 3 dan 6 dilewatkan pada katrol melalui tali (lihat gambar).

Massa katrol dan tali dapat diabaikan. Diketahui c 1,2 m dan semula diam di atas bidang datar

a. Hitung laju di atas lintasan ketika 7 37] b. Pada sudut minimum berapa dari sumbu vertikal, harus dilepas supaya tepat akan

terangkat dari bidang datar

Page 16: FISIKA TPB

SOLUSI UJIAN TENGAH SEMESTER I

FISIKA DASAR I

TAHUN 2003/2004

1. Diketahui : ' 0,5 6 CDDDE 2H 6F

a. Ditanyakan : Kurva percepatan dalam arah sumbu-x terhadap waktu Jawab : Untuk membuat kurva percepatan dalam arah sumbu-x terhadap waktu, terlebih dahulu akan dihitung percepatan untuk tiap-tiap selang waktu Untuk selang waktu 0 q q 8

;2 ∆Ir∆( IrstIr

st 0 m/s2

Untuk selang waktu 8q q 12

;2 ∆Ir∆( IrtIrs

ts 3 m/s2

Kurva percepatan dalam arah sumbu-x terhadap waktu dapat digambarkan sebagai berikut

b. Ditanyakan : Besar perpindahan dari 0 sampai dengan 124

Jawab : Besar perpindahan dalam arah sumbu-x dapat dihitung dari luas daerah di bawah grafik 2 terhadap pada selang 0 sampai 124

∆M 8 u 6 u 2 u 6

u 2 u 6 48 m

Besar perpindahan dalam arah sumbu-y dapat dihitung dari integral

∆ L 'B L 0,5 6B

36 m Sedangkan besar perpindahan adalah

∆C v∆M ∆ √48 36 60 m

c. Ditanyakan : Posisi pada saat 124 Jawab : CE12 CDDDE ∆CE 2H 6F ∆MH ∆F CE12 2H 6F 48H 36F 50H 30F m

d. Ditanyakan : Besarnya kecepatan rata-rata total dari 0s sampai 12s adalah

Jawab : w ∆Z∆( 0

5 m/s

2. Diketahui : 1kg, 100mN/m # v/ 10m rad/s M0 0,032m dan 0 0,24 m m/s

a. Ditanyakan : Simpangan maksimum dan tetapan fase Jawab : Pada saat tertentu berlaku

M <=4 7 # cos 7 2(_

456 7 # sin 7 I(|_

Lalu digunakan identitas trigonometri untuk mendapatkan besar A sebagai fungsi posisi dan kecepatan sesaat

456 7 <=4 7 +2(_ - +tI(

|_ - 1

Page 17: FISIKA TPB

Sehingga didapatkan

lM0~ I| 0,032 +t,

- 0,04 m

Untuk mendapatkan tetapan fase, kita bagi fungsi sinus dan cosinus yang telah didapatkan dan kita peroleh

;67 IO|2O # 7 ;C<;6 + IO

|2O- 0,64C;B 37]

b. Ditanyakan : Fungsi simpangan terhadap waktu

Jawab : M <=4 7 0,04cos 10m 0,64 m

c. Ditanyakan : Kecepatan saat M 0,016

Jawab : Dari hubungan +2(_ - +tI(

|_ - 1 didapatkan

v M~ 10mv0,04 0,016 1,15 m/s 3. Diketahui : 10 ; 15 ; :9 0,3 ; : 0,2

a. Ditanyakan : Kemungkinan gerak sistem Jawab : Asumsikan sistem akan bergerak ke kanan. Terdapat tiga kemungkinan nilai resultan gaya. Jika nilainya nol, maka sistem akan diam. Jika nilainya positif, maka sistem akan bergerak sesuai dengan asumsi semula, yaitu ke kanan. Sedangkan jika nilainya negatif, maka sistem akan bergerak ke kiri

Perhatikan gambar, 8 dan 8 akan memiliki arah ke kiri karena asumsi gerak sistem adalah ke kanan. Gaya gesek ini merupakan gaya gesek statik. ∑ 45653] 8 8 45637] ∑ 45653] :9 <=453] :9 <=437] 45637] ∑ 120 27 24 60 9 N Dengan mencermati hasil tersebut dapat kita lihat bahwa resultan gaya sesuai dengan asumsi semula, yaitu ke kanan

b. Ditanyakan : Diagram Gaya Jawab : Benda sudah dapat dipastikan bergerak sehingga gaya gesek yang sekarang bekerja adalah gaya gesek kinetik. Sehingga akan didapatkan resultan gaya ∑ 45653] 8 8 45637] ∑ 45653] : <=453] : <=437] 45637] ∑ 120 18 16 60 26V

Page 18: FISIKA TPB

c. Ditanyakan : Tegangan tali

Jawab : Benda 1 dan 2 terhubung oleh tali dan akan memiliki percepatan yang sama. Dengan menerapkan hukum Newton maka akan didapatkan ∑ ;

26 10 15; # ; 0 m/s2

Maka untuk benda 1 didapatkan ∑ ; 3 45637] 8 ; 3 45637] 8 ; 86,4 N Untuk benda 2 didapatkan ∑ ; 3 45653] 8 ; 3 45653] 8 ; 86,4 N

4. Diketahui : 0,5 ; :9 0,4 ; : 0,2 ; 200V/ ; 4 4 ; 7 53]

a. Ditanyakan : Laju benda sebelum menyentuh pegas Jawab : d(](@^ ∆\ Usaha total disebabkan oleh gaya gravitasi dan gaya gesek

84 4 4567 0

: 4 <=47 4 4567

Didapatkan v244567 :<=47 7,38 m/s

b. Ditanyakan : Simpangan maksimum pegas Jawab : d(](@^ ∆\ Usaha total disebabkan oleh gaya gravitasi, gaya pegas, dan gaya gesek

8M M M 4567 0

Catatan : Usaha akibat gaya gesek dan pegas bertanda negatif karena memiliki arah yang berlawanan dengan arah perpindahan

M : <=47 4567

0

100M 3,4M 13,6 0 Dengan menggunakan rumus penyelesaian persamaan kuadrat didapatkan hasil untuk M

M tt/,vt/,Ytt/,0

Maka didapatkan M 0,39 m M 0,35 m (tidak memenuhi)

c. Ditanyakan : Jarak maksimum benda setelah terpental Jawab : d(](@^ ∆\ 84@2

M 4@2 4567 0

Page 19: FISIKA TPB

Catatan : Kecepatan awal dan akhir benda sama dengan nol. Sementara itu benda bergerak ke atas sehingga usaha yang diakibatkan oleh gaya gravitasi berharga negatif

4@2 2Yf]9h9abh 3,3 m

5. Diketahui : 3 ; 6 ; c 1,2

a. Ditanyakan : Laju di dasar lintasan ketika 7 37]

Jawab : Untuk menghitung kecepatan di dasar lintasan dapat digunakan prinsip konservasi energi mekanik. Perhatikan gambar

`_ _ `W

W

`_ `W W

, kecepatan awal sama dengan nol W v2c1 <=47 2,2 m/s

b. Ditanyakan : Sudut minimum agar tepat akan terangkat Jawab : Perhatikan gambar

Tegangan tali pada benda akan maksimum apabila berada pada titik terendah (titik B). Karena bergerak melingkar maka menurut hukum Newton dapat dituliskan

∑ 9? IYZ # 3 I[Y

e

Pada saat benda tepat akan bergerak, gaya normal yang bekerja pada benda tersebut sama dengan nol, sehingga

I[Ye

Karena telah diketahui bahwa W v2c1 <=47 maka didapatkan

vet]9he

2 1 <=47 cos 7 1 Yt

0,5 # 7 60]

Page 20: FISIKA TPB

SOAL UJIAN TENGAH SEMESTER I

FISIKA DASAR I

TAHUN 2004/2005 1. a) Sebuah benda dilempar secara vertikal ke atas dengan laju awal . Bagaimana kecepatan dan

percepatannya saat i. benda mencapai titik tertinggi ii. benda tersebut tepat kembali ke permukaan bumi b) Apakah mungkin terdapat gerak dengan laju tetap yang memiliki percepatan? Jelaskan! 2. a) Sebuah benda bergerak lurus pada suatu bidang mendatar akibat suatu gaya konstan yang

diberikan padanya. Jenis gerak apa yang dihasilkan bila gaya tersebut : i. lebih besar dari gaya gesek ii. sama dengan gaya gesek iii. lebih kecil dari gaya gesek iv. bila tiba-tiba gayanya dihilangkan b) Kapan kita dapat mengatakan bahwa usaha atau kerja adalah negatif dari perubahan energi

potensial dan berikan satu contoh peristiwa yang memenuhi pernyataan di atas c) Sebuah bola dijatuhkan ke lantai. Bola tersebut terpukul hingga mencapai ketinggian semula,

jatuh lagi dan terpantul lagi, demikian terus menerus. Apakah gerak tersebut termasuk gerak harmonik sederhana atau bukan? Jelaskan!

3. Sebuah mobil pembersih jalan bergerak dengan kurva

kecepatan terhadap waktu seperti gambar a. Hitunglah kecepatan rata-rata dari 04 hingga 604 b. Gambarkan kurva percepatan terhadap waktu c. Gambarkan kurva posisi terhadap waktu jika posisi

awalnya 100m

4. Sebuah benda A bermassa _ berada di atas benda B bermassa W. Pada benda B bekerja gaya sebesar yang membentuk sudut dari bidang datar sedemikian sehingga kedua benda bergerak bersama. Koefisien gesek statik dan kinetik antara permukaan benda A dengan benda B serta benda B dengan permukaan lantai dinyatakan oleh :9 dan :

a. Gambarkan diagram gaya pada masing-masing benda b. Berapakah percepatan gerak sistem tersebut 5. Suatu benda terletak di suatu titik pada permukaan bumi khatulistiwa (equator). Jari-jari bumi 6,37 u 100 meter. Misalkan pengaruh gerak bumi terhadap matahari serta gerak matahari terhadap

pusat galaksi bima sakti diabaikan a. Hitung kecepatan sudut benda tersebut b. Hitung pula kecepatan linearnya c. Percepatan apakah yang dialami benda tersebut dan berapa besarnya 6. Sebuah benda bermassa 50 gr dihubungkan pada ujung pegas mendatar yang massanya dapat

diabaikan. Benda ditarik mendatar sejauh 10 cm kemudian diepaskan sehingga berosilasi harmonik sederhana dengan frekuensi sudut 4π rad/s

a. Tentukan besarnya nilai konstanta pegas b. Tentukan kecepatan gerak sistem ketika benda pada posisi 2 cm dari posisi setimbangnya

Page 21: FISIKA TPB

c. Tentukan besarnya percepatan maksimum dari gerak tersebut 7. Suatu mainan kereta luncur bermassa memiliki lintasan yang terdiri dari lengkungan, lingkaran

serta garis lurus seperti pada gambar. Lintasan dianggap licin kecuali bagian sepanjang BD (dengan jarak d) dengan koefisien gesek kinetik µ. Di bagian ujung (titik D) dipasang pegas dengan massa yang dapat diabaikan

a. Nyatakan dalam R (jari-jari lingkaran) ketinggian minimum ab kereta luncur agar dapat melalui lintasan lingkaran dengan sempurna

b. Jika keadaan pada soal (a) terpenuhi, tentukan kecepatan di B (dalam R) dan perubahan energi potensial pegas saat tertekan maksimum terhadap titik setimbangnya dalam µ, g, R, m, dan d

Page 22: FISIKA TPB

1. Diketahui : benda memiliki laju awal a. Ditanyakan : Kecepatan dan percepatan benda Jawab : (i) di titik tertinggi (ii) di titik semula b. Ditanyakan : Mungkinkah terdapat gerak dengan laju tetap yang memiliki percepatan? Jawab : Sangatlah mungkin ada gerak dengan laju konstan

tetapi memiliki perubahan berubah. Maka dalam kasus ini akan muncul percepatangerak melingkat beraturan yang memiliki laju tetap tetapi memiliki percepatan sentripetal yang arahnya selalu menuju ke pusat lingkaran.

2. a. Ditanyakan : Jenis gerak yang dihasilkan Jawab :

Misalkan gaya konstannya adalah (i) 8, benda bergerak dipercepat(ii) 8, benda bergerak dengan kecepatan konstan(iii) 8, benda tidak mengalami perlambatan(iv) tiba-tiba dihilangkan, gerak diperlambat

b. Ditanyakan : Contoh peristiwa dimana usaha dan kerja adalah negatif dari perubahan energi

potensial Jawab : d ∆ jika gayanya bersifat konservatif contoh : gaya gravitasi, gaya pegas, gaya listrik

c. Ditanyakan : Jenis gerak bola jatuh ke lantaiJawab : Gerak harmonik simpangan. Berdasarkan hal tersebut dapat kita simpulkan bahwa kasus tersebut bukanlah termasuk gerak harmonik sederhana kayaitu ke bawah, tidak peduli arah simpangan benda

3. Diketahui : Kurva kecepatan mobil pembersih jalan diberikan a. Ditanyakan : Kecepatan rata

Jawab : Z@(@tZ@(@ b. Ditanyakan : Kurva percepatan terhadap waktu Jawab :

untuk 0 q q 20, ; untuk 20 q q 50 untuk 50 q q 60 c. Ditanyakan : Kurva posisi terhadap waktu Jawab :

SOLUSI UJIAN TENGAH SEMESTER I

FISIKA DASAR I

TAHUN 2004/2005

benda memiliki laju awal

Kecepatan dan percepatan benda

0 dan ; dan ;

Mungkinkah terdapat gerak dengan laju tetap yang memiliki percepatan? Sangatlah mungkin ada gerak dengan laju konstan

memiliki perubahan arah sehingga vektor kecepatan berubah. Maka dalam kasus ini akan muncul percepatan. Seperti gerak melingkat beraturan yang memiliki laju tetap tetapi memiliki percepatan sentripetal yang arahnya selalu menuju ke

Jenis gerak yang dihasilkan

Misalkan gaya konstannya adalah , benda bergerak dipercepat , benda bergerak dengan kecepatan konstan , benda tidak mengalami perlambatan tiba dihilangkan, gerak diperlambat

Contoh peristiwa dimana usaha dan kerja adalah negatif dari perubahan energi

jika gayanya bersifat konservatif contoh : gaya gravitasi, gaya pegas, gaya listrik

Jenis gerak bola jatuh ke lantai Gerak harmonik dicirikan oleh arah percepatan yang selalu berlawanan dengan arah

simpangan. Berdasarkan hal tersebut dapat kita simpulkan bahwa kasus tersebut bukanlah termasuk gerak harmonik sederhana karena percepatan gravitasi hanya memiliki satu arah saja yaitu ke bawah, tidak peduli arah simpangan benda

Kurva kecepatan mobil pembersih jalan diberikan

Kecepatan rata-rata dari 0 sampai 60 s

∆2∆( ^@9 Z@a &a @@A ZI@ It(

09 30 m/s

Kurva percepatan terhadap waktu

; 2 m/s2

50, ; 0 m/s2

60, ; t 4 m/s2

Kurva posisi terhadap waktu

Mungkinkah terdapat gerak dengan laju tetap yang memiliki percepatan?

Contoh peristiwa dimana usaha dan kerja adalah negatif dari perubahan energi

dicirikan oleh arah percepatan yang selalu berlawanan dengan arah simpangan. Berdasarkan hal tersebut dapat kita simpulkan bahwa kasus tersebut bukanlah

rena percepatan gravitasi hanya memiliki satu arah saja

Page 23: FISIKA TPB

M M L B( M

;

untuk 0 q q 20, M 500 m untuk 20 q q 50, M 1700 m untuk 50 q q 60, M 1900 m 4. Diketahui : Sistem dua benda diberikan gaya dengan sudut tertentu a. Ditanyakan : Diagram benda bebas Jawab :

b. Ditanyakan : Percepatan gerak sistem Jawab : Tinjau sistem massa A dan B dalam arah vertikal 456 VW,^@b(@a VW_ dW V_W d_ 0 V_W VW_, sehingga saling menghilangkan VW,^@b(@a dW d_ 456 Dalam arah horizontal ∑ 89 9;9 # <=4 89,W_ 8W 89,_W 9;9

;9 ]9tg[[ ]9tfg[,KNK

[ ]9tfg[t9ab[

5. Diketahui : Suatu benda terletak di permukaan bumi khatulistiwa a. Ditanyakan : Kecepatan sudut Jawab : Karena rotasi bumi, benda di khatulistiwa bergerak melingkar dengan jari-jari sama

dengan jari-jari bumi dengan periode 24 jam. Benda berada di suatu titik (kecepatan sudutnya sama dengan rotasi bumi)

2m8 7,27 u 10t rad/s

b. Ditanyakan : Kelajuan linear Jawab : c 464 m/s c. Diketahui : Percepatan yang dialami benda

Jawab : ; ;9kb(Za?k(@^ IYe 33,7 u 10t m/s2

Benda tersebut juga mengalami percepatan gravitasi sebesar 9,8 m/s2 6. Diketahui : 5gr ; 0,1 m ; 4m rad/s

Page 24: FISIKA TPB

a. Ditanyakan : Tetapan pegas Jawab : Pada kasus osilasi pegas ini, resultan gaya yang bekerja pada benda merupakan gaya

pemulih. Jika gerak pegas dimodelkan sebagai gerak harmonik sederhana maka ∑ ; # M ; <=4 <=4 0,0516m 8 N/m b. Ditanyakan : Kelajuan saat simpangan 2 cm Jawab : M <=4 0,02 0,1<=4 <=4 0,2 # 456

√6

Maka 456 4m0,1 √6 ,s

√6 m/s

c. Ditanyakan : Percepatan maksimum Jawab : ; 1,6m m/s2 7. Diketahui : Kereta luncur melewati suatu lengkungan, lintasan melingkar, lintasan lurus kasar,

dan pada akhirnya menumbuk pegas

a. Ditanyakan : Ketinggian minimum agar kereta luncur dapat melalui lintasan lingkaran

Jawab : Agar dapat sampai di C, gaya gravitasi harus cukup menyediakan gaya sentripetal

IYe , gaya normal di C sama dengan nol

vc Dari prinsip konservasi energi didapatkan \i_ \j_ \iW \jW

@ `@

`

Dengan memasukkan nilai yang telah diperoleh dan kondisi diam di titik A (@ 0)

didapatkan ab 2,5c b. Ditanyakan : Kecepatan di B dan perubahan energi potensial pegas saat tertekan maksimum Jawab : Prinsip konservasi energi mekanik, \_ \W

2,5c W # W v5c

Pada saat tertekan maksimum d(](@^ ∆\i 8B

M 0 v5c

M 2,5 c : B

∆?k@9 2,5c :B

Page 25: FISIKA TPB

SOAL UJIAN TENGAH SEMESTER I

FISIKA DASAR I

TAHUN 2005/2006 1. Sebuah benda suatu saat berada di posisi CE 2H 3F m. Benda tersebut mengalami perpindahan

selama 10 detik mengikuti hubungan ∆CE 8H 10F m. a. Tentukan besar perpindahan dari 04 sampai 104 b. Tentukan posisi benda tersebut pada saat 104 c. Gambarkan grafik kecepatan dan percepatan sebagai fungsi waktu untuk komponen dalam

sumbu-y, mulai dari saat 04 sampai 104 2. Gerak sebuah benda dalam arah sumbu-x sebagai fungsi dari waktu ditunjukkan oleh gambar di

bawah a. Tentukan panjang lintasan pada saat 2, 4, 8, 10, dan 124 b. Tentukan waktu pada saat benda berhenti c. Tentukan kecepatan rata-rata dari 04 sampai 124

3. Sebuah benda dengan massa 25kg pada bidang miring dengan

kemiringan sebesar 30]. Benda tersebut dari keadaan diam didorong dengan gaya horisontal sebesar & 20V dan ditarik ke atas dengan gaya sebesar ( 40V pada arah 37] terhadap bidang miring seperti terlihat pada gambar. Bidang miring tersebut memiliki koefisien gesek statik dan kinetik berturut-turut 0,3 dan 0,2. Gunakan g = 10 m/s2.

a. Gambarkan diagram gaya yang bekerja pada benda b. Tentukan besar gaya dan arah percepatan gerak benda c. Tentukan harga kecepatan benda setelah menempuh 2m 4. Tinjaulah sistem benda pada gambar. Diketahui bahwa koefisien gesek statik dan kinetik antara dan berturut-turut adalah 0,4 dan 0,2. Diketahui 2 , 3 , dan / 1 .

Gunakan g=10m/s2.

0

2

4

6

8

10

12

14

0 1 2 3 4 5 6 7 8

t[det]

x(t)[m

]

Page 26: FISIKA TPB

a. Buat diagram gaya untuk , , dan / b. Tentukan besar percepatan pada soal (a) c. Tentukan besar tegangan tali T1 dan T2

5. Suatu gaya luar bekerja pada benda bermassa 0,2

kg sehingga menekan pegas berkonstanta pegas 1200N/m sejauh M dari titik setimbang. Kemudian gaya luar tersebut dilepaskan sehingga benda menempuh lintasan horisontal , dan lintasan melingkar seperti pada gambar. Lintasan dan adalah bidang licin sedangkan lintasan kasar dengan koefisien gesek kinetik 0,2. Gunakan g=10m/s2.

a. Tentukan simpangan minimal pegas agar benda dapat mencapai titik D

b. Tentukan besar dan arah percepatan benda di titik C saat kondisi soal (a) dipenuhi c. Setelah mencapai D, benda kemudian bergerak parabolik. Tentukan jarak jatuhnya bola terhadap

titik B

6. Sebuah balok, bermassa 1kg diikatkan pada ujung sebuah pegas yang memiliki konstanta pegas sebesar 120 N/m dan diletakkan pada lantai licin. Di atas benda tersebut diletakkan balok lain, , bermassa 0,5 kg. Dari keadaan setimbangnya pegas ditarik sejauh 5 cm kemudian dilepaskan. Selanjutnya sistem dua benda tersebut bergerak osilasi harmonik.

a. Tentukan syarat yang harus dipenuhi agar kedua benda tersebut dapat bergerak osilasi harmonik b. Tentukan periode osilasi sistem tersebut c. Tentukan kecepatan osilasi pada saat simpangan osilasi 2 cm

Page 27: FISIKA TPB

SOLUSI UJIAN TENGAH SEMESTER I

FISIKA DASAR I

TAHUN 2005/2006 1. Diketahui : CE 2H 3F m ∆CE 8H 10F m

a. Ditanyakan : Perpindahan dari 0 sampai 10 s Jawab : ∆CE10 810H 1010F 80H 1000F m

b. Ditanyakan : Posisi benda pada saat 10 s Jawab : CE CDDDE ∆CE CE10 CDDDE ∆CE10 2H 3F 80H 1000F 82H 1003F m

c. Ditanyakan : Grafik kecepatan dan percepatan untuk komponen y Jawab : Dari persamaan CE CDDDE ∆CE 8 2H 10 3F, diperoleh komponen

perpindahan dalam arah y adalah 10 3. Kecepatan dalam arah sumbu-y : ' &'&(

20 m/s Grafik ' terhadap adalah

Percepatan dalam arah sumbu-y :

;' &I&( 20 m/s2

Grafik ;' terhadap adalah

2. Diketahui : Diberikan kurva posisi sebuah benda dalam sumbu-x terhadap waktu a. Ditanyakan : Panjang lintasan

Jawab : Berdasarkan grafik yang diberikan pada soal didapatkan panjang lintasan (seluruh lintasan ditambahkan)

Pada saat 2 s adalah 8 m; Pada saat 4 s adalah 13 m; Pada saat 8 s adalah 19 m; Pada saat 10 s adalah 20 m; Pada saat 12 s adalah 28 m b. Ditanyakan : Waktu pada saat benda berhenti Jawab : Benda berhenti ditandai oleh suatu waktu pada saat gradien grafik M sama dengan

nol Sehingga benda berhenti pada saat 2, 3, 6, 8, 10, dan 12 s c. Ditanyakan : Kecepatan rata-rata dari 0 sampai 12 s Jawab :

Page 28: FISIKA TPB

w 2t2t

/ m/s

3. Diketahui : 25 kg ; & 20 N ; :9 0,3 ; : 0,2 a. Ditanyakan : Diagram benda bebas Jawab : Asumsi : Benda bergerak ke bawah dengan percepatan ;

b. Ditanyakan : Besar dan arah percepatan benda Jawab : Dari diagram benda bebas di atas, kita dapat menuliskan hukum Newton

- Dalam arah sumbu-x ∑ 2 ; 45630] &<=430] (<=437] 8 ;

- Dalam arah sumbu-y ∑ ' 0 V (45637] <=430] &45630] 0

Gaya gesek kinetik diberikan oleh persamaan 8 :V. Dengan menggunakan persamaan sebelumya didapatkan V <=430] &45630] (45637] Maka, 8 :V 8 : <=430] &45630] (45637] Setelah memasukkan nilai-nilai yang telah diketahui sebelumnya maka didapatkan 8 40,5 V Dengan menggunakan persamaan resultan gaya pada sumbu-x akan didapatkan percepatan

; 45630] &<=430] (<=437] 8 1,41 m/s2

Karena percepatan yang didapatkan bernilai positif, maka hal ini mengindikasikan bahwa asumsi yang dipakai sebelumnya memang benar. Jadi benda akan bergerak turun dengan percepatan sebesar 1,41 m/s2.

c. Diketahui : Kecepatan benda setelah menempuh jarak 2 m

Jawab : ( 2;4 5,64; ( v5,64 2,37 m/s 4. Diketahui : :9 0,4 ; : 0,2 ; 2 ; 3 kg ; / 1 kg a. Ditanyakan : Diagram gaya Jawab :

Page 29: FISIKA TPB

b. Ditanyakan : Besar percepatan Jawab : Dari gambar di atas dapat disimpulkan bahwa benda diam (karena ditahan oleh

tali T1), sedangkan benda dan benda / bergerak dengan percepatan yang sama, ; ;/ ;. Kita terapkan hukum Newton untuk masing-masing benda dan kita dapatkan Untuk benda 1 : Untuk benda 2 : Untuk benda 3 :

' 0 # V

2 ;

3 8 ; 3 : ;

; 0 ' ;

/ 3 ;

Dengan mengeliminasi T2 dari persamaan sebelumnya akan didapatkan

; ¡ / : 2 m/s2

Sehingga diperoleh percepatan benda 1 dan benda 3 adalah sama besar yaitu 2 m/s2 dan

percepatan benda 2 adalah nol. c. Ditanyakan : Besar tegangan tali T1 dan T2 Jawab : Dari persamaan hukum Newton untuk benda 2 akan diperoleh ∑ 2 0 3 8 0 3 : 4 N Dari persamaan hukum Newton untuk benda 3 akan diperoleh ∑ ' ; / 3 ; 3 / ; 6 N 5. Diketahui : 0,2 kg ; 1200 N/m ; : 0,2 ; c 1 m ; 4 2,5 m a. Ditanyakan : Simpangan pegas minimum agar benda dapat mencapai titik D Jawab : Perhatikan gambar berikut Karena sepanjang lintasan BCD benda bergerak melingkar, maka pada titik D berlaku 9?

V I¢Ye . Agar dapat mencapai titik D, gaya normal di titik tersebut harus lebih besar atau

sama dengan nol V X 0, sehingga kecepatan minimum di titik D adalah £ c (yaitu ketika gaya normalnya nol).

Page 30: FISIKA TPB

Karena lintasan BCD licin maka kita dapat menggunakan prinsip konservasi energi mekanik. Dengan mengambil titik B sebagai acuan (energi potensial di titik B bernilai nol), akan diperoleh

\iW \i£ \j£ £ `£

\iW c 2c

c

Pada lintasan AB berlaku \i_ dk9k \iW

\i_ 84 c :

c

Lintasan licin sehingga berlaku konservasi energi \j_ \i_

M :

c : 2,5c

Maka simpangan minimum, M lfge 0,095 m

b. Ditanyakan : Besar dan arah percepatan di titik C Jawab : Berdasarkan gambar di samping, dapat dihitung besar percepatan di titik C

; l;9? l+I¤Ye -

Untuk mencari kita dapat menggunakan prinsip konservasi energi

dengan mengambil titik C sebagai acuan (energi potensial di C bernilai nol), sehingga

\i¥ \i£ \j£

¥

£ `£

¥ £ 2c c 2c 3c Substitusikan hasil ini ke persamaan percepatan maka akan didapatkan

; l+I¤Ye - l+/e

e - √10 10√10 m/s2

Untuk menentukan arah percepatan di titik C, perhatikan gambar berikut tan 7 @§J

3 7 ;6t3 71,56]

c. Ditanyakan : Jarak jatuhnya bola setelah melewati titik D Jawab : Perhatikan gambar berikut

Benda akan bergerak dengan lintasan parabola, untuk itu akan kita analisis gerak untuk masing-

masing komponen (komponen-x dan komponen-y)

Page 31: FISIKA TPB

Untuk komponen-x berlaku gerak lurus beraturan, B £ Untuk komponen-y berlaku gerak lurus berubah

beraturan, 2c

Substitusikan yang diperoleh dari komponen-y ke persamaan komponen-x, sehingga kita dapatkan

B 2£le 2cle

2c 2 m

Sehingga benda akan jatuh pada posisi 2 meter di sebelah kiri titik B. 6. Diketahui : 1 kg ; 120 N/m ; 0,5 kg ; 0,05 m a. Ditanyakan : Syarat yang harus dipenuhi agar kedua benda dapat bergerak osilasi harmonik Jawab : Agar balok dan dapat bergerak bersama, maka gaya gesekan statis antara kedua

balok tersebut harus lebih besar atau sama dengan gaya pegas maksimum 89 X # :9 X

:9 X _Y 1,2

Jadi syarat yang harus dipenuhi agar kedua benda dapat bergerak osilasi harmonik adalah

koefisien gesek statik antara kedua benda tersebut haruslah lebih besar atau sama dengan 1,2 b. Ditanyakan : Periode osilasi sistem Jawab :

3 2mlY 2ml,

0,70 s

c. Ditanyakan : Kecepatan osilasi pada saat simpangan 2 cm Jawab : Dengan menggunakan prinsip konservasi energi didapatkan

Sehingga didapatkan

l Y l

, 0,05 0,02 0,41 m/s

Page 32: FISIKA TPB

SOAL UJIAN TENGAH SEMESTER I

FISIKA DASAR I

TAHUN 2006/2007 Gunakan 9,80 /4 1. Seorang tentara tengah berlatih menembak dari ketinggian 10 m di atas tanah. Tentara tersebut

menembakkan peluru dengan laju 100 m/s dan sudut elevasi ;6 3/4

a. Tentukanlah posisi peluru sebagai fungsi waktu dengan menganggap bahwa peluru ditembakkan pada saat 04. Gambarkan secara skematik koordinat x-y yang saudara pilih dan tentukan posisi peluru pada saat 14

b. Berapakah ketinggian maksimum yang dapat dicapai peluru pada saat 14

c. Pada saat berapakah peluru mencapai tanah 2. Balok A dan balok B ( _ 5 dan W 1) terhubung dengan seutas tali melalui katrol tidak

bermassa. Anggap tali dan katrol tidak bermassa. Balok A berada di atas bidang miring kasar

dengan sudut kemiringan 7 ;67 /. Koefisien gesekan bidang miring adalah :9

0,25 dan : 0,2; sedangkan koefisien gesekan statik bidang vertikal adalah :9 0,25 a. Mula-mula balok B ditahan dengan cara member gaya horisontal pada balok B, sehingga

balok B menekan bidang vertikal. Dalam kondisi ini kedua balok adalah setimbang (diam). Gambarkan diagram gaya benda bebas balok A dan balok B

b. Berapa besar gaya minimum agar sistem dalam keadaan diam. Hitung besar gaya normal pada balok B dan ke mana arahnya

c. Jika gaya dilepas, hitung besar percepatan balok (anggap balok B dengan dinding vertikal tidak ada gaya gesekan)

3. Sebuah benda bermassa 1 kg mula-mula berada di titik A (lihat

gambar). Pada benda tersebut dikenakan gaya E H MF N, benda bergerak menuju titik C dengan 2 jalur yang ditempuh, yaitu A(-2,0) B(-2,8) C(6,8) dan A D(6,0)

C a. Hitunglah usaha yang dibutuhkan untuk bergerak di masing-

masing jalur b. Apakah gaya konservatif? Jelaskan pendapat saudara c. Ketika benda menempuh jalur AB, tentukanlah besar

kecepatan benda di titik (-2,4), jika pada saat awal kecepatan benda adalah nol

4. Sebuah benda bermassa 0,1 kg dihubungkan dengan sebuah pegas dengan konstanta pegas 10 N/m.

Benda ditempatkan pada bidang datar yang licin sempurna. Benda ditarik sehingga pegas memanjang 10 cm, dan kemudian benda dilepaskan pada 04

a. Dengan menggunakan hukum II Newton, turunkan persamaan diferensial osilasi harmonik dari benda tersebut

b. Tentukan solusi dari persamaan diferensial di atas dinyatakan dalam fungsi sinus c. Tentukanlah energi kinetik pada saat simpangannya 0,4 kali besar amplitude

Page 33: FISIKA TPB

5. Dalam permainan softball, seorang pelempar bola melemparkan bola (massa 250 gr) dengan kecepatan 30F m/s. Bola dipukul sehingga kecepatannya berubah menjadi 10H 20F 20T m/s

a. Hitunglah impuls pada bola b. Jika tumbukan bola dengan pemukul terjadi selama 0,01 detik, hitung besar gaya rata-rata pada

bola

Page 34: FISIKA TPB

SOLUSI UJIAN TENGAH SEMESTER I

FISIKA DASAR I

TAHUN 2006/2007 1. Diketahui : ` 10 m ; 100 m/s ; tan 3/4 a) Ditanyakan : Posisi peluru sebagai fungsi waktu Jawab : Keadaan awal : posisi dan kecepatan awal CE0 10F m E0 100 cos H 100 sin F 80H 60F m/s

Mengingat E &ZE&( dan ;E F 9,8 /4 maka

CE CE0 E0 ;E

CE 10F 80H 60F 4,9F 80H 10 60 4,9F Pada saat 1 diperoleh CE1 801H 10 601 4,91F 80H 65,1F b) Ditanyakan : Ketinggian maksimum peluru saat 1 s Jawab : Ketika peluru ditembakkan ke atas, pada ketinggian maksimumnya (sesaat sebelum berganti

arah), kecepatan peluru dalam arah-y adalah nol sehingga ' '0 ;' Substitusi ' 0 ; ;' ; ' sin didapatkan

I ¨©ª 6,1224

0

Sehingga ketinggian maksimum adalah

0 10 IY9abY

193,67 m

c) Ditanyakan : Kapan peluru mencapai tanah Jawab : Ketika mencapai tanah, 0 Dari jawaban poin (a) didapatkan 10 60 4,9 0

maka t0v0Ytt,«t,« 6,122 ¬ 6,287

Sehingga yang memenuhi hanyalah 12,409 s 2. Diketahui : _ 5 kg ; W 1 kg ; tan 7 3/4 ; :9 0,25 ; : 0,2 a) Ditanyakan : Diagram gaya Jawab :

Page 35: FISIKA TPB

Karena sistem dalam keadaan diam dan memiliki kecenderungan gerak ke kanan maka 89_ memiliki arah ke kiri dan 89W memiliki arah ke bawah b) Ditanyakan : minimum agar sistem diam Jawab : minimum dicapai ketika 89_ maksimum dan 89W maksimum

Untuk benda A Untuk benda B

_ 0

_ sin 7 3 89_ 0 _ sin 7 3 :9 _ cos 7 0 3 _sin 7 :9 cos 7

W 0

3 89W W 0 3 :9VW W 0 , VW 3 :9 W 0

maka t[f§

Dengan substitusi pada tegangan tali didapatkan

¨©ª htf§ ²³¨ ht[f§ 39,2 V

c) Ditanyakan : Percepatan balok jika dilepas Jawab : Jika dilepas maka tidak ada gaya gesek antara benda B dengan dinding vertikal ∑ ∑ ; _ sin 7 8_ W _ W; _ sin 7 : _ cos 7 W _ W;

; ¨©ª htfg ²³¨ ht[[ 1,96 m/s2

3. Diketahui : 1 kg ; E H MF ; Jalur tempuh A-B-C dan A-D-C a) Ditanyakan : Usaha pada masing-masing jalur Jawab : E H MF ; BCE BMH BF ; E · BCE BM MB Jalur ABC

d L E · BCE L BM MB¥_ L BM MBW

_ L BM MB¥W

Karena pada lintasan AB tidak ada perubahan x dan pada lintasan BC tdak ada perubahan y, maka elemen dx pada proses integrasi lintasan AB sama dengan nol dan elemen dy pada proses integrasi lintasan BC juga sama dengan nol. Integral di atas kemudian menjadi

d L MBW_ L BM¥

W Pada integrasi lintasan AB terjadi perubahan titik koordinat-y dari 0 ke 8 dengan x konstan

di titik -2, dan pada integrasi lintasan BC terjadi perubahan titik koordinat-x dari -2 ke 6 dengan y konstan di titik 8. Dengan demikian

d L 2Bs L 8BM0

t 80 J

Page 36: FISIKA TPB

Jalur ADC

d L E · BCE L BM MB¥_ L BM MB£

_ L BM MB¥£

Karena pada lintasan AD tidak ada perubahan y dan pada lintasan DC tdak ada perubahan x, maka elemen dx pada proses integrasi lintasan AD sama dengan nol dan elemen dy pada proses integrasi lintasan DC juga sama dengan nol. Integral di atas kemudian menjadi

d L MBW_ L BM¥

W Pada integrasi lintasan AD terjadi perubahan titik koordinat-x dari -2 ke 6 dengan y konstan

di titik 0, dan pada integrasi lintasan DC terjadi perubahan titik koordinat-y dari 0 ke 8 dengan x konstan di titik 6. Dengan demikian

d L 0BM0t L 6Bs

48 J b) Ditanyakan : Apakah konservatif Jawab : Sifat gaya tidak konservatif karena d_W¥ µ d_£¥ c) Ditanyakan : Besar kecepatan di titik (-2,4)

Jawab : d L E · BCE L BM MBt,t,

Karena tidak ada perubahan titik koordinat-x maka dx=0 sehingga persamaan tersebut menjadi

d L MB L 2B

8 J

d 8 # 4 m/s2

4. Diketahui : 0,1 kg ; 10 N/m ; ∆M 10 cm a) Ditanyakan : Persamaan gerak Jawab : Gunakan persamaan diferensial untuk hukum Newton

; &Y2&(Y ,M &Y2

&(Y

&Y2&(Y

M 0 ,&Y2&(Y M 0 , dengan l

b) Ditanyakan : Solusi persamaan diferensial Jawab : Salah satu solusi persamaan diferensial di atas adalah dalam bentuk sinus

M sin dengan l l

, 10 C;B/4 dan 0,1 m

Sehingga solusinya menjadi M 0,1 sin10 Pada saat 04, M 0,1 0,1 0,1 sin100 sin 1 #

Sehingga bentuk persamaan lengkapnya adalah M 0,1 sin10

c) Ditanyakan : Energi kinetik saat simpangan 0,4A Jawab : Gunakan prinsip konservasi energi

M

\i

M 0,042 J

5. Diketahui : 0,25 kg ; E 30F m/s ; E@ 10H 20F 20T m/s

Page 37: FISIKA TPB

a) Ditanyakan : Impuls bola Jawab : Gunakan hubungan impuls-momentum ¶E ∆jDE jDE@AaZ jDE@@^ ¶E E@AaZ E@@^ ¶E 0,25·S10H 20F 20TU 30F¸ 2,5H 12,5F 5T Ns b) Ditanyakan : Gaya rata-rata pada bola Jawab : Gunakan hubungan impuls-gaya ¶E E · ∆

E ¹E∆( ,Qt,ºT

, 250H 1250F 500T

Maka »E» v250 1250 500 1369,3 N

Page 38: FISIKA TPB

SOAL UJIAN TENGAH SEMESTER I

FISIKA DASAR I

TAHUN 2007/2008

1. Sebuah benda bergerak sepanjang lintasan OPA (lurus) dan ABO (setengah lingkaran) seperti tampak pada gambar di samping. Besar kecepatan benda di setiap titik lintasan adalah 5 m/s. Tentukanlah : a. Vektor posisi benda ketika berada di titik P dan ketika

berada di titik B b. Waktu yang diperlukan benda untuk bergerak dari titik P ke

titik B c. Vektor kecepatan rata-rata benda untuk bergerak dari titik P ke titik B d. Laju rata-rata benda untuk bergerak dari titik P ke titik B

2. Sebuah balok dengan massa 1 kg pada bidang horisontal licin dihubungkan dengan sebuah pegas

(massa pegas diabaikan) sehingga berosilasi sejajar bidang tersebut (sumbu-x) dengan frekuensi osilasi 2 Hz. Jika pada 14 benda berada di M 5√3 u 10t m dan 20m u 10t m/s. Tentukan :

a. Konstanta pegas dan amplitude b. Simpangan sebagai fungsi waktu 3. Sebuah benda P ditekankan pada sebuah pegas (konstanta pegas 50 N/m) sehingga pegas

memendek 5 u 10t m (lihat gambar di bawah). Ketika tekanan dilepas, benda P bergerak dan menumbuk benda Q, yang mula-mula diam, secara elastik sempurna. Akhirnya benda Q berhenti di titik C. Bidang datar AB licin dan bidang datar BC kasar dengan koefisien gesek kinetik antara benda dan bidang 0,2. Kedua benda mempunyai massa yang sama sebesar 2 kg. Tentukanlah :

a. Kecepatan P sesaat sebelum menumbuk Q b. Kecepatan P dan Q sesaat sesudah tumbukan c. Jarak BC

4. Perhatikan gambar di samping. Massa katrol dan tali diabaikan dan panjang

tali tetap. Koefisien gesek antara benda A dan benda B adalah : 0,2 dan : 0,1. Jika lantai licin dan massa masing-masing benda adalah _ 3 ; W 5 ; ¥ 0,3, maka a. Gambarkan diagram gaya benda bebas pada masing-masing benda A, benda

B, benda C b. Periksalah apakah benda A dan B bergerak bersama c. Hitunglah percepatan untuk masing-masing benda B dan benda C

5. Sebuah gaya 4M5 (untuk Fisika IB, gaya 35) Newton bekerja pada

sebuah benda yang bermassa dengan lintasan seperti ditunjukkan pada gambar di samping

a. Tentukan kerja yang dilakukan gaya tersebut untuk perpindahan benda dari titik O ke titik B :

i. dengan lintasan 1 (OB) ii. dengan lintasan 2 (OAB) b. Apakah gaya di atas konservatif? Jelaskan

Page 39: FISIKA TPB

SOLUSI UJIAN TENGAH SEMESTER I

FISIKA DASAR I

TAHUN 2007/2008 1. Diketahui : 5 m/s ; Lintasan OPA dan ABO a. Ditanyakan : Vektor posisi benda ketika berada di titik P dan B Jawab : Dari grafik terhadap M dapat diketahui CE? 10H m CEW 10H 10F m b. Ditanyakan : Waktu yang diperlukan dari titik P ke B

Jawab : ¼W ¼_ _W ½¾I ½[

I ,·

5,14 s

c. Ditanyakan : Vektor kecepatan rata-rata

Jawab : w Z[tZ¾(¾[ QºtQ

, 1,95F m/s

d. Ditanyakan : Laju rata-rata

Jawab : Z@(@tZ@(@ 9N¿NK(N¿NK ½¾½[

(¾[ 5 m/s

2. Diketahui : 1 kg ; 8 2 Hz M 1 5√3 u 10t m ; 1 20m u 10t m/s a. Ditanyakan : Konstanta pegas

Jawab : 8 l

# 4m8 158 N/m

Amplitudo:

M # lIY2Y

Dengan memasukkan nilai-nilai variabel yang telah diketahui pada 14 maka akan

didapatkan A = 0,1 m b. Ditanyakan : Simpangan sebagai fungsi waktu Jawab : M cos M 0,1 cos4m Substitusi nilai M 5√3 · 10t m ketika 14 5√3 · 10t 0,1 cos4m cos4m1 cos 0,5√3 Maka didapatkan ¬

0

3. Diketahui : 50 N/m ; ∆M 5 u 10t m ; 2 kg ; : 0,2 a. Ditanyakan : Kecepatan P sebelum menumbuk Q Jawab : Akan terjadi perubahan bentuk energi dari energi potensial pegas menjadi energi

kinetik ketika pegas tertekan tersebut dilepas

Page 40: FISIKA TPB

M

Ml 5 · 10tl

25 · 10t m/s

b. Ditanyakan : Kecepatan setelah tumbukan Jawab : Hukum kekekalan momentum , , 0 Gunakan hubungan koefisien restitusi À ItIY

ItIY

Dengan memasukkan nilai À 1 (elastik sempurna), 0 atau Substitusi menghasilkan 2 2 atau 25 · 10t /4 Dengan mensubstitusikan hasil tersebut didapatkan 0 c. Ditanyakan : Jarak BC Jawab : Ketika menempuh jarak BC, akan bekerja usaha oleh gaya gesek sehingga benda

akhirnya berhenti. d ∆\i

8 · 4 @AaZ

@@^ ; @AaZ 0 ; 8 :

: 4 0

Didapatkan 4

IKÁKYf 0,0156 m

4. Diketahui : :9 0,2 ; : 0,1 ; _ 3 kg ; W 5 kg ; ¥ 0,3 kg a. Ditanyakan : Diagram gaya bebas masing-masing benda Jawab :

b. Ditanyakan : Apakah A dan B bergerak bersama Jawab : Untuk mengetahui apakah benda A dan B

bergerak bersama, kita cari dahulu percepatan sistem ketika kondisi tersebut terpenuhi (A tidak slip terhadap B)

Logika soal : Untuk mengerti persoalan ini kita tinjau benda B. Dari tinjauan gaya, benda B dapat

bergerak ke kanan disebabkan oleh gaya gesek yang mendorongnya ke kanan (lihat diagram gaya benda B). Dengan kata lain, jika antara B dan A tidak terdapat gesekan (licin), maka B tidak akan bergerak. Gaya gesek tersebut akan memberikan percepatan ;W sesuai dengan hukum Newton ∑ 2 W;W

Pada saat sistem bergerak bersama (A diam relatif terhadap B), besar gaya gesek yang

bekerja pada B adalah gaya gesek statik. Yang jadi poin penting adalah gaya gesek (sebagai penyebab bergeraknya benda B) yang bekerja pada benda B memiliki nilai maksimum yaitu 8 @2 :V yang memberikan percepatan maksimum pada benda B sebesar

∑ 2 W;W : _ W;W @2 ;W @2 f

[ 1,2 m/s2

Page 41: FISIKA TPB

Jika percepatan sistem total lebih besar dari ;W @2 maka benda A dan B tidak akan bergerak bersama (gaya gesek tidak mampu memberikan percepatan lebih besar dari ;W @2)

Percepatan sistem :

Tinjau benda A Tinjau benda B Tinjau benda C

2 ;

3 8 _;

2 ;

8 W;

' ;

¥ 3 ¥;

Substitusi menghasilkan 3 W; _; # 3 W; _; Maka didapatkan ¥ 3 ¥; ¥ W; _; ¥; ; ¤

¤[ 0,36 m/s2

Karena percepatan ini lebih kecil dibanding percepatan maksimum maka benda A dan B

bergerak bersama-sama c. Ditanyakan : Percepatan masing-masing Jawab : Karena sistem bergerak bersama-sama maka ;_ ;W ;¥ 0,36 m/s2 5. Diketahui : E 4M H (Fisika IA) ; E 3 H (Fisika IB) Fisika Dasar IA

a. Ditanyakan : Kerja untuk berpindah dari O ke B Jawab : 4MH

Usaha lintasan 1 d L E · B4E ; B4E B4 cos 7 H B4 sin 7 F d L 4MHW

B4 cos 7 H B4 sin 7 F L 4MB4 cos 7W

Karena M 4 cos 7 # BM B4 cos 7 dapat dituliskan

d L 4M BM 8 J

Usaha lintasan 2

d L E · BME_ L E · BEW

_ L 4M BM 8 J

b. Ditanyakan : Apakah gaya konservatif

Jawab : Gaya tersebut konservatif karena usaha yang dilakukan tidak bergantung lintasan Fisika Dasar IB

a. Ditanyakan : Kerja untuk berpindah dari O ke B Jawab : 3H

Usaha lintasan 1 d L E · B4E ; B4E B4 cos 7 H B4 sin 7 F d L 3HW

B4 cos 7 H B4 sin 7 F L 3B4 cos 7W

Karena M 4 cos 7 # BM B4 cos 7 dapat dituliskan

d L 3 BM 6 J

Usaha lintasan 2

d L E · BME_ L E · BEW

_ L 3 BM 6 J

Page 42: FISIKA TPB

b. Ditanyakan : Apakah gaya konservatif Jawab : Gaya tersebut konservatif karena usaha yang dilakukan tidak bergantung lintasan

Page 43: FISIKA TPB

SOAL UJIAN TENGAH SEMESTER I

FISIKA DASAR I A

TAHUN 2008/2009

1. Sebuah kendaraan berjalan sepanjang sumbu –x

dengan kecepatan seperti terlihat pada gambar. Pada t = 0 kendaraan tersebut ada pada posisi M= 10 m. a) Gambarkan kurva percepatan kendaraan tersebut

terhadap waktu untuk selang t = 0 sampai dengan t = 30s.

b) Hitung jarak yang ditempuh kendaraan itu dari t = 0 hingga t = 30s.

c) Tentukan posisi kendaraan pada saat t = 30s. d) Gambarkan sketsa posisi terhadap waktu untuk selang t = 0 sampai dengan t = 30s.

2. Suatu sistem dua benda dan katrol terlihat seperti gambar disamping. Bidang miring kasar(tan7 = 3 4Â dan :9 = 0,4), katrol dan tali tidak memiliki massa. Tidak ada gesekan antara tali dan katrol,

dan katrol dapat bebas bergerak.

Gambarkan diagram benda bebas untuk , dan katrol , jika cenderung turun.

a) Tentukan massa minimum benda (dalam ) supaya benda tepat akan bergerak ke atas. b) Apabila bidang miring licin dan benda = 0,2 ke arah mana gerak benda dan berapa

percepatan benda . 3. Sebuah benda mendapat sejumlah gaya dengan

resultan gayaEe = (6H + 15F - 7T)N. Awalnya benda berada

di posisi CE = (H + F + T)m dengan energi kinetik awal K =

50J. Setelah 4 detik benda di posisi cDE = (10H +25F - T)m, tentukan:

a) Usaha yang dilakukan oleh gaya resultan tersebut. b) Energi kinetik benda pada posisiCE.

c) Daya yang diberikan pada benda selama selang waktu 4s. 4. Benda 1 ( = 1 kg) berada pada ketinggian 5 meter di atas bukit(titik A). Kemudian diluncurkan

pada lintasan licin dan menumbuk secara lenting sempurna benda 2 ( = 3 kg). Setelah tumbukan, benda 2 bergerak melintasi bidang kasar yang panjang 3,5 m (: = 0,3). Tentukanlah a) Kecepatan benda 1 dan benda 2 setelah tumbukan. b) Kecepatan benda 2 di titik C. c) Tinggi kalau benda 2 jatuh sejauh 2 m dari sisi tebing (titik D).

Page 44: FISIKA TPB

- 44 -

5. Sebuah silinder pejal dengan massa M dan jari-jari penampang R semula diam di tarik pada sumbu

putar (1 = 1 2Â c) yang melewati tengah siinder dengan gaya F yang membentuk sudut 7

dengan bidang horizontal (lihat gambar). Apabila silinder menggelinding murni dan permukaan silinder dan lantai memiliki gaya gesek, tentukanlah: a) Percepatan translasi pusat massa dan gaya gesek(dalam F, M dan 7). b) Energi kinetik total setelah silinder menempuh jarak d meter.

Page 45: FISIKA TPB

SOLUSI UJIAN TENGAH SEMESTER I

FISIKA DASAR I A

TAHUN 2008/2009

1. a) Kurva percepatan terhadap waktu

b) Jarak yang ditempuh kendaraan adalah luas total kurva, yaitu 200 m. c) Posisi kendaraan adalah luas kurva di atas sumbu-M dikurangi luas kurva di bawah sumbu- ditambah posisi awal, sehingga posisi adalah 150 50 10 110. d) Kurva posisi terhadap waktu

2. a)

b) Dari diagram benda bebas didapat persamaan 3 sin 7 V: ; V cos 7

-2.5

-2

-1.5

-1

-0.5

0

0.5

1

1.5

2

2.5

0 5 10 15 20 25 30 35

0

20

40

60

80

100

120

140

160

180

0 5 10 15 20 25 30 35

Page 46: FISIKA TPB

3 23 3 ; ; 2; Kondisi supaya benda tepat akan bergerak ke atas adalah 3 sin 7 V: 0 3 sin 7 cos 7 : 3 23 3 2 sin 7 cos 7 : Akhirnya akan didapat massa minimum 0.46 c) Karena kurang dari massa minimum maka akan bergerak ke bawah Persamaan geraknya adalah sin 7 3 ; 3 23 3 ; ; 2; Dengan memasukkan ; 2; ke persamaan 3 ; akan didapat 3 @

, kemudian dengan memasukkan 3 23 ke persamaan sin 7 3 ; akan

didapat sin 7 2 ; ;, dan hasilnya akan didapat 10 u 0.6 2 u 0.2 u10 0.2 ;, sehingga didapat ; 1.67 4Â

3. a) d L E. B4E atau d E. ∆CE Dari informasi yang didapat dari soal didapat bahwa

d eDDDDE. CDDDE CDDDE d S6H 15F 7TU. S9H 24F 2TU d 428 Ä

b) d ∆i, misalkan energi kinetik benda pada posisi CDDDE adalah i, maka d i i, didapat i d i, i 478 Ä c) j d  , j 107 d

4. a) Kecepatan benda 1 ketika bertumbukan dengan benda 2 adalah v2` 10 4Â ,

dengan menggunakan hukum kekekalan momentum akan didapatkan ′ ′ Karena tumbukan bersifat lenting sempurna maka ′ ′ Kecepatan benda 2 sesaat sebelum bertumbukan adalah , maka persamaan di atas menjadi

′ ′ ′ ′ Dengan memasukkan 1 , dan 3 didapat ′ 3′ ′ ′ ′ 3 3′ 2 4′ Didapat ′

, ′ 5 4⁄ , tanda minus menunjukkan bahwa benda 1 bergerak

berlawanan arah setelah tumbukan. Dan dengan memasukkan ′ akan didapatkan ′ 5 4⁄ b) Kecepatan benda 2 di titik C dapat dihitung dengan persamaan 2:B, dimana adalah kecepatan benda 2 setelah tumbukan yaitu 5 4⁄ . Jadi 25 2 u 0.3 u 10 u 3.5, didapat 2 4⁄

c) Waktu yang diperlukan oleh benda 2 untuk mencapai lantai adalah 9I

1 4, lAY ,

didapat 5 5. a) Persamaan gerak rotasi adalah cos 7 c ¶Æ

Page 47: FISIKA TPB

cos 7 c ¶ ;c

Dimana ¶ adalah momen inersia dengan sumbu pada dasar bola, menurut teorema sumbu sejajar didapat ¶ 3 2Â c

cos 7 c 3 2Â c;

Didapat ; ²³¨ h/Ç

Persamaan gerak translasi adalah : ;, : ²³¨ h/Ç , ²³¨ h

/ :,

didapat : Ç ²³¨ h

b) Waktu yang diperlukan silinder untuk mencapai jarak B didapat dari persamaan B ;,

l 0&Ç ²³¨ h, kecepatan translasi ketika mencapai B adalah ; ²³¨ h

/Ç l 0&Ç ²³¨ h,

l ²³¨ h/Ç √2B, sedangkan kecepatan rotasi adalah Æ @

e l 0&Ç ²³¨ h, l ²³¨ h

/Ç√&

e , energi

kinetik total adalah energi kinetik translasi ditambah energi kinetik rotasi. \

¶ ²³¨ h

/Ç 2B u /

c ²³¨ h/Ç

&eY & ²³¨ h

/ B cos 7 & ²³¨ h/

Page 48: FISIKA TPB

UJIAN TENGAH SEMESTER II

FISIKA DASAR I

Page 49: FISIKA TPB

SOAL UJIAN TENGAH SEMESTER II

FISIKA DASAR I

TAHUN 2002/2003

1. Dalam suatu rumah bertingkat, air dipompakan dari lantai dasar dengan menggunakan pipa berdiameter 2,5 cm dengan laju 40 cm/s ke sebuah bak penampungan air di lantai tiga pada ketinggian 10 meter dari pompa, air tersebut mengalir pada sebuah keran dengan diameter 1 cm dan tekanan air pada ketinggian tersebut adalah 1,3 u 10 N/m2.

a. Berapa laju gerak air sesaat keluar dari keran di lantai tiga rumah itu b. Hitunglah tekanan air di lantai dasar 2. Dua orang anak A dan B ( _ 35kg ; W 30kg) berdiri

di atas ujung-ujung sebuah perahu ( ?kZ@A 60kg ; panjang perahu È 2 m) yang sedang diam (seperti terlihat pada gambar), massa perahu tersebut terdistribusi secara merata. Terdapat juga sebuah benda ( kb&@ 5kg) yang dipegang oleh A. (catatan : perahu dapat bergerak bebas di dalam air, gesekan air dapat diabaikan)

a. Jika anak-anak tersebut dengan laju yang sama bertukar tempat, ke arah mana perahu bergerak berapa jauhnya?

b. Jika sebelum bertukar tempat anak A melempar benda yang ia pegang secara horizontal ke luar perahu (arah H) dengan laju 10 m/s, tentukan kecepatan perahu sesaat setelah lemparan tersebut

3. Sebuah kubus pejal dengan panjang sisi 0,75 cm. Kubus tersebut mengapung di atas minyak (rapat

massa minyak 800 kg/m3) dengan sepertiga volum kubus tersebut berada di atas permukaan minyak a. Berapa gaya apung pada kubus tersebut b. Berapa rapat massa bahan kubus tersebut

4. Sebuah katrol dengan dua alur berbeda seperti pada gambar ( 5 kg ; É 8

kg m2) dihubungkan dengan benda A ( _ 20 kg) dan benda B dengan tali yang massanya dapat diabaikan seperti pada gambar. Jika C_ 0,15 meter dan CW 0,10 meter, dan katrol bergerak tanpa gesekan

a. Hitung massa benda B agar sistem dalam keadaan setimbang b. Jika diketahui W 40 kg, tentukan : i. Besar dan arah percepatan sudut ii. Tegangan tali pada masing-masing benda 5. Satu mol gas ideal monoatomik (Ê 5/3) mula-mula mempunyai volume 8,31 u 10t/ m3 dan

tekanan 3 u 10 N/m2. Gas tersebut mengalami proses pemanasan pada tekanan tetap hingga volumnya menjadi dua kali lipat volume sebelumnya, kemudian dilakukan pendinginan ke suhu awal pada volume tetap. Kemudian gas tersebut mengalami pemampatan isotermis sehingga kembali ke keadaan semula

a. Gambarkan diagram j Ë untuk proses yang dialami gas ideal tersebut b. Tentukan Ì, d, ∆ untuk tiap proses yang dialami gas tersebut c. Tentukan efisiensi suatu mesin yang prosesnya mengikuti siklus gas tersebut. Jika diketahui

efisiensi merupakan perbandingan kerja total yang dilakukan oleh gas dengan kalor yang masuk.

Page 50: FISIKA TPB

SOLUSI UJIAN TENGAH SEMESTER II

FISIKA DASAR I

TAHUN 2002/2003 1. Diketahui : B 2,5cm ; B 1cm ; 0,4 m/s ` 0 ; ` 10 m ; j 1,3 · 10 V/m

a. Ditanyakan : Laju gerak air saat keluar keran Jawab : Dengan menggunakan persamaan kontinuitas dapat diperoleh

; m +&-

Maka,

+__Y- +&

&Y- 2,5 m/s

b. Ditanyakan : Tekanan air di dasar

Jawab : Dari persamaan Bernoulli didapatkan

j Í` Í j Í`

Í

j j Í` ` Í 2,33 · 10 N/m2

2. Diketahui : _ 35 kg ; W 30 kg ; kb&@ 5 kg ; ? 60 kg ; È 2 m

a. Ditanyakan : Arah gerak perahu Jawab : Karena tidak ada gaya luar yang bekerja, maka pusat massa sistem tidak akan berubah. Dengan memilih anak A sebagai acuan, kita dapat menggambarkan posisi anak A, anak B, dan titik pusat massa dalam sebuah sistem koordinat Posisi pusat massa relatif terhadap titik A adalah

Î?,_ 2¾S2JÏ,¾U[2[[¾

/ m

Apabila terjadi pertukaran tempat, misalkan perahunya bergerak sejauh ; dari A, maka

Î?,_ @¾@[@[¾ /@

/ / m

sehingga didapatkan ; / H m

b. Ditanyakan : Kecepatan perahu sesaat setelah lemparan

Jawab : Karena tidak ada gaya luar yang bekerja pada perahu, maka momentum perahu akan kekal, sehingga jDE@@^ jDE@AaZ Akan tetapi pada awalnya perahu dalam keadaan diam, sehingga momentum awal perahu sama dengan nol 0 S ? _ WU?

sehingga didapatkan ? H 0,4H m/s

3. Diketahui : C 0,75 cm # Ë C/ 0,421875 cm3 Rapat massa fluida, Ð 800 kg/m3

Page 51: FISIKA TPB

a. Ditanyakan : Gaya apung kubus Jawab : Gaya apung dapat ditentukan dengan persamaan

_ ÍËg dengan Ë adalah volume benda yang tercelup dalam fluida, Ë / Ë

Sehingga didapatkan _ ÍËg Í +/ Ë- g 2,25 · 10/ V

b. Ditanyakan : Rapat massa kubus

Jawab : Karena benda mengapung (sistem setimbang), maka _ Ñ ÍËg sehingga didapatkan

ÍË ÍË # Í ÒÓÔÕÓÔÕ 0,533 gr/cm3

4. Diketahui : 5 kg ; É 8 kg m2 ; _ 20 kg ; C_ 0,15 m ; C 0,10 m a. Ditanyakan : Massa benda B agar sistem dalam keadaan setimbang Jawab : Perhatikan diagram benda bebas untuk masing-masing benda dan katrol

Jika 3_ dan 3W disubstitusikan maka didapatkan 3_C_ 3CW # d_C_ dWCW

dW ÖZZ[ ; W Z

Z[ 30 kg

b. Ditanyakan : Percepatan dan tegangan tali jika diketahui W 40 kg Jawab : i. Besar dan arah percepatan sudut katrol Karena W 40 kg 30 kg, maka katrol akan bergerak berlawanan arah jarum jam

dengan percepatan sudut , atau benda A akan bergerak ke atas dengan percepatan linear ;_ C_, benda B bergerak ke bawah dengan percepatan linear ;W CW Sehingga didapatkan persamaan menurut hukum Newton 3_ d_ _;_ _ C_ dW 3W W;W W CW 3_C_ 3WCW ¶ Kalikan kedua ruas persamaan pertama dengan C_ dan persamaan kedua dengan CW lalu

jumlahkan keduanya dengan persamaan ketiga, maka didapatkan dWCW d_C_ ¶ W CW _ C_

[Z[tZ¹gZY[Z[Y 1,13 rad/s

ii. Tegangan tali masing-masing benda ;_ C_ 0,151,13 0,17 m/s2 ;W CW 0,11,13 0,113 m/s2 Sehingga didapatkan 3_ _g _;_ 203,4 N

Page 52: FISIKA TPB

3_ Wg W;W 395,48 N 5. Diketahui : 6 1 mol gas ideal monoatomik 3_ j_Ë_/6c 300i Ë_ 8,31 · 10t/ 3W jWËW/6c 600i j_ 3 · 10 N/m2 3¥ 3_ 300i jW 3 · 10 N/m2 j¥ 6c3¥/Ë¥ 1,5 · 10

ËW 2Ë_ 1,662 · 10t Ê /

Ë¥ ËW 1,662 · 10t

a. Ditanyakan : Diagram j Ë untuk proses yang dialami gas Jawab :

b. Ditanyakan : Ì, d, ∆ untuk tiap proses Jawab : Untuk proses AB d L jBË j_ËW Ë_ 2493 J

∆ I∆3 / 6c∆3 3739,5 J

Ì ∆ d 6232 J (Qin) Untuk proses BC d L jBË 0

∆ I∆3 / 6c∆3 3739,5 J

Ì ∆ d 3739,5 J Untuk proses CA

d L jBË L beÔ BË be

Ô ln +ÔÔ¤- 1728 J

∆ I∆3 0 Ì ∆ d 1728 J

c. Ditanyakan : Efisiensi mesin yang prosesnya mengikuti siklus gas Jawab : Efisiensi adalah perbandingan antara kerja yang dilakukan oleh gas dengan kalor yang masuk, sehingga efisiensi

× |Ù|t|Ù¿ÚN||Ù| u 100% Ö[Ö[¤Ö¤

Ù u 100% 12,3%

Page 53: FISIKA TPB

SOAL UJIAN TENGAH SEMESTER II

FISIKA DASAR IA

TAHUN 2003/2004 1. Bola A bermassa 2 kg diam di atas bidang datar (M ) yang licin kemudian dipukul dengan gaya mendatar ke kanan (sumbu-x positif) seperti pada gambar

Setelah dipukul, bola A menumbuk bola B bermassa 1 kg yang diam di depannya. Jika setelah

tumbukan bola A mempunyai kecepatan 1 m/s ke arah sumbu-y positif. Tentukan : a. Vektor kecepatan bola A setelah dipukul, dan b. Vektor kecepatan bola B setelah ditumbuk bola A

2. Sebuah katrol bermassa 2,5 kg, jari-jari c 20 cm (momen inersia ¶ 0,5c digantungkan vertikal seperti terlihat pada gambar. Sebuah balok

bermassa 0,5 kg digantungkan dengan tali ideal (massa dapat diabaikan dan tidak mulur) yang dililitkan pada katrol tersebut. Setelah balok dilepaskan dari keadaan diam, tentukan :

a. Percepatan jatuhnya balok b. Tegangan pada tali c. Percepatan sudut katrol 3. Air pompa seperti terlihat pada gambar di bawah ini

Luas penampang 8g cm2 ; 0,5 ; dan / . Tekanan udara luar sebesar 105 Pa. Jika tekanan di U adalah 1,5 u 10 Pa dan air mengalir di U dengan laju 5 m/s, tentukan :

a. Debit aliran air di T b. Tekanan air di S c. Naiknya air di pipa vertikal ()

4. Suatu sistem termodinamik, yang berupa gas ideal monoatomik dengan keadaan awal (j, Ë, 3 2 u 10 Pa ; 4 u 10t/ m3 ; 300 K mengalami ekspansi dengan tekanan tetap hingga volumenya

menjadi dua kali lipat. Selanjutnya ia dimampatkan pada suhu konstan hingga mencapai volume semula dan akhirnya didinginkan pada volume konstan hingga mencapai tekanan semula

a. Gambarkan proses tersebut dalam diagram j Ë b. Tentukan suhu pada proses isotermalnya c. Tentukan kerja dan kalor selama satu siklus d. Apakah sistem melakukan kerja dan apakah sistem menerima kalor

5. Sebuah batang kaku yang panjangnya È 1 m dan massanya 0,3 kg. Ujung atasnya (titik O)

digantung pada sebuah engsel tanpa gesekan (lihat gambar). Pada ujung bawah batang diletakkan sebuah bola pejal dengan massa 1,5 kg dan jari-jari c 10 cm (¶]^@ terhadap sumbu

melalui pusat bola = c)

Page 54: FISIKA TPB

a. Tentukan momen inersia sistem batang-bola terhadap titik O

b. Tentukan jarak titik pusat massa sistem batang-bola terhadap titik O

c. Jika sebuah peluru dengan massa 0,01 kg ditembakkan horizontal tepat ke arah titik pusat massa bola dengan kecepatan tumbukan peluru tertancap di pusat bola, tentukan kecepatan sudut sistem bola-batang-peluru tepat setelah peluru menumbuk bola

Page 55: FISIKA TPB

SOLUSI UJIAN TENGAH SEMESTER II

FISIKA DASAR IA

TAHUN 2003/2004 1. Diketahui : _ 2 kg ; W 1 kg ; E_ 1 F m/s a. Ditanyakan : Vektor kecepatan bola A setelah dipukul Jawab : Impuls yang diberikan kepada bola A adalah ¶ L B luas daerah di bawah kurva terhadap 0,50,1100 5 Ns Perubahan momentum bola A adalah ¶ ∆Ð __ 5 _ 2,5 H m/s b. Ditanyakan : Vektor kecepatan bola B setelah ditumbuk bola A Jawab : Dengan menggunakan hukum kekekalan momentum akan diperoleh

_E_ _DDDE_ WDDDEW

DDDEW 5H 2F m/s 2. Diketahui : 2kg ; c 0,2 m ; ¶ 0,5c ; 0,5 kg a. Ditanyakan : Percepatan jatuhnya balok Jawab : Perhatikan gambar berikut

Hukum Newton untuk kasus ini adalah ∑ ; # 3 ; ∑ Þ ¶ # 3c ¶ Dengan mengeliminasi 3 dari kedua persamaan tersebut akan diperoleh

¹e ;

,ÇeYe

@e ;

; Ç 2,86 m/s2

b. Ditanyakan : Tegangan tali Jawab : ∑ ; 3 ; # 3 ; 0,510 2,86 3,57 N c. Ditanyakan : Percepatan sudut katrol Jawab : ∑ Þ ¶

3c ¶ # e¹ /,ß,

,,,Y 14,28 rad/s2

3. Diketahui : 8 cm2 ; / 0,5 ; j 10 Pa ; / 4 m j 1,5 · 10 Pa ; 5 m/s ; j/ 10 Pa

a. Ditanyakan : Debit air di T Jawab : Ì // 4 · 10t/ kg/m3

Page 56: FISIKA TPB

b. Ditanyakan : Tekanan di S

Jawab : Gunakan persamaan kontinuitas untuk memperoleh besar kecepatan di titik 2

# __Y 10 m/s

Dengan menggunakan persamaan Bernoulli didapatkan

j Í` Í j9 Í`9

Í

j9 j Í` `9 Í

c. Ditanyakan : Naiknya air pada pipa vertikal

Jawab : Air dalam pipa vertikal diam sehingga hukum yang berlaku adalah fluida statik. Ketinggian pipa vertikal memenuhi hubungan j j Í`

Maka didapatkan ` ¼t¼OÒ 5 m

4. Diketahui : (j_, Ë_, 3_) = (2 · 10 Pa ; 4 · 10t/ m3 ; 300 K)

a. Ditanyakan : Diagram j Ë Jawab :

b. Ditanyakan : Suhu pada proses isothermal

Jawab :

Gunakan hubungan Ô Ô[

[

Sehingga didapatkan suhu pada proses isothermal adalah 3W 23_ 600 K

c. Ditanyakan : Kerja dan kalor selama satu siklus Jawab : Tinjau proses isobaric (tekanan konstan)

d L jBËÔ[Ô j_ËW Ë_ , ËW 2Ë_

d j_Ë_ 800 J Tinjau proses isothermal (suhu konstan)

d L jBËÔ¤Ô[ L be[Ô BËÔ¤Ô[ , Ë_ Ë¥

d 6c3W ln ÔÔ[ 6c3W ln

1109 J

Tinjau proses isokhorik (volume konstan) d L jBË Karena dalam proses ini tidak ada perubahan volume maka kerja proses sama dengan nol (d 0) Kalor untuk satu siklus sama dengan jumlah kerja dalam satu siklus Ì da9]@Za da9](kZ@^ da9]A]Za 309 J

d. Ditanyakan : Apakah sistem melakukan kerja dan apakah sistem menerima kalor Jawab : Berdasarkan konvensi, harga negatif yang terdapat pada kalor siklus mengindikasikan bahwa sistem menerima usaha dari luar dan memenghasilkan kalor

5. Diketahui : È 1 m ; 0,3 kg ; 1,5 kg ; c 10 cm a. Ditanyakan : Momen inersia sistem batang-bola Jawab : Momen inersia sistem = momen inersia batang terhadap titik O + momen inersia bola

terhadap titik O

Momen inersia batang terhadap titik O = / È

Page 57: FISIKA TPB

Momen inersia bola terhadap titik O dapat dicari dengan menggunakan teorema sumbu sejajar, ¶ ¶? ∆M, dengan ∆M merupakan besar perpindahan sumbu putar. Dalam kasus ini sumbu putar berpindah dari pusat massa ke titik O sejauh ∆M È c sehingga didapatkan

¶]^@ c È c

Jadi ¶9a9(k / È

c È c 1,921 kg m2

b. Ditanyakan : Jarak titik pusat massa sistem batang-bola terhadap titik O Jawab : Ambil titik O sebagai pusat dari koordinat kartesian dengan batang dan bola berada pada

sumbu-y. Dari gambar pada soal dapat kita ketahui titik pusat massa batang berada pada M à.

Sedangkan titik pusat massa dari bola sendiri berada di M È c (terhadap titik O). Maka titik pusat sistem batang-bola terhadap titik O adalah

M? Ç2ÇY2YÇÇY 1 m

c. Ditanyakan : Kecepatan sudut sistem sesaat setelah tumbukan Jawab : Untuk meninjau tumbukan yang terjadi kita gunakan hukum kekekalan momentum.

Akan tetapi karena sistem yang kita tinjau sistem benda tegar (bukan titik) maka yang digunakan adalah hukum kekekalan momentum sudut. Jenis tumbukan yang terjadi tidak elastik sama sekali

È È # ??È c ¶9a9(k ??È c Peluru yang bersarang di bola menyebabkan terpenuhinya hubungan È c ? sehingga

didapatkan ??È c ¶9a9(k ?È c

JIJàe¹§§NáÏJàeY , ¶9a9(k = momen inersia sistem batang bola

Page 58: FISIKA TPB

1. a. Dapatkah sistem yang terdiri dari dua benda atau lebih mempunyai energi kinetik total tidak

sama dan memiliki momentum sama dengan nol? Jelaskan b. Bagaimana mungkin gaya yang besar dapat 2. Bola tenis bermassa 400

akibatnya bola memantul dengan arah yang berlawanan arah semula dengan laju 20 m/s a. Tentukan besar dan arah impuls yang dial b. Bila waktu kontak antara bola dengan dinding adalah 0,01 detik, hitunglah besarnya gaya rata

rata yang dialami bola selama kontaknya 3. Sebuah tabung berbentuk huruf L terisi penuh cairan dengan titik A dan B ada pada dasar tabung

(lihat gambar). Gunakan tekanan udara luar a. Besar tekanan di A dan B, dan b. Gaya hidrostatik yang dialami sebuah potongan kaca yang luasnya

4. Sebuah kawat terbuat dari besi berdiameter 2 cm (modulus Young besi =

bersambung secara vertikal dengan kawat baja berdiameter 1 cm (modulus Young baja = 10 N/m2). Panjang masingmasing kawat jika sistem tersebut dibebani dengan massa 400 kg?

5. Seekor jerapah memerlukan jantung yang kuat karena lehernya panjang. Diketahui perbedaan

antara katup nadi (tempat dimana pembuluh darah nadi keluar dari jantung) dan kepala adalah 2,5m serta pembuluh nadi dekat katup nadi sampai ke kepala memiliki penampang lintang yang sama dengan 3,0 mm. Darah merupakan fluida tak kompresibel dengan rapat massa 1,0 g/cm

a. Hitunglah tekanan minimum ke kepala dengan tekanan nol)

b. Jika kecepatan aliran darah dalam soal (a) adalah 7,0 cm/detik, tentukan debit darah yang mengalir

6. Bola pejal A (¶_ c

c) memiliki massa dan jari

kg dan c 0,05 m. Keduanya digelindingkan tanpa selip dari ketinggian miring (tan 0,75)

SOAL UJIAN TENGAH SEMESTER II

FISIKA DASAR I

TAHUN 2004/2005

Dapatkah sistem yang terdiri dari dua benda atau lebih mempunyai energi kinetik total tidak sama dan memiliki momentum sama dengan nol? Jelaskan Bagaimana mungkin gaya yang besar dapat menghasilkan momen gaya nol? Jelaskan

400 gr menumbuk dinding dalam arah horizontal dengan laju 30 m/s, akibatnya bola memantul dengan arah yang berlawanan arah semula dengan laju 20 m/s

Tentukan besar dan arah impuls yang dialami bola Bila waktu kontak antara bola dengan dinding adalah 0,01 detik, hitunglah besarnya gaya ratarata yang dialami bola selama kontaknya

Sebuah tabung berbentuk huruf L terisi penuh cairan dengan titik A dan B ada pada dasar tabung ar). Gunakan tekanan udara luar Í N/m2 dan rapat massa cairan

Besar tekanan di A dan B, dan Gaya hidrostatik yang dialami sebuah potongan kaca yang luasnya m2

Sebuah kawat terbuat dari besi berdiameter 2 cm (modulus Young besi = bersambung secara vertikal dengan kawat baja berdiameter 1 cm (modulus Young baja =

). Panjang masing-masing kawat adalah 6 m. Berapakah pertambahan panjang masmasing kawat jika sistem tersebut dibebani dengan massa 400 kg?

Seekor jerapah memerlukan jantung yang kuat karena lehernya panjang. Diketahui perbedaan antara katup nadi (tempat dimana pembuluh darah nadi keluar dari jantung) dan kepala adalah 2,5m serta pembuluh nadi dekat katup nadi sampai ke kepala memiliki penampang lintang yang sama dengan 3,0 mm. Darah merupakan fluida tak kompresibel dengan rapat massa 1,0 g/cm

Hitunglah tekanan minimum pada katup nadi jerapah (tekanan yang menyebabkake kepala dengan tekanan nol) Jika kecepatan aliran darah dalam soal (a) adalah 7,0 cm/detik, tentukan debit darah yang

) dan silinder pejal B (¶W ) memiliki massa dan jari-jari sama yaitu 0,5

m. Keduanya digelindingkan tanpa 2 meter pada sebuah bidang

Dapatkah sistem yang terdiri dari dua benda atau lebih mempunyai energi kinetik total tidak

menghasilkan momen gaya nol? Jelaskan

gr menumbuk dinding dalam arah horizontal dengan laju 30 m/s, akibatnya bola memantul dengan arah yang berlawanan arah semula dengan laju 20 m/s

Bila waktu kontak antara bola dengan dinding adalah 0,01 detik, hitunglah besarnya gaya rata-

Sebuah tabung berbentuk huruf L terisi penuh cairan dengan titik A dan B ada pada dasar tabung dan rapat massa cairan Í kg/m3, lalu tentukan

2 di B

Sebuah kawat terbuat dari besi berdiameter 2 cm (modulus Young besi = 100 u 10« N/m2) bersambung secara vertikal dengan kawat baja berdiameter 1 cm (modulus Young baja = 22 u

masing kawat adalah 6 m. Berapakah pertambahan panjang masing-

Seekor jerapah memerlukan jantung yang kuat karena lehernya panjang. Diketahui perbedaan antara katup nadi (tempat dimana pembuluh darah nadi keluar dari jantung) dan kepala adalah 2,5 m serta pembuluh nadi dekat katup nadi sampai ke kepala memiliki penampang lintang yang sama dengan 3,0 mm. Darah merupakan fluida tak kompresibel dengan rapat massa 1,0 g/cm3.

pada katup nadi jerapah (tekanan yang menyebabkan darah sampai

Jika kecepatan aliran darah dalam soal (a) adalah 7,0 cm/detik, tentukan debit darah yang

Page 59: FISIKA TPB

a. Gambarkan gaya-gaya yang bekerja pada bola dan silinder b. Hitunglah percepatan translasi masing c. Bila bidang miring licin, bagaimana keadaan gerak benda dan berapa beda waktu yang

diperlukan oleh A dan B untuk sampai ke dasar bidang miring? 7. Sejumlah 2 mol gas ideal monoatomik mengalami proses siklus A

gambar di bawah. Proses Diketahui j_ 150 kPa; Ë

a. Apakah proses C-A adiabatik atau isotermik? b. Untuk masing-masing proses hitung c. Hitung efisiensi siklus ini

gaya yang bekerja pada bola dan silinder Hitunglah percepatan translasi masing-masing benda

miring licin, bagaimana keadaan gerak benda dan berapa beda waktu yang diperlukan oleh A dan B untuk sampai ke dasar bidang miring?

Sejumlah 2 mol gas ideal monoatomik mengalami proses siklus A-B-C-A seperti ditunjukkan pada gambar di bawah. Proses C-A adalah satu di antara dua proses berikut : adiabatik atau isotermik. Ë_ 0,03 m3; ËW 0,045 m3; j¥ 100kPa.

A adiabatik atau isotermik? masing proses hitung Ì, d, dan ∆

efisiensi siklus ini

miring licin, bagaimana keadaan gerak benda dan berapa beda waktu yang

A seperti ditunjukkan pada A adalah satu di antara dua proses berikut : adiabatik atau isotermik.

Page 60: FISIKA TPB

SOLUSI UJIAN TENGAH SEMESTER II

1. Jawab : a. Dapat. Untuk N buah benda yang masing

energi kinetik total adalah

nol, karena momentum merupakan vektor sehingga tidak hanya ditentukan oleh besarnya saja tetapi juga arahnya.

b. Momen gaya ÞE CE u E

momen gaya dapat bernilai nol jika jarak dari titik kerja gaya ke sumbu putar (atau jika sudut yang dibentuk antara vektor

2. Diketahui : ]^@ 0,4

a. Ditanyakan : Besar dan araoleh bola

Jawab : ¶E SDDDE EU 0,4 Jadi besar impuls yang dialami oleh bola adalah 20 kg m/s dengan arah ke sumbu

b. Ditanyakan : Gaya rataJawab :

¶ Z@(@tZ@(@∆ # 3. Diketahui : Tabung berbentuk L terisi cairan a. Ditanyakan : Besar tekanan di titik A dan B Jawab : Karena titik A dan B berada pada

kedua titik tersebut adalah sama j_ jW j Í b. Ditanyakan : Gaya hidrostatik yang dialami potongan kaca Jawab : jW

4. Diketahui : B 2 cm ; B âk9a 100 u Ditanyakan : Pertambahan panjang Jawab : Pertambahan panjang masing

hukum Hooke _ â ∆à

à . Karena disusun seri maka tegangan masing

sama yaitu sebesar Pertambahan panjang kawat besi

&  Y âk9a ∆àÕá§à #

Pertambahan panjang kawat baja

SOLUSI UJIAN TENGAH SEMESTER II

FISIKA DASAR I

TAHUN 2004/2005

Dapat. Untuk N buah benda yang masing-masing massanya a dan kecepatan

energi kinetik total adalah \i ∑ aa µ 0. Momentum ÐE ∑ a

nol, karena momentum merupakan vektor sehingga tidak hanya ditentukan oleh besarnya saja

E, |Þ| |C||| sin 7; 7 sudut antara vektor CE dan vektor bernilai nol jika jarak dari titik kerja gaya ke sumbu putar (

atau jika sudut yang dibentuk antara vektor CE dan E (yaitu 7) sama dengan nol.

kg ; E 30H m/s ; DDDE 20H m/s

Besar dan arah impuls yang dialami

20H 30H 20H kg m/s Jadi besar impuls yang dialami oleh bola adalah 20

kg m/s dengan arah ke sumbu-x negatif

Gaya rata-rata yang dialami bola

Z@(@tZ@(@ ¹∆( 2000 N

Tabung berbentuk L terisi cairan

Besar tekanan di titik A dan B Karena titik A dan B berada pada kedalaman yang sama, maka tekanan hidrostatik di

kedua titik tersebut adalah sama Í`

Gaya hidrostatik yang dialami potongan kaca j Í`

B 1 cm ; È È 6 m u 10« N/m2 ; âk9a 22 u 10 N/m2

Pertambahan panjang masing-masing kawat

panjang masing-masing kawat dapat dihitung dengan menggunakan

. Karena disusun seri maka tegangan masing-masing kawat

, maka Pertambahan panjang kawat besi

# ∆Èk9a à&  YãÕá§

, m

Pertambahan panjang kawat baja

dan kecepatan a µ 0, maka

Ea, nilai Ð mungkin saja

nol, karena momentum merupakan vektor sehingga tidak hanya ditentukan oleh besarnya saja

dan vektor E, apabila E µ 0, bernilai nol jika jarak dari titik kerja gaya ke sumbu putar (C) bernilai nol,

) sama dengan nol.

kedalaman yang sama, maka tekanan hidrostatik di

masing kawat dapat dihitung dengan menggunakan

masing kawat

Page 61: FISIKA TPB

&Y  Y â@ä@ ∆àÕKåKàY

5. Diketahui : ` 2,5 m ; Í a. Ditanyakan : Tekanan minimum pada katup nadi jerapah Jawab :

Gunakan persamaan Bernoulli

Pilih `b 0 dan j

lintang yang sama maka kecepatang aliran di nadi dan kepala sama. persamaan di atas dapat disederhanakan sebagai berikut

jb j Í b. Ditanyakan : Debit aliran darah Jawab :

Debit Ì 7

6. Diketahui : ¶_ c ;

a. Ditanyakan : Gambar g Jawab :

b. Ditanyakan : Percepatan translasi masing Jawab : Bola ∑ ; sin 7 8 ∑ Þ ¶ 8c ¶ @JÏ

e

Dengan menghilangkan sin 7 ¶ sin 7

;?

ß sin Silinder ∑ ; sin 7 8 ∑ Þ ¶ 8c ¶ @JÏ

e

ÕKåK # ∆È@ä@ àY&Y  YãÕKåK

, m

Í 1,0 gr/cm3

Tekanan minimum pada katup nadi jerapah

Gunakan persamaan Bernoulli jb Í`b Íb j Í`

j 0 (tekanan darah di kepala nol). Karena memiliki luas penampang

lintang yang sama maka kecepatang aliran di nadi dan kepala sama. persamaan di atas dapat disederhanakan sebagai berikut Í` 2,5 · 10/ Pa

bit aliran darah

u 10tm +/·æ¡ - 0/

m · 10ts m3/s

; ¶W c ; 0,5 kg ; c 0,05 m ; 2

Gambar gaya-gaya yang bekerja bola dan silinder

Percepatan translasi masing-masing benda

8 ;?

Dengan menghilangkan 8 maka didapatkan @JÏeY ;?

c @JÏ

eY ;?

sin 7 /ß m/s2

8 ;?

Í

(tekanan darah di kepala nol). Karena memiliki luas penampang lintang yang sama maka kecepatang aliran di nadi dan kepala sama. b sehingga

m ; tan 3/4

Page 62: FISIKA TPB

Dengan menghilangkan 8 maka didapatkan sin 7 ¶ @JÏ

eY ;?

sin 7 c @JÏ

eY ;?

;? / sin 7 4 m/s2

c. Ditanyakan : Keadaan gerak benda dan beda waktu antara A dan B Jawab : Jika tidak ada gaya gesek maka torka akan bernilai nol sehingga kedua benda hanya

akan bergerak translasi (tidak menggelinding). Jika kedua benda pada saat 0 berada pada ketinggian yang sama dan memiliki kecepatan awal yang sama maka kedua benda tersebut akan tiba di dasar bidang miring pada waktu yang sama

7. Diketahui : mol gas ideal monoatomik, siklus A-B-C-A j_ 150 kPa ; Ë_ 0,03 m3 ; ËW 0,045 m3 ; j¥ 100 kPa a. Ditanyakan : Jenis proses C-A Jawab : Periksa suhu di titik A dan C. Jika sama maka proses AC isothermal, jika tidak sama

maka proses tersebut adiabatic

3_ ¼Ôbe 270,7 K

3¥ ¼¤Ô¤be 270,7 K

Karena 3_ 3¥ maka proses CA adalah isothermal

Sedangkan suhu di titik B ditentukan sebagai berikut 3W ¼[Ô[be 406,1 K

b. Ditanyakan : Nilai Ì, ∆, Ì masing-masing proses Jawab :

Proses AB (isobaric) d L jBË jËW Ë_ 2250 J

∆ 6I∆3 / 6c∆3 3375 J

Ì ∆ d 3367 2250 5625 J (Qin) Proses BC (isokhorik)

d ç jBË 0

∆ 6I∆3 / 6c∆3 3375 J

Ì ∆ d 3375 J Proses CA (isothermal)

d L jBË L beÔ BË 6c3 ln +Ô

Ô¤- 1824 J

∆ 6I∆3 0

Ì ∆ d 1824 J (Qout) c. Ditanyakan : Efisiensi siklus Jawab : Kalor yang masuk ke sistem : Ìab Ì_W 5625 J Kalor yang keluar dari sistem : Ì]( |ÌW¥| |Ì¥_| 5199 J Netto kalor Ì Ìab Ì]( 5625 5199 426 J Dengan demikian efisiensi dari mesin kalor tersebut adalah

× ÖÙ 0

0 u 100% 7,6%

Page 63: FISIKA TPB

SOAL UJIAN TENGAH SEMESTER II

FISIKA DASAR I

TAHUN 2005/2006 1. Sebuah bola padat yang mula-mula dalam keadaan diam, pecah menjadi tiga bagian. Pecahan

pertama bergerak dengan momentum 1,2H kg m/s. Pecahan kedua memiliki momentum 0,5F kg m/s. a. Kemanakah arah pecahan ketiga akan bergerak? (nyatakan dalam sudut relatif terhadap sumbu-y

positif) b. Jika massa pecahan ketiga adalah 0,5 kg, berapakah energi kinetik pecahan ketiga tersebut?

2. Dua buah benda masing-masing bermassa 4 dan 1 dihubungkan dengan tali tak

bermassa dan katrol yang memiliki momen inersia 0,05 kg m2. Benda berada pada bidang miring dengan sudut kemiringan , koefisien gesek statik dan kinetik antara benda bidang miring adalah :9 0,2 ; : 0,1 jika sin 0,6. Tentukanlah

a. Arah gerak benda b. Percepatan benda , dan c. Tegangan tali 3 dan 3

3. Di dalam ruang kuliah yang sangat besar, bandul dibuat dengan menggantungkan

bola bermassa 5 kg di ujung kawat baja yang memiliki panjang 2 meter. Kawat baja tersebut disambungkan dengan kawat tembaga yang panjangnya 1 meter. Kedua jenis kawat tersebut memiliki luas penampang yang sama, yaitu 1 mm2. Modulus Young baja adalah 20 u 10 N/m2, sedangkan modulus Young tembaga adalah 10 u 10 N/m2

a. Hitunglah regangan masing-masing kawat b. Hitunglah tegangan pada masing-masing kawat 4. Partikel π-meson memiliki waktu hidup rata-rata sebesar 2,6 u 10ts detik (diukur

dalam kerangka diam terhadap meson di laboratorium) a. Jika partikel tersebut bergerak terhadap laboratorium dengan kecepatan 0,8c (< 3 u 10s m/s),

berapa waktu hidupnya diukur oleh pengamat di laboratorium? b. Bila diukur di laboratorium, hitunglah jarak yang ditempuh selama waktu hidupnya

5. Sebuah tangki air yang tertutup berisi air hingga

setringgi 1 meter dan di atas permukaannya diberi tekanan absolute 1,2 atm. Di sisi samping di dekat dasar tangki dilubangi sehingga air bisa mengalir keluar (lihat gambar). Luas penampang tangki jauh lebih besar dibandingkan lubang itu. Rapat massa air adalah 1 gr/cm3. Tekanan udara luar 1 atm

a. Pada saat kran air B masih tertutup sehingga air tak mengalir berapa atm-kah tekanan di titik A di dasar

b. Jika kran B dibuka berapakah kecepatan air keluar di B c. Pada keadaan soal (b) tetapi kran D dibuka, berapakah tinggi kolom air di atas titik A, bilamana

luas di lubang B = 2 cm2 dan luas pipa A = 4 cm2

Page 64: FISIKA TPB

6. Sebuah model mesin Stirling ideal menggunakan 0,01 mol gas ideal sebagai bahan penggeraknya. Mesin bekerja pada reservoir bertemperatur tinggi lipat saat ekspansi

a. Tentukan kerja yang dilakukan mesin dalam satu b. Berapa jumlah kalor yang masuk ke dalam sistem dalam satu siklus c. Hitung efisiensi mesin

Sebuah model mesin Stirling ideal menggunakan 0,01 mol gas ideal sebagai bahan penggeraknya. Mesin bekerja pada reservoir bertemperatur tinggi 3 300 K. Volume kerja menjadi dua kali

Tentukan kerja yang dilakukan mesin dalam satu siklus Berapa jumlah kalor yang masuk ke dalam sistem dalam satu siklus

Sebuah model mesin Stirling ideal menggunakan 0,01 mol gas ideal sebagai bahan penggeraknya. K. Volume kerja menjadi dua kali

Page 65: FISIKA TPB

SOLUSI UJIAN TENGAH SEMESTER II

1. Diketahui : ÐE a. Ditanyakan : Arah Jawab : Dengan menggunakan hukum kekekalan momentum kita akan memperoleh

b. Ditanyakan : Energi kinetik pecahan ketiga Jawab :

\i/ /

2. Diketahui : 4 ; :9 0,2 ; :

a. Ditanyakan : Arah gerak bendaJawab : Perhatikan gambar berikut

Dari gambar di atas dapat dilihat bahwa untuk menentukan arah gerak dari benda, cukup dengan membandingkan besarnya gaya yang bekerja pada besarnya gaya yang bekerja pada Sekarang akan kita periksa besarnya gaya pada masing sin

SOLUSI UJIAN TENGAH SEMESTER II

FISIKA DASAR I

TAHUN 2005/2006

0 ; ÐE 1,2H ; ÐE 0,5F ; / 0,5 kg

Arah pecahan ketiga Dengan menggunakan hukum kekekalan momentum kita akan memperoleh

ÐE ÐE ÐE ÐE/ # ÐE/ ÐE ÐE ÐE ÐE/ 0 1,2H 0,5F 1,2H 0,5F kg m/s Secara grafik, vektor ÐE/ dapat digambarkan pada gambar di samping

7 ;6t +t,

t,- 112,62]

Jadi pecahan ketiga akan bergerak ke arah 112,62sumbu-y positif

Energi kinetik pecahan ketiga

/ ?¡Y¡ t,Yt,Y, 1,69 J

1 ; ¶ 0,05 kg m2 0,1 ; sin 0,6

Arah gerak benda

Perhatikan gambar berikut

Dari gambar di atas dapat dilihat bahwa untuk menentukan arah gerak dari benda, cukup dengan membandingkan besarnya gaya yang bekerja pada yaitu besarnya gaya yang bekerja pada yaitu

Sekarang akan kita periksa besarnya gaya pada masing-masing benda 89 sin :9V sin :9 cos

Dengan menggunakan hukum kekekalan momentum kita akan memperoleh E kg m/s

dapat digambarkan pada gambar di

Jadi pecahan ketiga akan bergerak ke arah 112,62o terhadap

Dari gambar di atas dapat dilihat bahwa untuk menentukan arah gerak dari benda, cukup yaitu sin 89 dengan

masing benda

Page 66: FISIKA TPB

sin :9 cos 17,6 N Dengan memasukkan nilai-nilai yang telah diperoleh didapatkan bahwa sin 89 Sehingga benda bergerak ke bawah dan benda bergerak ke atas

b. Ditanyakan : Percepatan benda Jawab : Percepatan untuk benda sama dengan benda ; ; ; Hukum Newton untuk masing-masing benda sin 3 8 ; 3 ; 3 3 ¶ @

eY Dengan menjumlahkan persamaan-persamaan tersebut akan diperoleh

sin 8 + ¹eY- ;

Gaya gesekan kinetik diberikan oleh 8 :V : cos Substitusi akan menghasilkan

; ¨©ªtYtfg ²³¨Y è

éY 1,08 m/s2

c. Ditanyakan : Tegangan tali 3dan 3

Jawab : Lihat kembali gambar diagram di atas sin 3 8 ; # 3 sin : cos ; 24 3,2 4,32 16,48 N 3 ; # 3 ; 10 1,08 11,08 N

3. Diketahui : W 5 kg ; È 2 m ; B B( ; 1mm3 â 20 u 10 N/m2 ; â( 10 u 10N/m2

a. Ditanyakan : Regangan pada masing-masing kawat

Jawab : Dari hukum Hooke didapatkan hubungan _ â ∆à

à , dengan ∆àà adalah regangan dan

_

adalah tegangan Karena kawat terhubung secara seri, maka (

Regangan kawat baja ÕãÕ_ Õ

ãÕ_ 2,5 u 10t

Regangan kawat tembaga NãN_ Õ

ãN_ 5 u 10t

b. Ditanyakan : Tegangan pada masing-masing kawat

Jawab : Karena gaya yang bekerja pada masing-masing kawat dan luas penampang masing-masing kawat adalah sama, maka tegangan pada masing-masing kawat adalah sama, yaitu sebesar

Tegangan _ Õ

_ 5 u 10ß N/m2

4. Diketahui : ∆ 2,6 u 10ts s ; 0,8< a. Ditanyakan : Waktu hidup yang diukur oleh pengamat di laboratorium Jawab : Pengamat di laboratorium akan mengamati waktu hidup meson lebih lama dari

seharusnya (karena meson bergerak mendekati kecepatan cahaya) yaitu

Page 67: FISIKA TPB

∆ ∆(OltêY

ëY ,0uæì

ltO,ìëYëY

4,33 u 10ts s

b. Ditanyakan : Jarak yang ditempuh Jawab : ∆È ∆ 0,8 u 10s4,33 u 10ts 10,4 m 5. Diketahui : í 1 m ; j 1,2 atm 1,2 u 1,013 u 10 Pa a. Ditanyakan : Tekanan di titik A sebelum kran B dibuka Jawab : j_ j Í` 1,3156 u 10 Pa b. Ditanyakan : Kecepatan keluarnya air di B Jawab : Dengan menggunakan persamaan Bernoulli untuk titik B dan C akan diperoleh

j Í` Í j Í`

Í

Karena tangki memiliki luas penampang yang jauh lebih besar dibandingkan lubang, maka

pergerakan fluida di C dapat diabaikan ( 0). Maka akan didapatkan

j Í` j Í

l¼ëÒît¼OÒ 7,78 m/s

c. Ditanyakan : Tinggi kolom air di atas titik A Jawab : Untuk mencari ketinggian air di atas titik A kita harus cari terlebih dahulu tekanan di

titik A karena ketinggian air di atas titik A berkaitan dengan tekanan di titik A. Dengan persamaan Bernoulli didapatkan

j@ Í`@ Í@ j Í`

Í , j j

Karena @ ` maka

j@ j Í

Í@

Dengan menggunakan hubungan _ _[_ W didapatkan

j@ j Í ï1 +_[

_-ð

Air dalam pipa di atas titik A diam sehingga hukum yang berlaku adalah hukum fluida statik dengan persamaan j@ j Í`

j Í ï1 +_[

_-ð j Í`

Maka didapatkan tinggi kolom di atas titik A

` IÕYñtï[ðYò 2,27 m

6. Diketahui : 6 0,01 ; 3 360 K ; 3 300 K ; c 8,314 a. Ditanyakan : Kerja yang dilakukan mesin dalam satu siklus Jawab :

Langkah AB : Proses isotermik # d@ 6c3 ln +ÔÕÔK-

Karena ketika proses ekspansi, volume kerja menjadi dua kali lipat. Maka didapatkan Ë 2Ë@, sehingga d@ 6c3 ln 2

Langkah BC : Proses isokhorik # d 0 (Ë Ë@)

Page 68: FISIKA TPB

Langkah CD : Proses isotermik # d& 6c3 ln +ÔóÔë -

Karena ketika proses kompresi, volume kerja menjadi setengah dari volume semula, maka d& 6c3 ln 2 Langkah DA : Proses isokhorik # d&@ 0 (Ë& Ë@) Jadi kerja yang dilakukan mesin dalam satu siklus adalah d d@ d d& d&@ 6c3 ln 2 6c3 ln 2 3,46 J b. Ditanyakan : Kalor yang masuk dalam satu siklus Jawab : Langkah AB : Proses isotermik Ì@ d@ 6c3 ln 2 (kalor masuk ke sistem) Langkah BC : Proses isobaric Ì ?3 3 (kalor keluar dari sistem) Langkah CD : Proses isotermik Ì& d& 6c3 ln 2 (kalor keluar ke sistem) Langkah DA : Proses isobaric Ì&@ ?3 3 (kalor masuk ke sistem) Jadi kalor yang masuk dalam satu siklus

Ìab 6c3 ln 2 6c3 3

Dengan menganggap gas ideal monoatomik maka ? 6c sehingga

Ìab 6c3 ln 2 6c3 3 33,22 J

c. Ditanyakan: Efisiensi mesin

Jawab : × ÖÙ u 100% /,0

//, u 100% 10,41%

Page 69: FISIKA TPB

SOAL UJIAN TENGAH SEMESTER II

FISIKA DASAR IA

TAHUN 2006/2007 1. Kalian diberikan dua buah silinder besi yang memiliki massa sama dan panjang sama È. Silinder

pertama pejal dengan jari-jari c dan silinder kedua berongga konsentris dengan jari-jari dalam c dan jari-jari luar c. Kedua silinder dilepaskan pada bidang miring yang memiliki koefisien gesekan statik :9 0,3 dan koefisien gesekan kinetik : 0,2. Berdasarkan informasi tersebut tentukan :

Setelah dilepas, potongan plastik melakukan gerak osilasi dengan kecepatan sudut . Tentukan kecepatan sudut tersebut! (Petunjuk : Analogikan gaya yang dialami plastik dengan hukum Hooke. Cari “konstanta pegas” berdasarkan analogi tersebut. Lalu cari kecepatan sudut osilasi) a. Momen inersia silinder pejal dan silinder berongga dinyatakan dalam dan c b. Sudut kemiringan bidang miring agar silinder pejal tetap menggelinding sempurna (tidak slip)

dan sudut kemiringan bidang miring agar silinder berongga tetap menggelinding sempurna (catatan : untuk masing-masing silinder, sudut kemiringan bidang bukan satu tetapi banyak), dan

c. Sudut kemiringan bidang miring agar kedua silinder tetap menggelinding sempurna d. Jika kedua silinder dilepaskan dari ketinggian ` 4 meter pada bidang miring, tentukan laju

translasi pusat massa masing-masing silinder saat tepat mencapai dasar bidang miring jika dua silinder menggelinding sempurna (gunakan teorema usaha energi)

2. Kerjakan poin-poin berikut a. Sebanyak 100 mL air dimasukkan ke dalam sebuah gelas ukur. Ke dalam air tersebut kemudian

dimasukkan sepotong plastik (plastik terapung di air) hingga permukaan air dalam gelas ukur menunjuk angka 150 mL. HItunglah massa potongan air tersebut. Diberikan rapat massa air 1 gr/mL

b. Ke dalam gelas ukur sejenis dimasukkan air garam (terlarut sempurna) hingga permukaannya menunjuk angka 100 mL, kemudian potongan plastik yang sama dimasukkan sehingga permukaan air garam naik ke level 140 mL. Hitunglah massa jenis air garam

c. Anggap potongan plastik berbentuk silinder dengan panjang È dan luas penampang . Kemudian plastik ditekan sehingga masuk sejauh x dari posisi setimbang (anggap x sangat kecil). Berapa gaya yang dialami potongan plastik saat tiba-tiba tangan dilepas? Anggap massa jenis plastik Í? dan massa jenis zat cair Í dan penampang gelas ukur jauh lebih besar daripada penampang plastik

3. Lumpur Lapindo Brantas di Porong, Jawa Timur berasal dari reservoir yang berada pada

kedalaman sekitar 2 km di bawah permukaan tanah. Lumpur dari kedalaman tersebut bergerak keluar melalui lubang pengeboran vertikal dan muncrat hingga ketinggian 30 meter di atas permukaan tanah. Jika massa jenis lumpur sekitar 1,3 g/cm3, tentukan

a. Kecepatan lumpur tepat mencapai permukaan tanah b. Luas penampang lubang tempat keluar lumpur, dan c. Tekanan lumpur di permukaan atas reservoir (anggap percepatan gravitasi hingga kedalaman 2

km sama dengan di permukaan tanah dan kecepatan lumpur di permukaan reservoir nol) 4. Diketahui suatu siklus yang berupa sistem 6 mol gas ideal monoatomik. Proses dari titik mula A

kembali ke A, yaitu A-B-C-D-A berturut-turut adalah isothermal, isokhorik, isothermal, isokhorik. a. Tentukan perubahan energi dalam, kerja yang dilakukan lingkungan pada gas dan kalor pada

masing-masing tahap siklus : AB, B C, C D, D A b. Pada tahap siklus mana kalor masuk ke dalam sistem dan pada tahap siklus mana kalor keluar

meninggalkan sistem. Beri alasannya c. Berapa kerja total yang dilakukan lingkungan pada gas dalam satu siklus dan efisiensi mesin?

(Catatan : hati-hati pada tanda pada definisi kerja)

Page 70: FISIKA TPB

5. Tiga orang sahabat : Mega, Buana, dan Topan melakukan relativistik. Mereka merancang dua buah

pesawat yang panjangnya masing-masing 10 meter dan dapat bergerak dengan kecepatan mendekati kecepatan cahaya. Buana dan Topan masing-masing mengawaki pesawat sedangkan Mega mengamati dari bumi. Pada saat percobaan, tiba-tiba pesawat Buana dan Topan melintasi bumi dalam arah yang sama dan Mega mencatat bahwa panjang pesawat Buana adalah 6 meter dan panjang pesawat Topan adalah 8 meter. Berdasarkan informasi tersebut

a. Berapa laju pesawat Topan dan Buana terhadap bumi b. Berapa laju relative pesawat Buana terhadap pesawat Topan c. Berapa panjang pesawat Buana menurut Topan dan panjang pesawat Topan menurut Buana d. Misalkan pada dua pesawat masing-masing ada jam yang dapat diamati oleh Buana maupun

Topan. Ketika Buana mencatat jamnya sudah bergerak selama 1 menit, berapa lama pergerakan jam tersebut menurut Buana?

Page 71: FISIKA TPB

SOLUSI UJIAN TENGAH SEMESTER II

FISIKA DASAR IA

TAHUN 2006/2007 1. Diketahui : :9 0,3 ; : 0,2 a. Ditanyakan : Momen inersia silinder pejal dan berongga Jawab : Massa silinder pejal memenuhi mcÈÒ Massa silinder berongga memenuhi mc cÈÒ Karena diminta momen inersia dalam variabel c, maka kita cari terlebih dahulu hubungan c dan c Diketahui massa silinder sama sehingga berlaku mc cÈÒ mcÈÒ # c 2c

c √2c Momen inersia silinder pejal terhadap sumbu pusat adalah

¶? c

Momen inersia silinder berongga terhadap sumbu pusat

¶Z c c

2c c / c

b. Ditanyakan : Sudut kemiringan bidang agar silinder tidak slip Jawab : Agar tetap menggelinding sempurna, bidang miring harus bisa menyediakan gaya

gesek yang dibutuhkan oleh silinder untuk menggelinding sempurna. Untuk menghitung besar gaya gesek yang bekerja pada silinder, pertama-tama tinjau persamaan gaya sistem

∑ ; sin 7 89 ;Z Lalu tinjau persamaan torsi sistem silinder pejal ∑ Þ ¶ 89c

c

Catatan : gaya berat tidak memberikan torsi karena bekerja pada pusat sumbu rotasi Untuk gerak menggelinding sempurna berlaku c ;

89 c

;

Substitusi menghasilkan

sin 7 89 +§Ç - # 89

/ sin 7

Gaya gesek maksimum yang bisa disediakan lantai adalah 89,@2 :9V dengan V cos 7.

Jadi agar silinder dapat menggelinding tanpa slip harus terpenuhi 89 89,@2

/ sin 7 :9 cos 7

tan 7 q 3:9 # 7 q 42] Sehingga didapatkan agar silinder pejal tidak slip maka sudut kemiringan bidang harus lebih

kecil atau sama dengan 42o

Page 72: FISIKA TPB

Dengan cara yang sama didapatkan bahwa sudut kemiringan silinder berongga agar tetap

menggelinding memenuhi tan 7 qß/ :9 # 7 q 35]

c. Ditanyakan : Sudut kemiringan bidang agar kedua silinder tidak slip Jawab : Agar kedua silinder dapat tetap bergerak menggelinding maka sudut kemiringan harus

kurang dari 35] karena daerah sudut tersebut yang dipenuhi oleh silinder pejal maupun silinder berongga

d. Ditanyakan : Laju translasi pusat massa masing-masing silinder Jawab : Untuk menentukan laju translasi pusat massa masing-masing, kita dapat menggunakan

hukum kekekalan energi. Akan tetapi harus diperhatikan bahwa gaya gesek statik tidak melakukan kerja karena tidak ada pergeseran antara permukaan bola dengan lantai selama pergerakan bola menggelinding.

\@@^ \@AaZ ` \iZ](@9a \i(Z@b9^@9a Untuk silinder pejal

`

¶?

`

+ c-

Untuk gerak menggelinding sempurna dipenuhi c

` /

Sehingga didapatkan

l/ ` 7,3 m/s

Dengan cara yang sama, laju translasi silinder berongga adalah

l/ ` 6,8 m/s

2. Jawab : a. Karena plastik terapung dalam cairan, maka ∑ ?^@9(a @?b 0 ?^@9(a Í@aZË@aZ ?^@9(a Í@aZË@aZ 50 gr b. Berat plastik sama dengan berat air garam yang dipidahkan ?^@9(a Í@aZ @Z@Ë@aZ @Z@

Í@aZ @Z@ JK§NgÔKô õKôKÏ 1,25 gr/mL

c. Sebelum ditekan, plastik berada dalam kesetimbangan dengan persamaan gaya ∑ 0 @?b ?^@9(a 0 , dengan @?b Í@Ëk^? Ketika ditekan sejauh M, maka Ëk^? plastik akan bertambah menjadi Èk^? M Sehingga ketika dilepaskan ∑ @?b ?^@9(a ∑ Í@SÈk^? MU ?^@9(a ∑ Í@Èk^? Í@M ?^@9(a Karena telah didapatkan Ík^?Èk^? ?^@9(a 0 maka ∑ Ík^?M

Page 73: FISIKA TPB

3. Diketahui : B 2 km ; 30 m ; Í 1,3 g/cm3 a. Diketahui : Kecepatan lumpur saat mencapai permukaan tanah Jawab : Misalkan kecepatan lumpur saat tepat mencapai permukaan adalah , maka dengan

kecepatan tersebut partikel-partikel air akan mencapai ketinggian maksimum 30 m dari permukaan tanah. Tinjau proses naiknya partikel lumpur tersebut dengan hukum kekekalan energi

\@@^ \@AaZ

`

v2` 24,5 m/s b. Ditanyakan : Luas penampang lubang Jawab : Gunakan hubungan debit lumpur, kecepatan aliran, dan luas penampang Ì u

,0, 0,05 m2

c. Ditanyakan : Tekanan lumpur di permukaan atas reservoir Jawab : Untuk mencari tekanan lumpur di permukaan, kita gunakan hukum Bernoulli antara

titik di permukaan reservoir (keadaan 1) dan di permukaan tanah (keadaan 2)

j Í` Í j Í`

Í

Berdasarkan asumsi yang diberikan, kecepatan lumpur di permukaan reservoir 0. Karena

keadaan 2 merupakan keadaan pada permukaan tanah tempat keluar lumpur maka j 1,01 u10 Pa (tekanan atmosfer) Kemudian persamaan Bernoulli akan menjadi

j j Í Í` `, dengan ` 2 km

j 1,01 u 10 130024,5 1300102000 2,65 u 10ß Pa

4. Diketahui : Siklus 6 mol gas ideal monoatomik Siklus A-B-C-D-A, isothermal-isokhorik-sotermal-isokhorik a. Ditanyakan : ∆, d, dan Ì masing-masing siklus Jawab : Tahap A-B : proses isothermal ∆ 0

d L j BË L beYÔ

ÔYÔ BË 6c3 ln +ÔYÔ-

Ì ∆ d 6c3 ln +ÔYÔ-

Tahap B-C : proses isokhorik d L j BË 0

∆ I∆3 ; untuk gas monoatomik I / 6c

∆ / 6c∆3 /

6c3 3

Ì ∆ d / 6c3 3

Tahap C-D : proses isothermal ∆ 0

Page 74: FISIKA TPB

d L j BË L beÔ

ÔÔY BË 6c3 ln +ÔÔY-

Ì ∆ d 6c3 ln +ÔÔY-

Tahap D-A : proses isokhorik d 0

∆ I∆3 / 6c∆3 /

6c3 3

Ì ∆ d / 6c3 3

b. Ditanyakan : Menentukan kalor yang masuk dan keluar sistem Jawab : Tahap A-B

Ì 6c3 ln +ÔYÔ-, karena Ë Ë maka Ì 0 sehingga kalor masuk ke dalam sistem

Tahap B-C

Ì / 6c3 3, karena 3 3 maka Ì 0 sehingga kalor keluar dari sistem

Tahap C-D

Ì 6c3 ln +ÔÔY-, karena Ë Ë maka Ì 0 sehingga kalor keluar dari sistem

Tahap D-A

Ì / 6c3 3, 3 3 maka Ì 0 sehingga kalor masuk ke dalam sistem

c. Ditanyakan : Kerja total dan efisiensi mesin Jawab : d d_W dW¥ d¥£ d£_

d 6c3 ln +ÔYÔ- 0 6c3 ln +Ô

ÔY- 0

d 6c3 ln +ÔYÔ- 6c3 ln +ÔY

Ô- 6c3 3 ln +ÔYÔ-

Kalor yang masuk ke dalam sistem Ìab Ì_W Ì£_

Ìab 6c3 ln +ÔYÔ- /

6c3 3

Sehingga didapatkan efisiensi

× ÖÙ beYt öª+÷Y÷-

beY öª+÷Y÷-¡YbeYt Yt öª+÷Y÷-

Y öª+÷Y÷-¡YYt

5. Diketahui : È 10 m ; ÈW 6 m ; È 8 m a. Ditanyakan : Laju pesawat Buana terhadap bumi Jawab : Panjang pesawat Buana ÈW lebih pendek dari sebenarnya È

ÈW È , Ê ltêY

ëY 1

ÈW àOø Èl1 I[Y

Y # W <l1 à[YàOY <l1 0YY 0,8<

Dengan menggunakan cara yang sama, laju pesawat Topan terhadap bumi menurut Mega adalah

<l1 àùYàOY l1 sYY 0,6<

b. Ditanyakan : Laju relatif pesawat Buana terhadap pesawat Topan Jawab : Gunakan formula pertambahan kecepatan secara relativistik

Page 75: FISIKA TPB

W I[tIùtê[êùëY

0,38<

c. Ditanyakan : Panjang pesawat Buana menurut Topan dan sebaliknya Jawab : Topan melihat Buana bergerak dengan kecepatan W

ÈW È maka ÈW àOø , Ê X 1

ÈW Èl1 I[ùYY 9,2 m

Buana akan melihat Topan dengan laju yang sama pula W 0,38< sehingga Buana pun

akan melihat panjang pesawat Topan menjadi lebih pendek dari yang sebenarnya. Karena laju relatif kedua pesawat adalah sama, maka ÈW ÈW 9,2 m

d. Ditanyakan : Pergerakan jam menurut Buana Jawab : Topan yang melihat Buana bergerak dengan kecepatan W 0,38< akan melihat

seluruh pergerakan Buana menjadi lebih lambat. Hal ini mengakibatkan Topan akan mencatat waktu yang lebih lama (∆) daripada yang dicatat Buana (∆)

∆ ∆ maka ∆ Ê∆ , Ê X 1

∆ ∆(Oltêù[Y

ëY 1,1 menit

Di saat yang sama, Buana akan melihat Topan bergerak dengan laju yang sama pula W 0,38< sehingga Buana akan melihat seluruh pergerakan Topan lebih lambat dari yang

sebenarnya. Hal ini akan mengakibatkan Buana mencatat waktu yang lebih lama ∆ pula dari apa yang dicatat oleh Topan (∆)

∆ ∆(Oltêù[Y

ëY 1,1 jam

Page 76: FISIKA TPB

1. Sebuah silinder baja dengan diameter 4,8 cm menyembul dari suatu dinding bangunan dengan arah

horizontal sepanjang 5,3 cm. Sebuah beban dengan massa 1200 kg digantung pada ujung luar silinder baja tersebut. Modulus geser (shear modulus) silinder baja adalah mengabaikan massa silinder baja

a. Hitung tegangan geser (shear stress) pada silinder baja b. Tentukan perbandingan secara vertikal ujung silinder baja tersebut relatif terhadap posisinya

ketika belum mendapat beban 2. (Fisika IA) Sebuah bola pa 0,56 g (massa terdistribusi seragam) menggelinding murni di

permukaan dalam suatu lintasan berbentuk lingkaran dengan jaric 15 cm. Bola mulai bergerak dari ketinggian dimana pusat massanya berada di ketingg(¶]^@ 2/5 m(d/2)2 relatif terhadap sumbu yang melalui pusat massanya)

a. Hitung energi kinetik benda saat berada di titik terendah b. Hitung besar kecepatan linier benda saat berada di titik terendah c. Hitung besar gaya normal terhadap benda ketika bola mencapai titik terendah 2. (Fisika IB) Pada gambar di samping, sebuah pegas diikatkan pada suatu penyangga statis dan ujung

lainnya diikatkan pada piston 1 (luas penampang A) suatu pompa hidrolik. Sebuadengan massa yang sangat ringan dan dapat diabaikan diletakkan pada piston 2 (luas penampang A/18). Pada awalnya, pegas berada pada keadaan setimbang (konstanta pegas Sejumlah pasir yang harus dimasukkan ke tabung silinder aga

3. Pada gambar di samping, air mengalir dari pipa bagian kiri

dengan jejari C 2.00c melalui pipa bagian tengah dengan jejari c yang mengecil ke pipa bagian kanan dengan jejari C/ 3.00c. Kecepatan air saat berada di pipa bagian tengah adalah 0.5 m/s (massa jenis air

a. Berapa kecepatan air pada pipa bagian kiri b. Berapa kecepatan air pada pipa bagian kanan c. Berapa usaha yang dilakukan untuk memindahkan

bagian kanan

SOAL UJIAN TENGAH SEMESTER II

FISIKA DASAR I

TAHUN 2007/2008

baja dengan diameter 4,8 cm menyembul dari suatu dinding bangunan dengan arah horizontal sepanjang 5,3 cm. Sebuah beban dengan massa 1200 kg digantung pada ujung luar

baja tersebut. Modulus geser (shear modulus) silinder baja adalah mengabaikan massa silinder baja

Hitung tegangan geser (shear stress) pada silinder baja Tentukan perbandingan secara vertikal ujung silinder baja tersebut relatif terhadap posisinya ketika belum mendapat beban

(Fisika IA) Sebuah bola padat dengan diameter B 0,5 cm dan massa g (massa terdistribusi seragam) menggelinding murni di

permukaan dalam suatu lintasan berbentuk lingkaran dengan jari-jari cm. Bola mulai bergerak dari ketinggian dimana pusat

massanya berada di ketinggian R dari titik terendah permukaan lintasan relatif terhadap sumbu yang melalui pusat

Hitung energi kinetik benda saat berada di titik terendah Hitung besar kecepatan linier benda saat berada di titik terendah Hitung besar gaya normal terhadap benda ketika bola mencapai titik terendah

Pada gambar di samping, sebuah pegas diikatkan pada suatu penyangga statis dan ujung pada piston 1 (luas penampang A) suatu pompa hidrolik. Sebua

dengan massa yang sangat ringan dan dapat diabaikan diletakkan pada piston 2 (luas penampang A/18). Pada awalnya, pegas berada pada keadaan setimbang (konstanta pegas Sejumlah pasir yang harus dimasukkan ke tabung silinder agar pegas tertekan sejauh 5 cm?

Pada gambar di samping, air mengalir dari pipa bagian kiri melalui pipa bagian tengah dengan

yang mengecil ke pipa bagian kanan dengan jejari . Kecepatan air saat berada di pipa bagian tengah m/s (massa jenis air 1000 kg/m3)

Berapa kecepatan air pada pipa bagian kiri Berapa kecepatan air pada pipa bagian kanan Berapa usaha yang dilakukan untuk memindahkan 0.4 m3 air dari p

baja dengan diameter 4,8 cm menyembul dari suatu dinding bangunan dengan arah horizontal sepanjang 5,3 cm. Sebuah beban dengan massa 1200 kg digantung pada ujung luar

baja tersebut. Modulus geser (shear modulus) silinder baja adalah 3 u 10 N/m2. Dengan

Tentukan perbandingan secara vertikal ujung silinder baja tersebut relatif terhadap posisinya

Hitung besar gaya normal terhadap benda ketika bola mencapai titik terendah

Pada gambar di samping, sebuah pegas diikatkan pada suatu penyangga statis dan ujung pada piston 1 (luas penampang A) suatu pompa hidrolik. Sebuah tabung silinder

dengan massa yang sangat ringan dan dapat diabaikan diletakkan pada piston 2 (luas penampang A/18). Pada awalnya, pegas berada pada keadaan setimbang (konstanta pegas 3 u 10 N/m).

r pegas tertekan sejauh 5 cm?

air dari pipa bagian kiri ke pipa

Page 77: FISIKA TPB

4. Suatu sistem gas ideal monoatomik yang memiliki tekanan awal 6 u 10 N/m2 pada suhu 330 K mengalami pemuaian volume dari 550 cm3 menjadi 1500 cm3. Jika proses pemuaian itu adalah isothermal, tentukan

a. Tekanan akhir b. Usaha yang dilakukan oleh sistem Jika proses pemuaian tersebut adalah adiabatik, tentukan c. Usaha yang dilakukan sistem 5. Terdapat kerangka acuan inersia ú bergerak dengan kecepatan relatif terhadap kerangka acuan ú,

dengan arah M. Pengamat di ú mendeteksi dua kedipan cahaya. Mula-mula kedipan cahaya merah terjadi pada M 1200 m. Kedipan cahaya biru terjadi 5 µs kemudian pada M 480 m. Untuk pengamat di ú, kedua peristiwa kedipan cahaya tersebut terjadi pada M yang sama.

a. Tentukan kecepatan relatif ú dalam c (kecepatan cahaya) b. Apakah ú bergerak ke arah M positif atau M negatif? c. Untuk pengamat di ú, kedipan mana yang terjadi pertama-tama? d. Untuk pengamat di ú, berapa interval waktu antara kedua peristiwa kedipan cahaya?

Page 78: FISIKA TPB

SOLUSI UJIAN TENGAH SEMESTER II

1. Diketahui : B 4,8 cm ; a. Ditanyakan : Tegangan geser Jawab :

_

û&Y 6,63 u 100

b. Ditanyakan : Perpindahan Jawab :

_ ü ∆2

A # ∆M Aý_

2. (FISIKA IA)

Diketahui : B 0,5 cm ;

a. Ditanyakan : Energi kinetik saat di titik terendah Jawab : Energi kinetik berasal dari energi potensial \i ` c \i 8,26 u 10t J b. Ditanyakan : Kecepatan linier benda saat di titik terendah Jawab : Karena bola menggelinding maka energi kinetik terdiri dari komponen kinetik

translasi dan rotasi, sebagai berikut \i \i(Z@b9^@9a \i

Karena bola menggelinding sempurna maka terpenuhi

\i

Sehingga didapatkan

c. Ditanyakan : Gaya normal saat benda di titik terendah Jawab : Untuk mencari gaya normal digunakan hukum Newton

∑ IYZ

V V + 2. (FISIKA IB) Diketahui : ; Ditanyakan : Jumlah pasir yang dimasukkan Jawab : j?@9aZ j?k@9

JK§ô_/s 2

_

SOLUSI UJIAN TENGAH SEMESTER II

FISIKA DASAR I

TAHUN 2007/2008

cm ; 5,3 cm ; 1200 kg

Tegangan geser

N/m2

Perpindahan vertikal

1,17 u 10t m

cm ; 0,56 g ; c 15 cm ; ¶]^@ B/2

Energi kinetik saat di titik terendah

Energi kinetik berasal dari energi potensial B 2⁄ dengan merupakan ketinggian pusat massa bola

Kecepatan linier benda saat di titik terendah Karena bola menggelinding maka energi kinetik terdiri dari komponen kinetik

tasi, sebagai berikut \iZ](@9a , dengan ¶

+&-

Karena bola menggelinding sempurna maka terpenuhi +&-

ß

Sehingga didapatkan lþß 1,45 m/s

Gaya normal saat benda di titik terendah Untuk mencari gaya normal digunakan hukum Newton

IYZ # V IY

Z

+ IYet& ⁄ - 1,36 u 10t N

/18 ; 3 u 10 N/m ; ∆M 5 cm

Jumlah pasir yang dimasukkan ?k@9

# ?@9aZ 2s 8,3 kg

merupakan ketinggian pusat massa bola

Karena bola menggelinding maka energi kinetik terdiri dari komponen kinetik

+ -

Page 79: FISIKA TPB

3. Diketahui : C 2c ; C 3c ; 0,5 m/s a. Ditanyakan : Kecepatan air pada pipa bagian kiri Jawab : m2c mc

0,5 0,125 m/s

b. Ditanyakan : Kecepatan air pada pipa bagian kanan Jawab : // mc m3c

« 0,5 0,056 m/s

c. Ditanyakan : Usaha untuk memindahkan 0,400 m3

Jawab : d ∆\i /

2,5 J

4. Diketahui : j 6 u 10 N/m2 ; i 330 K ; Ë 500 cm3 ; Ë 1500 cm3 a. Ditanyakan : Tekanan akhir Jawab : Tekanan akhir pada proses isothermal jË jË

j ¼ÔÔY 2 u 10 N/m2

b. Ditanyakan : Usaha yang dilakukan sistem Jawab : Usaha oleh sistem pada proses isotermal d L j BË Karena j merupakan fungsi dari Ë, maka j disubstitusi menggunakan hubungan gas ideal jË 6c3

d L beÔ

BË 6c3 ln +

- 328 J

c. Ditanyakan : Usaha jika pemuaiannya adiabatic Jawab : d L j BË Untuk mencari usaha dalam proses adiabatik, akan digunakan pendekatan Ì d ∆ , Ì 0 (adiabatic)

d ∆ / 6c3 3)

Diketahui jËø konstan , jË 6c3

+beÔ - Ëø konstan

3Ëøt konstan 3Ëøt 3Ëøt

3 ÔOæÔæ 3 , untuk gas monoatomik Ê ¥J

¥ê /

3 + -

¡t 330 K 158 K

Maka didapatkan usaha

d / 0,1098,31158 330 234 J

5. Diketahui : M 1200 m ; M 480 a. Ditanyakan : Kecepatan relatif ú dalam <

Page 80: FISIKA TPB

Jawab : Pertama-tama kita asumsikan kerangka 4 bergerak ke arah sumbu-x positif dengan kecepatan

Hubungan antara M pada kerangka 4 terhadap M di kerangka 4 adalah M ÊM Kedipan pertama di Mpada di kerangka 4 M ÊM Sementara kedipan kedua terjadi di M pada M ÊM Sehingga dapat dituliskan M M ÊM M < ~ Dalam soal diketahui bahwa dalam kerangka 4 kedipan cahaya terjadi di tempat yang sama

atau M M. Kedipan kedua terjadi 5µs kemudian dari kedipan pertama ( 5 µs atau 5 µs). Substitusikan seluruh nilai yang diketahui sehingga akan didapatkan 0 Ê1200 480 5 u 10t0~ 1,44 u 10s 0,48< (berlawanan dengan arah pergerakan yang diasumsikan) b. Ditanyakan : Arah gerak ú Jawab : Kerangka 4 bergerak ke arah sumbu-x negatif (berlawanan dengan asumsi awal ke

arah sumbu-x positif) c. Ditanyakan : Kedipan yang lebih dulu terjadi untuk pengamat di ú Jawab : Digunakan pendekatan seperti (a)

Ê + I2 - Ê + I2Y

-

Dapat dituliskan

Ê I2Yt2Y Ê3,85 u 10t0 s

Karena , maka dalam kerangka 4 kedipan cahaya merah akan teramati lebih dulu dibandingkan cahaya biru

d. Ditanyakan : Interval waktu antara kedua kedipan cahaya untuk pengamat di ú Jawab : Gunakan hubungan yang diturunkan pada (c)

Ê3,85 u 10t0 /,suælt+O,ûìë

ë -Y

∆ 4,39 u 10t0 s

Page 81: FISIKA TPB

SOAL UJIAN TENGAH SEMESTER II

FISIKA DASAR I A

TAHUN 2008/2009

1. Sebuah derek mengangkat balok besi (Ík9a = 7,990 x 10/ kg/ /dari dalam air laut (Í@aZ ^@(=

1,03 x 10/ kg/ /). Balok besi tersebut berukuran 0,25 m x 0,20 m x 10,00 m di permukaan laut yang bertekanan 1,01 x 10/ N/ . Modulus buk besi adalah 1,20 x 10 N/ .

Pada kedalaman 5,00 x 10/ m, tentukanlah (hingga empat angka berarti): a) Tekanan total yang dialami balok besi (anggap kerapatan air laut konstan). b) Volume balok besi. c) Gaya apung yang dialami balok besi.

2. Sebuah garobak kosong, bermassa m yang dapat bergerak tanpa gesekan diatas permukaan licin, kedua sisinya dihubungkan dengan dinding statik melalui dua buah pegas yang mempunyai konstanta pegas masing-masing dan seperti ditunjukkan pada gambar:

a) Jika gerobak kosong diberi sedikit simpangan sebesar ∆ ke kanan lalau dilepaskan sehingga

berosilasi harmonik sederhana, turunkan frekuensi sudut () osilasi gerobak dinyatakan dalam m, dan .

b) Jika sejumlah pasir dengan massa M ditambah ke dalam gerobak kosong kemudian gerobak diberi simpanagn sebesar ∆, tentukan perbandingan frekuensi sudut osilasi gerobak yang berisi pasir terhadap frekuensi sudut osilasi gerobak kosong.

c) Laju maksimum gerobak setelah pasir ditambahkan. 3. Sebuah gelombang merambat pada seutas tali yang tegang. Simpangan transversal partikel tali

diberikan oleh fungsi gelombang y(x,t) = 0,30 sin10m 2mM meter. Variabel x dan t masing-masing menyatakan posisi partikel tali (dalam meter) dan waktu rambat gelombang (dalam sekon). a) Tentukan arah dan laju rambat gelombang. b) Buatlah sketsa kurva y terhadap x pada t = 0 dan kurva y terhadap t pada x = 0. c) Hitunglah laju transversal partikel tali pasa x = 0,5 m dan t = 0,1 s.

4. Sebuah tangki berbentuk silinder dengan jari-jari penampang 2 m dan tinggi 5 m terisi penuh dengan air (Í@aZ = 1000 kg/ /). Di bagian dasar tangki terdapat lubang pembuangan yang berbentuk lingkaran dengan jari-jari 1 x 10t cm yang pada awalnya tertutup. Pada bagian atas tangki terdapat piston dengan massa 10 kg yang menekan seluruh permukaan air. Tekanan udara luar j = 1,01 x 10 N/ . Jika gesekan piston dengan dinding tangki diabaikan, tentukan : a) Tekanan pada kedalaman 2 m dari permukaan air. b) Laju air yang keluar dari lubang pembuangan sesaat setelah lubang pembuangan dibuka . c) Waktu yang diperlukan untuk mengkosongkan tangki.

Page 82: FISIKA TPB

5. Sejumlah n mol gas ideal monoatomik mengalami perubahan dari keadaan a ke keadaan b melalui

3 macam proses seperti ditunjukkan gambar. Kalor yang diterima gas dalam proses ab adalah 10 jaËa. Tentukanlah (dalam ja dan Ëa): a) Kalor yang dipindahkan dalam proses ac b.

b) Perubahan energi internal (∆ dalam proses ad b.

Page 83: FISIKA TPB

SOLUSI UJIAN TENGAH SEMESTER II

FISIKA DASAR I A

TAHUN 2008/2009

1. a) Tekanan yang dialami oleh alas besi adalah j j Í@aZ` 1,01 u 10 1,03 u 10/ u10 u 5000 1,01 u 10 515 u 10 516,01 u 10, tekanan yang dialami oleh tutup besi

adalah j j Í@aZ` 1,01 u 10 1,03 u 10/ u 10 u 4990 1,01 u 10 513,97 u10 514,98 u 10, tekanan yang dialami oleh selimut adalah j 4j L Í@aZB`, j 41,01 u 10 1,03 u 10/ u 10 u ∆`, j 41,01 u 10 1,03 u 10/ u 10 u 10 8,16 u 10, jadi tekanan total adalah 1039,15 u 10

b) ∆¼∆Ô ÔOÂ , dimana ∆j adalah perbedaan tekanan ketika balok besi berada di permukaan dengan

ketika berada di dalam kedalaman, jadi ∆j j(](@^ 6j 1033,09 u 10, kemudian didapat

∆Ë ÔO∆¼W ,u//,«u

,u 4,3 u 10t, ∆Ë Ë Ë, jadi Ë Ë ∆Ë, Ë 0,5 4,3 u10t 0,44957 c) _ Í@aZËk9a 4630,571 V

2. a) Persamaan gerak untuk benda tersebut adalah M &Y2&(Y, didapat lY

b) Persamaan gerak untuk benda tersebut adalah M &Y2&(Y, didapat

lYÇ , perbandingan  l

Ç

c) @9 akan terjadi jika ; 0. Persamaan gerak untuk Gerak Harmonik Sederhana adalah

M sin, &2&( cos, ; sin, ; akan nol apabila 0, jadi didapat

@9 3. a) Arah rambat gelombang adalah ke arah sumbu M positif, laju rambat gelombang adalah

|

5 4⁄

b) Kurva terhadap M pada 0

Kurva terhadap pada M 0

-0.4

-0.3

-0.2

-0.1

0

0.1

0.2

0.3

0.4

0 0.5 1 1.5 2 2.5

Page 84: FISIKA TPB

c) ' B BÂ 3m cos10m 2mM, ' pada M 0,5 dan 0,14, didapat ' 3m cosm m 3m

4. a) Tekanan piston terhadap air adalah j?a9(]b j _, dimana adalah gaya akibat berat, yaitu

10V, jadi j?a9(]b 1,01 u 10 ZY 1,01 u 10 0,07958 u 10 1,08958 u

10 V Â , tekanan pada kedalaman 2 adalah j j?a9(]b Í` 1,08958 u 10 1000 u10 u 2 1,08958 u 10 0.2 u 10 1,28958 u 10 b) Laju air pada lubang pembuangan dapat dihitung dengan hukum Bernoulli, yaitu j?a9(]b Í Í` j

Í Í`, 0,07958 u 10 Í u 0 1000 u 10 u 5

u 1000 u, 0,57958 u 10 500 u , didapat 10,76643 4⁄ c) Secara umum, karena tinggi air semakin berkurang terhadap waktu, maka dapat dinyatakan

sebagai fungsi , yaitu 0,07958 u 10 1000 u 10 u ` u 1000 u , didapat

v15,916 20`, kemudian dari persamaan debit, yaitu Ô( dapat dihitung waktu ,

_A(

v15,916 20`, A( v15,916 20`,

AY(Y 15,916 20`, ,«0 0.2157,

0,4644 4 5. a) untuk kasus ; # , ∆ Ì d, dimana d adalah usaha yang dilakukan pada proses ; # , yaitu luas daerah di bawah kurva ; # , yaitu 4jaËa, jadi didapat ∆ 10jaËa 4jaËa 6jaËa, karena ∆ hanya bergantung pada posisi awal dan akhir, maka ∆ yang

sama juga berlaku untuk proses ; # < # dan ; # B #. Sekarang untuk menemukan kalor yang dipindahkan dalam proses ; # < # maka cukup menghitung d, dimana d adalah usaha yang dilakukan pada proses ; # < # , yaitu luas daerah di bawah kurva ; # < # , didapat d 5jaËa, ∆ Ì d, 6jaËa Ì 5jaËa, Ì 11jaËa b) Perubahan energi internal pada proses ; # B # adalah 6jaËa

-0.4

-0.3

-0.2

-0.1

0

0.1

0.2

0.3

0.4

0 0.5 1 1.5 2 2.5

Page 85: FISIKA TPB

UJIAN AKHIR SEMESTER I

FISIKA DASAR I

(Untuk Mahasiswa Dengan nilai di bawah C

setelah UTS 1 dan UTS 2)

Page 86: FISIKA TPB

1. Sebuah paket bom di atas permukaan tanah bermassa

gaya 20 N seperti pada gambar. (koefisien gesekan permukaan tanah 0,2 dan kedalaman tebing 20 m)

a. Berapakah kecepatan paket bom setelah menempuh tanah sejauh 4 km?

b. Ketika di bibir tebing bom meledak dan pecah menjadi dua bagian yaitu bagian 1 = dan bagian 2 = (1/3) m. Jika serpihan bagian 1 ditemukan sejauh 120 m, dimanakah tim gegana akan menemukan serpihan bagian 2?

c. Jika ketika pecah bagian 1 mempunyai kecepatan E 6H 3F

2. Putaran roda pada gambar menggerakkan

piston yang dihubungkan dengan kedua ujung batang. Piston bergerak isolasi harmonik sederhana dengan simpangan Î cos , dimana satuan meter dan detik. Jari20 cm dengan putaran 4 putaran per detik

a. Berapa nilai amplitudo b. Berapa nilai maksimum kelajuan dan percepatan piston? 3. Sebuah tangki dengan luas permukaan 0,7 m2 diis

Tangki yang dilengkapi pistongambar

a. Berapa laju awal aliran air yang keluar dari lubang? b. Berapa debit keluarannya? 4. Sebuah peluru bermassa 10 g ditembakkan dengan

kecepatan 20 m/s ke arah sebuah keping logam bermpusatnya disangga oleh sebuah benang pengikat seperti pada gambar. Momen inersia keping

terhadap pusat rotasi diketahui

menyebabkan logam berputar

SOAL UJIAN AKHIR SEMESTER I

FISIKA DASAR I

TAHUN 2003/2004

Sebuah paket bom di atas permukaan tanah bermassa 2 kg didorong oleh tim gegana dengan N seperti pada gambar. (koefisien gesekan permukaan tanah 0,2 dan kedalaman

Berapakah kecepatan paket bom setelah

enempuh tanah sejauh 4 km? Ketika di bibir tebing bom meledak dan pecah menjadi dua bagian yaitu bagian 1 = (2/3) m dan bagian 2 = (1/3) m. Jika serpihan bagian 1 ditemukan sejauh 120 m, dimanakah tim gegana akan menemukan serpihan bagian 2?

etika pecah bagian 1 mempunyai m/s, berapakah ?

Putaran roda pada gambar menggerakkan piston yang dihubungkan dengan kedua ujung batang. Piston bergerak isolasi harmonik sederhana dengan simpangan

, dimana Î dan dalam satuan meter dan detik. Jari-jari roda adalah 20 cm dengan putaran 4 putaran per detik

dan periode tersebut? Berapa nilai maksimum kelajuan dan percepatan piston?

Sebuah tangki dengan luas permukaan 0,7 m2 diisi air. Tangki yang dilengkapi piston dengan massa 10 kg pada

Berapa laju awal aliran air yang keluar dari lubang? Berapa debit keluarannya?

Sebuah peluru bermassa 10 g ditembakkan dengan kecepatan 20 m/s ke arah sebuah keping logam bermassa 100 g dan berjaripusatnya disangga oleh sebuah benang pengikat seperti pada gambar. Momen inersia keping

terhadap pusat rotasi diketahui ¶ c. Peluru tersebut menempel pada tepi keping logam dan

menyebabkan logam berputar

kg didorong oleh tim gegana dengan N seperti pada gambar. (koefisien gesekan permukaan tanah 0,2 dan kedalaman

assa 100 g dan berjari-jari 25 cm yang pusatnya disangga oleh sebuah benang pengikat seperti pada gambar. Momen inersia keping

. Peluru tersebut menempel pada tepi keping logam dan

Page 87: FISIKA TPB

a. Hitung kecepatan putar keping logam setelah peluru tersebut menempel b. Selidiki apakah seluruh energi mekanik dapat dipakai untuk gerak rotasi sistem itu 5. Suatu ruang reaksi yang dilengkapi dengan piston penggerak memiliki volume 10-2 m3, pada suhu

ruang 27o C dan diisi oleh 1000 g gas nitrogen (N2) sampai tekanan mencapai 105 Pa. Gas N2 dianggap memenuhi sifat ideal gas

a. Tentukan berapa banyak partikel gas N2 yang ada dalam ruang reaksi tersebut b. Hitung kalor yang diperlukan untuk menaikkan suhu ruang tersebut sebesar 100 oC pada tekanan

tetap, dan hitung pula kenaikan energi dalamnya. Kapasitas molar gas pada volume tetap,

I 6c J/K

c. Selidiki apakah sistem melakukan kerja atau sebaliknya

Page 88: FISIKA TPB

SOLUSI UJIAN AKHIR SEMESTER I

FISIKA DASAR I

TAHUN 2003/2004 1. Diketahui : 2kg ; 20 N ; : 0,2 ; ` 20 m a. Ditanyakan : Kecepatan paket bom tepat di bibir tebing Jawab : Percepatan benda dapat dihitung dengan menggunakan hukum Newton 8 ; : ;

; t, 8 m/s2

Karena percepatan benda konstan maka kecepatan benda setelah menempuh jarak sejauh 4

adalah ( 2;4

( √2;4 v284 8 m/s b. Ditanyakan : Posisi serpihan bagian kedua Jawab : ]2 8 m/s ]' 0 m/s ` ` ]' 1 2⁄

lA l

2s

Sehingga M? ? 16 meter Jika M 120 meter didapatkan

M? / M

/ M

16 / 120

/ M # M 192 meter

c. Ditanyakan : Kecepatan

Jawab : '? / '

/ ' 0

' 23 6 m/s

2? / 2

/ 2 8

2 3 +8 / 6- 12 m/s

2. Diketahui : c 20 cm ; 20 cm a. Ditanyakan : Nilai amplitudo dan periode Jawab : Amplitudo/simpangan terjauh terjadi pada sisi terjauh roda dari piston yaitu pada c 20 cm, sehingga 20 cm

25 rad/s

b. Ditanyakan : Nilai maksimum kelajuan dan percepatan piston

Jawab : &2&( sin # @2 5 m/s

; &I&( cos # ;@2 1,3 u 10 m/s

3. Diketahui : 0,7 m2 ; 10 kg ; C & 0,007 m ; ` 0,6 m

Page 89: FISIKA TPB

a. Ditanyakan : Laju awal air yang keluar dari lubang Jawab : Gunakan persamaan Bernoulli di permukaan di air dan di lubang

j@ Í@ Í`@ j

Í Í`

Tekanan di permukaan air j@ j j?a9(]b dengan j?a9(]b / Tekanan di lubang adalah j @ 0 karena A pada permukaan air >> A pada lubang air

j j?a9(]b Í` j Í

+Ïõ ÒA-

Ò 2 +_Ò `- 12,3

3,5 m/s b. Ditanyakan : Debit keluaran Jawab : Ì mC 5,3 u 10t m/s

4. Diketahui : ? 10 g ; ? 20 m/s ; 100 g ; c 25 cm ; ¶ c

a. Ditanyakan : Kecepatan putar keping Dengan menggunakan hukum kekekalan momentum sudut, kita akan memperoleh Momentum sudut awal = momentum sudut akhir

??c S¶? ¶U + ?c c- ?

c

ñ JIJ+J

YÇ-eò 13,33 rad/s

b. Ditanyakan : Apakah seluruh energi mekanik dapat dipakai gerak rotasi sistem Jawab : Energi kinetik awal, \ia \ia

?? 2 J

Energi kinetik akhir, \i

\i@ ¶ + ?c

c- + ?

- c

\i@ 3,33 u 10t J \ia \i@ 2 0,033 1,967 J Sehingga pada peristiwa ini terdapat energi kinetik yang hilang sebesar 1,967 J. Dengan kata

lain tidak sepenuhnya energi kinetik peluru dipakai untuk gerak rotasi sistem 5. Diketahui : Ë 10t m3 ; 3 27 ; 1 kg ; j 10 Pa a. Ditanyakan : Jumlah partikel gas N2 Jawab : Dengan menggunakan persamaan gas ideal, akan diperoleh jË 6c3

6 ¼Ôe 0,4 mol

Jumlah partikel V 0,4V_ 0,4 u 6,02 u 10/ 2,4 u 10 partikel gas b. Ditanyakan : Kalor yang diperlukan Jawab : Kenaikan suhu ∆3 100 Kalor yang diperlukan, ∆Ì pada tekanan tetap adalah ∆Ì ?∆3 I 6c∆3 ∆Ì S

YbebeU∆3 8,31 J

Page 90: FISIKA TPB

SOAL UJIAN AKHIR SEMESTER I

FISIKA DASAR I

TAHUN 2004/2005 1. Sebuah benda memiliki kecepatan dalam arah sumbu-x yang

diperlihatkan pada grafik berikut ini. Apabila mula-mula benda tersebut berada di titik M 4, hitunglah

a. Percepatan rata-ratanya dalam selang waktu 24 hingga 44 b. Besarnya perpindahan benda 04 hingga 44 2. Benda bermassa bergerak dengan laju awal pada saat jaraknya

dengan konstanta pegas . Apabila tidak terdapat gesekan antara benda dengan lantai, tentukanlah besarnya energi yang diserap menjadi energi potensial pada pegas

3. Tiga buah partikel bermassa 4 kg, 8 kg, / 3 kg yang terletak di titik (-2,2)

m, (4,1) m, (1,-3) m, mengalami gaya eksternal. Masing-masing gaya eksternal tersebut adalah 6H N ; 16F N ; dan 14H N. Hitunglah a. Koordinat pusat massanya b. Percepatan pusat massa 4. Benda berbentuk kubus pejal dengan panjang sisinya 10 cm terapung di air dengan 25% bagiannya

muncul di permukaan. Apabila massa jenis air adalah 1 g/cm3, hitunglah massa benda tersebut 5. Sebuah benda ( 1kg) diikatkan pada sebuah pegas ( 100m N/m) berosilasi harmonik

sederhana secara horizontal pada bidang datar yang licin. Diketahui pada saat 04 simpangannya 0,032 m dan bergerak ke arah sumbu-x negatif dengan kecepatan 0,24π m/s. Tentukanlah

a. Amplitudo dan tetapan fase osilasi dengan hukum kekekalan energi b. Fungsi osilasi (dalam fungsi cosinus) 6. Satu mol gas ideal bertekanan awal 1,01 u `00 Pa mengalami proses isotermik pada suhu 350 K

sedemikian sehingga tekanannya naik menjadi 2,5 u 100 Pa a. Hitunglah volume awal dan akhir gas b. Hitunglah usaha yang dilakukan terhadap gas tersebut c. Perubahan energi dalam gas tersebut

Page 91: FISIKA TPB

1. Diketahui : M 4 a. Ditanyakan : Percepatan rata Jawab :

; ItIt 2 m/s

b. Ditanyakan : Perpindahan benda dalam selang waktu Jawab :

M4 L B M

2. Diketahui : Sistem balok pegas Ditanyakan : Energi yang diserap menjadi energi potensial Jawab : Energi yang diserap menjadi energi pegas

\@@^

\@AaZ ∆M

\?k@9

3. Diketahui : 4 kg, (-2,2) m, (4,1) m, (1, 6H N ;

a. Ditanyakan : Koordinat pusat massaJawab :

M? s

? s

b. Ditanyakan : Percepatan pusat massaJawab : 2 14 6 ' 16 N

E 8H 16 ;E? E

Ç

4. Diketahui : C 10 cm ; 25% terapung ; Ditanyakan : Massa benda Jawab : _ 0,75ÍË ÍË 5. Diketahui : 1 kg ;

SOLUSI UJIAN AKHIR SEMESTER I

FISIKA DASAR I

TAHUN 2004/2005

Percepatan rata-rata dalam selang waktu 2 s hingga m/s

Perpindahan benda dalam selang waktu 2 s hingga

0 4 4 8 m

Sistem balok pegas

Energi yang diserap menjadi energi potensial

Energi yang diserap menjadi energi pegas

\?

8 kg, / 3 kg 2,2) m, (4,1) m, (1,-3) m 16F N ; dan 14H N

Koordinat pusat massa

t0 1,8 m

t/0 0,25 m

Percepatan pusat massa

6V 8 N

16F N

0,5H F N

cm ; 25% terapung ; Í 1 gr/cm3

ËÍ 0,75ÍË 750 gram

100m N/m ; M0 0,032 m ; 0,24m

4 s

s hingga 4 s

Page 92: FISIKA TPB

a. Ditanyakan : Amplitudo dan tetapan fase Jawab :

Dari hukum kekekalan energi

M didapatkan

M , l

M # lM +IO| - 0,04 m

Misalkan Î cos maka sin , didapatkan M0 cos dan 0 sin Sehingga tan IO

|2O # 0,64 rad

b. Didapatkan : Fungsi osilasi Jawab : Fungsi osilasi dapat dituliskan sebagai M 0,032 cos10m 0,64 m 6. Diketahui : j 1,01 u 100 Pa ; 3 350 K ; j 1,01 u 100 Pa a. Ditanyakan : Volume awal dan akhir Jawab : Dari persamaan gas ideal jË 6c3 didapatkan

Ë be¼O 2,9 u 10t/ m3

Ë be¼ 1,2 u 10t/ m3

b. Ditanyakan : Usaha yang dilakukan terhadap gas Jawab :

d L jËÔÔO BË L be

ÔÔ

ÔO BË 6c3 ln ÔÔO 2,6 u 10/ J

c. Ditanyakan : Perubahan energi dalam Jawab : ∆ 6I∆3 isotermik # ∆3 0 # ∆ 0

Page 93: FISIKA TPB

SOAL UJIAN AKHIR SEMESTER I

FISIKA DASAR I

TAHUN 2007/2008

1. Sebuah motor listrik menggerakkan batu pengasah pisah (gerinda) berbentuk silinder. Kemudian

motor tersebut dimatikan pada saat gerinda memiliki laju rotasi 240 putaran/menit. Putaran gerinda tidak serta merta berhenti dan percepatan sudutnya dianggap konstan selama berputar tersebut. Setelah 2 detik laju rotasi gerinda 180 putaran/menit

a. Tentukan percepatan sudut gerinda b. Tentukan banyaknya putaran yang dialami gerinda selama 2 detik tersebut

2. Pada suatu letusan gunung api, bongkahan batu

padat terlempar keluar dari kawahnya. Bongkahan batu tersebut biasanya disebut bom vulkanik. Gambar di samping ini adalah penampang lintang gunung Merapi di Jawa Tengah

a. Pada laju awal berapa sebuah bom vulkanik dapat terlempar dari mulut kawah A dengan sudut 7 37], agar dapat jatuh di titik B, dengan jarak 2 km jarak horizontal B 10 km (abaikan pengaruh gesekan udara pada lintasan bom tersebut)

b. Berapakah waktu yang diperlukan oleh bom vulkanik tersebut untuk mencapai titik B tersebut 3. Sebuah bola titanium bermassa 300 g bergerak pada permukaan mendatar yang licin dengan laju

awal 1,2 m/s mengalami tumbukan elastik sempurna dengan bola titanium lainnya yang sedang diam dan massanya tidak diketahui. Setelah tumbukan, bola pertama bergerak dalam arah yang sama dengan arah semula dengan laju 0,6 m/s. Tentukanlah

a. Massa bola kedua b. Laju bola kedua tersebut setelah tumbukan

4. Sebuah pipa berdiameter 2 cm dialiri air. Air (rapat massa 1000 kg/m3) mengalir dalam pipa dengan laju awal 0,8 m/s dan bertekanan 150 kPa. Jika diameter pipa tersebut mengecil menjadi 1 cm dan naik menuju lantai ketiga dengan beda ketinggian 8,0 m, hitunglah a. Laju air pada lantai ketiga tersebut b. Tekanan air pada lantai ketiga tersebut

5. Bila suatu sistem gas ideal mengalami perubahan dari keadaan i menuju keadaan f melalui lintasan iaf (lihat gambar di samping), maka kalor yang diserap sistem Ì 50 J dan kerja oleh sistem pada lingkungan d 20 J

a. Jika energi dalam pada keadaan i (Ui) 10 J, hitunglah energi dalam pada keadaan f (Uf) b. Bila melalui lintasan ibf, Ì 36 J, hitunglah d bila melalui ibf c. Jika untuk kembali dari keadaan f ke keadaan i melalui lintasan fi diperlukan d 13J,

hitunglah kalor Ì yang harus dipindahkan untuk lintasan ini

Page 94: FISIKA TPB

1. Diketahui : 240 putaran/menit ; a. Ditanyakan : Percepatan sudut Jawab : 240 putaran 180 putaran/ Percepatan sudut

|t|O∆( 0ts

b. Ditanyakan : Banyaknya putaran Jawab :

7

2. Diketahui : 7 37] ; a. Ditanyakan : Laju awal bom vulkanik Jawab : Tinjau sumbu-x B cos 7 Tinjau sumbu-y

` sin 7 Substitusi nilai menghasilkan

` sin 7 + &IO ²³¨

` B tan 7

Sehingga didapatkan

l &

]9 b. Ditanyakan : Waktu yang diperlukan sampai ke titik B Jawab : Substitusikan nilai

&IO ²³¨ h

3. Diketahui : a. Ditanyakan : Massa bola kedua Jawab : Tinjau hukum kekekalan momentum sebelum dan sesudah tumbukan Tinjau persamaan koefisien restitusi À IYtI

IYtI Substitusi menghasilkan

SOLUSI UJIAN AKHIR SEMESTER I

FISIKA DASAR 1

TAHUN 2007/2008

putaran/menit ; 180 putaran/menit ; ∆ 2 detik

cepatan sudut

putaran/menit 4putaran/sekon 8m rad/s /menit 3putaran/sekon 6m rad/s

m rad/s2

Banyaknya putaran

8m2 m2 14m rad 7 putaran

2 km ; B 10 km

Laju awal bom vulkanik

menghasilkan

+ &²³¨ h-

+ &IO ²³¨ h-

&YIOY]9Yh

didapatkan &Y

]9Yh

& ª hA 287 m/s

Waktu yang diperlukan sampai ke titik B Substitusikan nilai sehingga didapatkan

u¡sß ²³¨ /ß¿ 43,6 s

300 gr ; 1,2 m/s ; 0,6 m/s

Massa bola kedua

Tinjau hukum kekekalan momentum sebelum dan sesudah tumbukan Tinjau persamaan koefisien restitusi

1 , 0 (elastik sempurna)

Substitusi menghasilkan

detik

Tinjau hukum kekekalan momentum sebelum dan sesudah tumbukan

Page 95: FISIKA TPB

Sehingga didapatkan ItI

II 0,1 kg

b. Ditanyakan : Laju bola kedua setelah tumbukan Jawab : 0,6 1,2 1,8 m/s 4. Diketahui : B 2 cm ; Í 1000 kg/m3 ; 0,8 m/s ; j 150 kPa B 1 cm ; 8 m a. Ditanyakan : Laju air pada lantai ketiga Jawab : Untuk mendapatkan laju digunakan persamaan kontinuitas debit

__Y +&

&Y- 3,2 m/s

b. Ditanyakan : Tekanan air pada lantai ketiga Jawab : Untuk mencari tekanan air kita gunakan persamaan Bernoulli

j Í Í` j

Í Í`

Untuk 0 didapatkan untuk j

j j Í Í` 65,2 kPa

5. Diketahui : Ì 50 J ; d 20 J a. Ditanyakan : Energi dalam pada keadaan f Jawab : Untuk mencari energi dalam akhir, perlu dicari terlebih dahulu perubahan energi dalam ∆

proses tersebut. Gunakan hukum termodinamika pertama Ì ∆ d ∆ Ì d 50 20 30 J a ∆ 10 30 40 J b. Ditanyakan : Kerja pada proses lintasan ibf Jawab : Ì ∆ d d Ì ∆ 36 30 6 J Catatan : pada diagram tekanan terhadap volume, berbeda dengan Ì dan d yang tergantung

pada lintasan yang diambil, perubahan energi dalam hanya ditentukan oleh posisi awal dan akhir dan tidak bergantung pada lintasan. Oleh sebab itu pada bagian (b), nilai ∆ yang digunakan sama dengan pada bagian (b) karena posisi awal dan akhir yang sama

c. Ditanyakan : Kalor untuk lintasan fi

Jawab : Ì ∆ d 30 13 17 J

Page 96: FISIKA TPB

UJIAN TENGAH SEMESTER I

FISIKA DASAR II

(Untuk Mahasiswa Dengan Nilai Di Bawah C Setelah Uts 1 Dan Uts 2)

Page 97: FISIKA TPB

SOAL UJIAN TENGAH SEMESTER I

FISIKA DASAR II

TAHUN 2002/2003

1. Sebuah dipole listrik bermuatan Ì dan –Ì, kedua muatan tersebut terpisah sejarak B a. Hitung vektor medan listrik di titik A b. Jika sebuah muatan titik 0,1Ì diletakkan di A, hitunglah gaya pada yang disebabkan oleh

dipole tersebut c. Jika titik A digeser ke posisi B dengan Ê B, hitung vector medan listrik di titik B dengan

menggunakan aproksimasi 1b 1 6; untuk 6 1 2. Kulit bola yang tebal dengan jari-jari dalam ; dan jari-jari luar bermuatan total 3Ì. Kemudian

sebuah muatan titik –Ì diletakkan di pusat bola. Tentukan : a. Besar dan arah kuat medan listrik di C ; ; ; C ; serta C b. Besar muatan induksi pada permukaan dalam dan luar kulit bola c. Gambar kurva potensial listrik di C ; ; ; C ; serta C . Asumsikan potensial di C , adalah 3. Sebuah kapasitor pelat sejajar terpisah oleh jarak 1 cm dan dengan luas penampang 0,2 m2. Mula-

mula kapasitor dihubungkan dengan sumber potensial 30 V. Setelah kapasitor jenuh muatan, sumber potensial dilepas kemudian bahan plastik bermuatan sama dengan tebal 0,5 cm disisipkan di antara pelat tersebut. Beda potensial pada keadaan ini adalah 20 V. Hitunglah

a. Besar kapasitansi sebelum diisi bahan plastik dan besar muatan pada masing-masing pelat b. Besar kapasitansi setelah diisi bahan plastik dan tetapan dielektriknya c. Besar kuat medan listrik dan rapat energi setelah diisi bahan plastik 4. Ion-ion positif dilewatkan pada selektor kecepatan yang memiliki beda potensial antara pelat

sebesar Ë dan medan magnet serba sama . Setelah keluar dari selektor kecepatan, hanya ion tertentu dengan massa dan muatan yang dapat memasuki ruang pembelok ion yang memiliki medan magnet serba sama

a. Tentukan laju ion yang akan memasuki ruang pembelok ion b. Buat sketsa lintasan ion setelah keluar dari celah ruang pembelok ion c. Tentukan posisi jatuhnya ion pada pelat detektor relative titik O, dinyatakan dalam besaran-

besaran yang diketahui 5. Perhatikan rangkaian listrik berikut ini. Bila sumber potensial 24Ë serta c cß 4 ; c c0 6 ; c/ c 3 ; dan c 1, hitunglah a. Arus yang melalui c, c, dan cß b. Beda potensial antara titik ; dan Ë@ Ë

Page 98: FISIKA TPB

SOLUSI UJIAN TENGAH SEMESTER I

FISIKA DASAR II

TAHUN 2002/2003

1. Diketahui : Muatan dipole dan –Ì terpisah sejauh B

a. Ditanyakan : Vektor medan listrik di A Jawab : Oleh muatan Ì

\_ Ù&ät&/ä&Y/&Y~¡/Y

Oleh muatan Ì

\_ Ù&ät&/ä&Y/&Y~¡/Y

Resultan

\_ \_ \_ tÙ&&Y/&Y~¡/Y

b. Ditanyakan : Gaya pada

Jawab : Gaya pada muatan di titik A adalah \_ dengan 0,1Ì maka

_ t,ÙY&&Y/&Y~¡/Y

c. Ditanyakan : Vektor medan listrik di B

Jawab : Untuk sebarang titik B (0,y) pada sumbu-y maka kuat medannya analog dengan hasil (a) dengan menggunakan y pada suku (2d)

\W tÙ&'Y/&Y~¡/Y tÙ&

'¡&/'Y~¡/Y

Aproksimasi 1b 1 6 memberikan

1 B/2~// 1 / + &

'- 1 /&Ys'Y sehingga didapatkan

\W tÙ&@¡óY

ìY

2. Diketahui : C& ; ; C ; (](@^ 3Ì

a. Ditanyakan : Besar dan arah kuat medan listrik Jawab :

Simetri persoalannya akan menghasilkan medan listrik yang berarah radial (bisa ke dalam maupun ke luar) oleh karena itu akan diselesaikan dengan hukum Gauss dengan pilihan permukaan berbentuk bola degan jari-jari C untuk masing-masing kasus C ;

\DE BE ÙO # \ ÙOZY

b. Ditanyakan : Muatan induksi Jawab : Karena \ 0 dalam kulit bola konduktor, maka menurut hukum Gauss \DE BE muatan induksi Ì@ 0 Ì Ì atau Ì@ 0

Page 99: FISIKA TPB

3. Diketahui : 0,2 m ; B 0,01 m ; Ë 30 V ; B?^@9(a 0,5 cm

a. Ditanyakan : Kapasitansi dan muatan sebelum diisi plastik Jawab :

_& ,

, 20 F

Muatan mula-mula Ì Ë 2030 600 C

b. Ditanyakan : Kapasitansi setelah diisi bahan plastik Jawab : Setelah diisi dan baterai dilepas

Ì kIË # kI ÙÔ 0O

30 F

c. Ditanyakan : Kuat medan listrik dan rapat energi

Jawab : Kuat medan listrik pada keadaan plastik terpasang

i. \ ÔO& /

, 3000 V/m

ii. \? þO // 1000 V/m

iii. Rapat energi pada bagian plastik 0,5\ 0,53000 4,5 u 100 J/m3

iv. Rapat energi pada bagian udara 0,5\? 0,531000 1,5 u 100 J/m3

4. Diketahui : Ion positif dilewatkan pada selektor kecepatan dan ruang pembelok ion a. Ditanyakan : Laju ion yang memasuki ruang pembelok ion Jawab : Ada dua gaya yang berlawanan yang bekerja pada ion dalam selektor kecepatan : gaya

listrik @ \ Ë/B, medan magnet . Ion yang akan lolos melewati selektor kecepatan adalah yang mengalami gaya @ tapi saling berlawanan arah maka

Ë/B # Ë/B

b. Ditanyakan: Lintasan ion Jawab : Dalam ruang pembelok ion megalami pembelokan karena gaya magnet (vektor E tegak lurus DE), karena gaya magnet selalu tegak lurus maka berfungsi sebagai gaya sentripetal, sehingga lintasannya berbentuk lingkaran yang membelok ke bawah

c. Ditanyakan : Posisi jatuh ion Jawab : Besar gaya magnet ketika dibelokkan

IYe # c IWY Ô&WWY

2 Ô&WWY

5. Diketahui : 24V ; c cß 4Ω ; c c0 6Ω ; c/ c 3Ω ; c 1Ω

a. Ditanyakan : Jawab : Soal ini dapat diselesaikan dengan hukum Kirchoff dengan menyederhanakan rangkaian secara bertahap c paralel dengan c/ : c/ 2 Ω c paralel dengan c0 : c0 2 Ω

Page 100: FISIKA TPB

c seri dengan c/ : c/ 6 Ω c seri dengan c0 : c0 3 Ω c/ paralel dengan c0 : c0 6 Ω c0 seri dengan cß : c(](@^ 6 Ω

Arus yang lewat cß 5ß þ

eN¿NK 0 4 A

Melihat arah arus di c0 maka Ë Ë@ Ë@ 5ßc(](@^ 42 8 V Ë@ 5c/ 5c0, sehingga 65 35 # 5: 5 1:2 5ß 5 5 4 A 5 4/3 A dan 5 8/3 A

b. Ditanyakan : Beda potensial antara titik a dan b Jawab : Ë@ Ë Ë@ 8 V

Page 101: FISIKA TPB

1. Dua buah muatan titik Ì

Hitunglah a. Gaya yang dialami oleh muatan b. Medan listrik di titik (0,0) c. Potensial listrik di titik (0, 2. Sebuah kapasitor keping mempunyai luas penampang 4 cm

Kapasitor tersebut kemudian dihubungkan dengan sumber tegangan 200V a. Tentukan besarnya kapasitansi, muatan, dan besar medan listrik pada kapasitor b. Tanpa melepas sumber tegangan, kapasitor tersebut kemudian disisipi dielektrik yang memiliki

permitivitas relatif (konstanta dielektrik) 30. Tentukan besar kapasitansi, muatan dan besar medan listriknya

c. Tegangan pada kapasitor tersebut listrik akhir yang bekerja

3. Bola isolator dengan muatan

merata pada seluruh volumenya, terletak sepusat di dalam sebuah konduktor berongga yang bermuatan gambar. Tentukan

a. Medan listrik \ pada posisi3c ; dan C 3c dan sketsa grafik b. Potensial listrik Ë

mengasumsikan bahwa 4. Medan magnetik seragam sebes

proton dalam medan magnetik tersebut mempunyai kecepatan a. Tentukan besar gaya dan arah gaya proton (abaikan gaya gravitasi proton) b. Gambarkan sketsa lintasan proton c. Jika selain medan magnetik ditambahkan medan listrik sebesar

medan magnetik, berapa gaya total yang dialami proton dan gambarkan sketsa lintasan protontersebut

5. Dari gambar di bawah, apabila diketahui

dalam keadaan tunak (kapasitor dalam keadaan terisi penuh), tentukan :

a. Nilai arus yang melewati masing b. Ë@ Ë dan muatan di dalam kapasitor c. Disipasi daya pada hambatan 1

SOAL UJIAN TENGAH SEMESTER I

FISIKA DASAR IIA

TAHUN 2003/2004

30: berada di titik (0,6) m sedangkan Ì Gaya yang dialami oleh muatan Ì Medan listrik di titik (0,0)

k (0,-6) m

keping mempunyai luas penampang 4 cm2 dengan jarak antar kepingnya 0,1 mm. Kapasitor tersebut kemudian dihubungkan dengan sumber tegangan 200V

Tentukan besarnya kapasitansi, muatan, dan besar medan listrik pada kapasitorTanpa melepas sumber tegangan, kapasitor tersebut kemudian disisipi dielektrik yang memiliki permitivitas relatif (konstanta dielektrik) 30. Tentukan besar kapasitansi, muatan dan besar

Tegangan pada kapasitor tersebut diputus dan kemudian dielektrik dicabut. Tentukanlah medan listrik akhir yang bekerja

isolator dengan muatan Ì yang terdistribusi secara merata pada seluruh volumenya, terletak sepusat di dalam sebuah konduktor berongga yang bermuatan 2Ì seperti pada

pada posisi-posisi C c ; c C dan sketsa grafik \ terhadap C

pada posisi C c, dengan mengasumsikan bahwa Ë 0 di tempat tak hingga

Medan magnetik seragam sebesar 10tT berarah tegak lurus memasuki bidang tulis. Jika sebuah proton dalam medan magnetik tersebut mempunyai kecepatan 10 m/s ke arah kanan

Tentukan besar gaya dan arah gaya proton (abaikan gaya gravitasi proton)Gambarkan sketsa lintasan proton dalam medan magnetik tersebut. Jelaskan mengapa demikianJika selain medan magnetik ditambahkan medan listrik sebesar 10 N/C ke bawah tegak lurus medan magnetik, berapa gaya total yang dialami proton dan gambarkan sketsa lintasan proton

Dari gambar di bawah, apabila diketahui Ë 9Ë dan kapasitor dalam keadaan tunak (kapasitor dalam keadaan terisi penuh),

Nilai arus yang melewati masing-masing hambatan dan muatan di dalam kapasitor

ambatan 1Ω

10: di titik (-8,0) m.

dengan jarak antar kepingnya 0,1 mm.

Tentukan besarnya kapasitansi, muatan, dan besar medan listrik pada kapasitor tersebut Tanpa melepas sumber tegangan, kapasitor tersebut kemudian disisipi dielektrik yang memiliki permitivitas relatif (konstanta dielektrik) 30. Tentukan besar kapasitansi, muatan dan besar

diputus dan kemudian dielektrik dicabut. Tentukanlah medan

T berarah tegak lurus memasuki bidang tulis. Jika sebuah m/s ke arah kanan

Tentukan besar gaya dan arah gaya proton (abaikan gaya gravitasi proton) dalam medan magnetik tersebut. Jelaskan mengapa demikian

N/C ke bawah tegak lurus medan magnetik, berapa gaya total yang dialami proton dan gambarkan sketsa lintasan proton

Page 102: FISIKA TPB

1. Diketahui : Ì 30: ; a. Ditanyakan : Gaya yang dialami muatan Jawab :

E O

Ù|ZE

E O

Ù|0º

E tßu¡

b. Diketahui : Medan listrik di (0,0)

Jawab : \DE O

Ù|tZE \DE 7,5 u 10

c. Ditanyakan : Potensial di (0,6)Jawab : Perhatikan gambar berikut

2. Diketahui : B 0,1mm a. Ditanyakan : Kapasitansi, muatan, dan medan listrik

Jawab : _& 8

Ì Ë 3, \ Ô

& 2 u b. Ditanyakan : Kapasitansi, muatan, dan medan listrik setelah disisipi dielektrik Jawab : Karena sumber tegangan tidak dilepas maka

_&

Ì Ë 1, \ Ô

& 2 u c. Ditanyakan : Medan listrik setelah tegangan diputus dan dielektrik dicabut Jawab : Muatan pada kapasitor Setelah dielektrik dicabut maka kapasitansi akan sama dengan kapasitansi pada (a)

3,54 u 10t

SOLUSI UJIAN TENGAH SEMESTER I

FISIKA DASAR IIA

TAHUN 2003/2004

CE 6F m ; Ì 10: ; CE 8H m

Gaya yang dialami muatan Ì

ÙÙYEtZEY|¡ CE CE

ÙÙYºsQ|¡ 6F 8H 9 u 10« S/uæUtuæ

»√0YsY»¡ 6uæ¡¡ 6F 8H 0,0162H 0,0216F

Medan listrik di (0,0) ZE|¡ 0 CE

OÙY|ZEY|¡ 0 CE

10/H 1,4 u 10/F N/C

Potensial di (0,6) Perhatikan gambar berikut

Ë 14m

ÌC 14m

ÌC

9 u 10« /uæ 9 u 10« tuæ

13,5 u 10/ V

10tm ; 4cm 4 u 10t m2

Kapasitansi, muatan, dan medan listrik

8,85 u 10t uæûæû 3,54 u 10t F

,54 u 10t u 200 7,08 u 10t« C

100 N/m

Kapasitansi, muatan, dan medan listrik setelah disisipi dielektrikKarena sumber tegangan tidak dilepas maka Ë Ë 200 V

30 u 8,85 u 10t uæûæû 1,06 u 10t« F

,06 u 10t« u 200 2,12 u 10tß C

100 N/m

Medan listrik setelah tegangan diputus dan dielektrik dicabut

Muatan pada kapasitor tidak berubah Ì 2,12 u 10tß C Setelah dielektrik dicabut maka kapasitansi akan sama dengan kapasitansi pada (a)

F

6F 8H

Kapasitansi, muatan, dan medan listrik setelah disisipi dielektrik

Medan listrik setelah tegangan diputus dan dielektrik dicabut

Setelah dielektrik dicabut maka kapasitansi akan sama dengan kapasitansi pada (a)

Page 103: FISIKA TPB

Dari hasil ini akan diperoleh potensial

Medan listrik \ Ô&

3. Diketahui : Bola isolator a. Ditanyakan : Medan listrik Jawab :

Untuk daerah C c

Rapat muatan pada konduktor

4mC Dengan menggunakan hukum gauss akan \DE BE O

\4mC O L Í BË Sehingga akan didapatkan

Untuk daerah c C \ 0 (daerah tersebut berada di dalam konduktor) Untuk daerah C 3c Muatan yang dilingkupi oleh permukaan Gauss adalah 2Ì Ì, yaitu muatan pada isolator ditambah muatan pada konduktor. Dengan menggunakan hukum Gauss akan diperoleh

\DE BE O # \ \

OÙZY

Sketsa grafik

b. Diketahui : Potensial listrik Jawab :

Ëc L/ Ë L

/e

4. Diketahui : 10

Dari hasil ini akan diperoleh potensial Ë Ù¥ ,uæ

/,uæ 6,24 uÔ& 0,u¡

æû 6,24 u 10ß N/m

Bola isolator Ì dan konduktor berongga 2Ì sepusat

Medan listrik \ dan sketsa grafik \

muatan pada konduktor Í ÙÔ Ù /Â e¡ dengan

Dengan menggunakan hukum gauss akan diperoleh

BË ÒO S4 3Â mcU

Sehingga akan didapatkan \ Ò/O C

3c (daerah tersebut berada di dalam konduktor)

Muatan yang dilingkupi oleh permukaan Gauss adalah ab Ì , yaitu muatan pada isolator ditambah muatan pada

konduktor. Dengan menggunakan hukum Gauss akan diperoleh 4mc tÙO

Potensial listrik Ë pada posisi C c

L \/e BC L \e/e BC

O

tÙZY BC L 0e

/e BC O

tÙ/e

OtÙe

10t T ; 10 m/s

u 10/ V

Page 104: FISIKA TPB

a. Ditanyakan : Gaya pada proton Jawab : Perhatikan gambar berikut

Besar gaya Lorentz yang dialami b. Diketahui : Sketsa lintasan proton Jawab :

Proton akan memiliki lintasan lingkaran karena gaya Lorentz yang bekerja selalu memiliki arah

tegak lurus kecepatan linier (memberikan c. Ditanyakan : Gaya total dan sketsa proton jika terdapat medan listrik Jawab :

5. Diketahui : Ë 9 V a. Ditanyakan : Arus yang melewati masing

hambatan Jawab : Perhatikan gambar disamping. Karena kapasitor telah terisi penuh maka tidak ada arus

yang melewati kapasitor sehingga arus yang melewati hambatan c sama dengan nol (

¶ ¶/ Dari loop 2 akan diperoleh

Gaya pada proton

Perhatikan gambar berikut

Besar gaya Lorentz yang dialami oleh proton adalah 1,6 u 10t«1010t 1,6 u 10t N

Sketsa lintasan proton

Proton akan memiliki lintasan lingkaran karena gaya Lorentz yang bekerja selalu memiliki arah tegak lurus kecepatan linier (memberikan gaya sentripetal)

Gaya total dan sketsa proton jika terdapat medan listrik

Besar gaya listrik þ \ 1,6 u 10t«10 1,6 u 10 Karena besar gaya magnetik sama dengan besar gaya listrik maka benda akan bergerak lurus dengan kecepatan konstan sebesar

Arus yang melewati masing-masing

disamping. Karena kapasitor telah terisi penuh maka tidak ada arus yang melewati kapasitor sehingga arus yang melewati

sama dengan nol (¶ 0)

Dari loop 2 akan diperoleh

Proton akan memiliki lintasan lingkaran karena gaya Lorentz yang bekerja selalu memiliki arah

Gaya total dan sketsa proton jika terdapat medan listrik

10t N

Karena besar gaya magnetik sama dengan besar gaya listrik an kecepatan konstan sebesar

Page 105: FISIKA TPB

∑ ∑ 5c Ë 3 ¶c ¶/c/ 9 3 4¶ # ¶ ¶/ 1,5 A Sehingga didapatkan arus yang melewati c dan c/ adalah 1,5 A b. Ditanyakan : (Ë@ Ë) dan muatan di dalam kapasitor Jawab : Dari loop 1 akan diperoleh ∑ ∑ 5c 15 Ë@ 3 ¶c Ë@ 15 3 1,5 10,5 V Muatan dalam kapasitor Ì Ë@ 5 u 10t010,5 5,25 u 10t C c. Ditanyakan : Disipasi daya pada hambatan 1 Jawab : j ¶c 1,51 2,25 W

Page 106: FISIKA TPB

1. Muatan titik A (106) dan B ( a. Besar dan gaya arah gaya Coulomb pada muatan b. Kerja yang diperlukan untuk memindahkan muatan

(5,15) cm 2. Muatan listrik Ì tersebar secara merata pada sebuah batang yang memiliki panjang a. Kuat medan listrik total yang disebabkan oleh batang logam pada titik P seperti terlihat pada

gambar b. Potensial listrik di titik tersebut di atas dengan acuan potensial d

3. Sebuah kapasitor silinder memiliki diameter dalam dielektrik dengan tetapan dielektrik relatif sebesar

tersebut adalah \DEC

a. Beda potensial antara silinder luar dan silinder dalam b. Kapasitansinya

4. Muatan total Ì 10: terdistribusi secara uniform pada sebuah bola isolator berjari-jari c 1 cm dengan tetapan dielektrik

a. Hitunglah kuat medan listrik di dalam bola b. Hitunglah medan listrik di luar bola (medium udara) c. Gambarkan kurva kuat medan terhadap 5. c 2 ; c/ 3 ; c

(muatan pada kapasitor telah penuh), arus yang mengalir pada a. Besarnya c b. Beda potensial titik a dan b c. Muatan pada kapasitor

SOAL UJIAN TENGAH SEMESTER I

FISIKA DASAR IIA

TAHUN 2005/2006

) dan B (106) terletak masing-masing di (0,0) dan Besar dan gaya arah gaya Coulomb pada muatan 56 di titik R (5,15) cm tersebutKerja yang diperlukan untuk memindahkan muatan 56 dari titik tak berhingga ke titik R

tersebar secara merata pada sebuah batang yang memiliki panjang Kuat medan listrik total yang disebabkan oleh batang logam pada titik P seperti terlihat pada

Potensial listrik di titik tersebut di atas dengan acuan potensial di titik tak berhingga adalah nol

Sebuah kapasitor silinder memiliki diameter dalam c& dan diameter luar dielektrik dengan tetapan dielektrik relatif sebesar Z. Jika kuat medan listrik di dalam dielektrik ôO

Z CE dengan λ menyatakan rapat muatan per satuan panjang, hitunglah

Beda potensial antara silinder luar dan silinder dalam

terdistribusi secara uniform pada sebuah bola cm dengan tetapan dielektrik κ=10

Hitunglah kuat medan listrik di dalam bola Hitunglah medan listrik di luar bola (medium udara) Gambarkan kurva kuat medan terhadap C

1 ; c 2 ; 6Ë ; dan 50:. Jika dalam keadaan stasioner (muatan pada kapasitor telah penuh), arus yang mengalir pada c adalah 0,75 A. Tentukan :

Beda potensial titik a dan b

masing di (0,0) dan (0,10) cm. Hitunglah di titik R (5,15) cm tersebut

dari titik tak berhingga ke titik R

tersebar secara merata pada sebuah batang yang memiliki panjang É. Hitunglah Kuat medan listrik total yang disebabkan oleh batang logam pada titik P seperti terlihat pada

i titik tak berhingga adalah nol

dan diameter luar c^ diisi oleh bahan

. Jika kuat medan listrik di dalam dielektrik

menyatakan rapat muatan per satuan panjang, hitunglah

. Jika dalam keadaan stasioner adalah 0,75 A. Tentukan :

Page 107: FISIKA TPB

6. Dua buah ion klorin dengan massa 35 amu dan 37 amu bergerak ke dalam medan magnet sebesar

0,50T dengan kecepatan 2, a. Jari-jari atom klorin dengan massa 35 amu b. Jari-jari atom klorin dengan massa 37

Dua buah ion klorin dengan massa 35 amu dan 37 amu bergerak ke dalam medan magnet sebesar ,0 u 10 m/s (1 amu 1,67 u 10tßkg). Hitung

jari atom klorin dengan massa 35 amu jari atom klorin dengan massa 37 amu

Dua buah ion klorin dengan massa 35 amu dan 37 amu bergerak ke dalam medan magnet sebesar ). Hitung

Page 108: FISIKA TPB

SOLUSI UJIAN TENGAH SEMESTER I

FISIKA DASAR IIA

TAHUN 2005/2006

1. Diketahui : Ì_ 106 ; ÌW 106 ; CE_ 0 ; CEW 10F cm a. Ditanyakan : Gaya Coulomb pada muatan +5nC di titik R (5,15) cm Jawab : Gaya Coulomb dalam bentuk vektor dapat dituliskan sebagai

E O

ÙÙY|ZEtZEY|¡ CE CE

Gaya Coulomb pada muatan yang berada di titik R Ee Ee_ EeW

Ee O

ÙéÙ|ZEétZE|¡ CEe CE_ O

ÙéÙ[|ZEétZE[|¡ CEe CEW

Ee ÙéO ñ Ù|ZEétZE|¡ CEe CE_ Ù[|ZEétZE[|¡ CEe CEWò

Ee 450 u 10t ñ Qº»√YY»¡ tQtº

»√YY»¡ò

c. Ditanyakan : Kerja yang diperlukan untuk memindahkan muatan +5nC dari titik tak berhingga

ke titik R (5,15) cm Jawab :

d ÌeËe ÙéO + Ù|ZEétZE| Ù[|ZEétZE[|- 450 u 10tß ñ

»√YY»¡ »√YY»¡ò

2. Diketahui : Muatan Ì tersebar merata pada batang É a. Ditanyakan : Kuat medan listrik di P Jawab : Perhatikan gambar berikut

Medan listrik di titik P oleh muatan yang terletak pada elemen kawat BM adalah B\. Karena komponen B\2 akan saling menghilangkan maka

B\ B\' B\ sin 7 &ZY sin 7 &2

ZY sin 7

Dari gambar di atas dapat disimpulkan bahwa M É cot 7 sehingga didapatkan

BM ^9abYh B7 dan C M É ^Y

9abYh

B\ &2ZY sin 7 sin 7 +9abYh

^Y - + ^9abYh B7- sin 7 B7

jadi

\ L B\ L sin 7 B7hYh cos 7 cos 7 ^ √5

b. Ditanyakan : Potensial listrik di P

Jawab : Ë L \ BÉ √5 L &^

^ √5 ln É

Page 109: FISIKA TPB

3. Diketahui : \DEC O

a. Ditanyakan : Beda potensial antara silinder luar dan silinder dalam

Jawab : Ë L \ BC b. Ditanyakan : Kapasitansi

Jawab : ÙÔ !

Y"#O#

4. Diketahui : Ì 10: ; c a. Ditanyakan : Kuat medan listrik di dalam bola Jawab : Dengan menggunakan hukum Gauss kita akan

dapatkan

L \DE BE $O # \4 Untuk C c, nilai ab Karena homogen, ab ÌC Ù

û¡e¡

/ mC/

Sehingga

\4mC $O + Ùe¡ C/

\ $O

Ùe¡ C

b. Ditanyakan : Medan listrik di luar bola Jawab : Dengan menggunakan hukum Gauss akan

didapatkan

L \DE BE $O \ Ù

OZY

c. Ditanyakan : Kurva kuat medan terhadap Jawab :

5. Diketahui : c 2 Ω ; c a. Ditanyakan : Besar

ôZ CE

Beda potensial antara silinder luar dan silinder dalam

BC L Oô

e¢e%&ZZ

Oô ln e%e¢

Kapasitansi ^#ô öªïé%é¢ð

c 1< ; 10

uat medan listrik di dalam bola Dengan menggunakan hukum Gauss kita akan

4mC ÙZ$O

ab tidak sama dengan Ì total ab akan memenuhi Ù

e¡ C/

+ /-

edan listrik di luar bola Dengan menggunakan hukum Gauss akan

O # \4mC ÙO

urva kuat medan terhadap C

c/ 3 Ω ; c 1 Ω ; c 2 Ω ; 6V ; 50Besar c

Jawab : Perhatikan gambar berikut Karena berada dalam keadaan stasioner, maka kapasitor tidak dilewati arus. Titik ab memiliki hambatan pengganti

c@ eYe¡eûeeYe¡eûe /

/ 1,875

50:

berada dalam keadaan stasioner, maka kapasitor tidak dilewati arus. Titik ab memiliki hambatan pengganti

875 Ω

Page 110: FISIKA TPB

Tinjau titik ac Ë@ ¶c ¶c@ c

¹ c@ 0,ß 1,875 6,13 Ω

b. Ditanyakan : Beda potensial di titik a dan b Jawab : Ë@ ¶c@ 0,751,875 1,41V c. Ditanyakan : Muatan pada kapasitor Jawab : Ì Ë 50 u 10t01,41 5,05 u 10t C

Page 111: FISIKA TPB

SOAL UJIAN TENGAH SEMESTER I

FISIKA DASAR IIA

TAHUN 2006/2007

1. Mula-mula tiga buah muatan disusun pada gambar di

samping. Besar muatan Ì adalah 2: yang jenisnya (tandanya) belum diketahui. Muatan Ì tidak diketahui besar dan jenisnya. Sedangkan jenis muatan Ì/ adalah positif dan besarnya 4:. Resultan gaya E yang bekerja pada muatan Ì/ ke arah sumbu-x negatif

a. Tentukanlah Ì dan Ì (besar dan tanda/jenis) b. Tentukanlah besarnya gaya E c. Tentukanlah potensial di titik A d. Jika posisi Ì dan Ì tetap seperti pada gambar

dan Ì/ bebas bergerak, apakah gaya total yang bekerja pada muatan Ì/ selalu tetap terhadap waktu? Jelaskan dengan ringkas dan singkat

2. Tinjau sebuah bola pejal isolator berjari-jari c yang mempunyai muatan Ì tersebar secara merata a. Dengan menggunakan hukum Gauss tentukanlah medan listrik sebagai fungsi dari jarak dari

pusat bola isolator di dalam dan di luar bola isolator b. Tentukanlah potensial listrik sebagai fungsi dari jarak dari pusat bola isolator di dalam dan di

luar bola isolator c. Jika bola isolator ini ditempatkan dalam daerah dengan kuat medan listrik \DE 5H N/C,

hitunglah fluks total yang melewati seluruh permukaan bola isolator tersebut 3. Diberikan rangkaian listrik seperti pada gambar. 2000:. Jika pada 0 kapasitor dalam

keadaan kosong, maka a. Hitunglah arus 5 sesaat setelah saklar s ditutup b. Tentukanlah Ë@ pada saat kapasitor dalam keadaan tunak (terisi penuh muatan) c. Tentukanlah energi yang tersimpan dalam kapasitor saat tunak 4. Sebuah kawat berupa bujur sangkar terletak dalam bidang M seperti pada gambar, dengan sumbu-z

positif keluar bidang kertas (abaikan pengaruh gravitasi dan medan magnet yang ditimbulkan kawat berarus 5

a. Tentukanlah gaya Lorentz yang bekerja pada kawat apabila medan magnet DE searah dengan sumbu-z positif

b. Jika medan magnetnya diganti menjadi DE F, hitunglah gaya Lorentz pada kawat tersebut c. Bagaimanakah gerak kawat untuk soal (b), jelaskan dengan ringkas dan singkat

Page 112: FISIKA TPB

5. Diberikan sistem spectrometer massa seperti pada gambar di bawah a. Tentukan besar dan arah dari gaya-gaya yang bekerja pada elektron di titik K dan M b. Turunkan persamaan kecepatan elektron di titik L c. Tentukan jari-jari lintasan elektron, dimanakah posisi detector harus ditempatkan? Gambarkan

jawaban Anda

Page 113: FISIKA TPB

SOLUSI UJIAN TENGAH SEMESTER I

FISIKA DASAR IIA

TAHUN 2006/2007

1. Diketahui : Ì 2 : ; Ì/ 4 : a. Ditanyakan : Tentukan Ì dan Ì Jawab : Susunan gaya yang memungkinkan agar

didapatkan hasil seperti pada gambar adalah sebagai berikut

Agar mendapatkan susunan seperti gambar, maka jenis muatan Ì haruslah positif dan jenis muatan Ì haruslah negatif. Selain itu, gaya yang diakibatkan muatan Ì dan Ì harus sama agar didapatkan resultan total hanya dalam arah sumbu-x. Oleh karena itu, besar muatan Ì harus sama dengan muatan Ì (jarak antara Ì dan Ì/ sama dengan jarak Ì dan Ì/). Maka dapat disimpulkan bahwa jenis muatan Ì adalah negatif dan jenis muatan Ì positif serta |Ì| |Ì|

b. Ditanyakan : Besar gaya E Jawab : Gaya total yang bekerja pada Ì/ diakibatkan oleh Ì dan Ì E(](@^ E/ E/ SE/ cos 7 H E/ sin 7 FU E/ cos 7 H E/ sin 7 F

E/ E/ ÙÙYZY

Gaya total dalam arah sumbu-y adalah nol sehingga

E 2 ÙÙYZY cos 7 H , dari gambar didapatkan cos 7 /

c. Ditanyakan : Potensial di titik A Jawab : Potensial listrik merupakan besaran skalar Ë_ Ë_ Ë_ Ë_/

Ë_ ÙZ ÙY

ZY Ù¡Z¡

Ì Ì , C_ C_ , sehingga akan didapatkan

Ë_ Ù¡Z¡

d. Ditanyakan : Apakah gaya total konstan

Jawab : Gaya total yang bekerja pada muatan Ì/ berubah terhadap waktu karena gaya yang dialami Ì/ bergantung pada jarak. Karena Ì/ bebas bergerak, maka posisinya akan berubah terhadap waktu (akibat gaya yang bekerja padanya) sehingga jarak Ì/ terhadap muatan Ì dan Ì akan berubah

2. Diketahui : bola pejal isolator bermuatan a. Ditanyakan : Medan listrik sebagai fungsi jarak Untuk C c L \DE BE O

Nilai muatan yang dilingkupi oleh permukaan Gauss untuk C c selalu sama dengan Ì

Page 114: FISIKA TPB

\4mC ÙO # \

Untuk C c L DE BE O

\4mC ÙZO

Untuk C c besar

permukaan Gauss yang diambil Karena muatan tersebar merata dapat dituliskan

ÌC L Í BË ÍË ÌC Ù

e¡ C/

Maka didapatkan medan listrik

\ O

Ùe¡ C

b. Ditanyakan : Potensial listrik Jawab : Untuk C c Potensial di titik yang berjarak

dengan titik lain di posisi tak hingga (potensial suatu titik di tak hingga

ËC LZ ËC + Karena bentuk medan listrik di dalam dan di luar bola berbeda, maka penurunan fungsi

potensial harus dilakukan secara dua tahap yaitu mencari beda potensial antara titik tak hingga dengan potensial di kulit luar bola berjariantara kulit berjari-jari di titik C adalah

Ë L \e Ë L +

e

Ë O ïÙ

e Ë

O +/

Ùe

Ë sO

Ùe +3

\ O

ÙZY

besar yang dilingkupi oleh permukaan Gauss bergantung pada besar jaripermukaan Gauss yang diambil

Karena muatan tersebar merata dapat dituliskan

ÍË ñ Ùû¡e¡ò

/ mC/

Maka didapatkan medan listrik

otensial listrik sebagai fungsi jarak

Potensial di titik yang berjarak C dari pusat bola adalah beda potensial antara titik tersebut dengan titik lain di posisi tak hingga (potensial suatu titik di tak hingga sama dengan nol)

\ BC L + O

ÙeY-Z BC

+ O

ÙeY-

Z

Karena bentuk medan listrik di dalam dan di luar bola berbeda, maka penurunan fungsi potensial harus dilakukan secara dua tahap yaitu mencari beda potensial antara titik tak hingga dengan potensial di kulit luar bola berjari-jari c kemudian dilanjutkan de

jari c tersebut terhadap potensial di titik C di dalam bola. Sehingga potensial

BC L \Ze BC

+ O

ÙZ- BC L +

e¡ C-Ze BC

ï

Ùe¡ C cð

+ Ùe

Ù

e¡ C-

+3 ZYeY-

yang dilingkupi oleh permukaan Gauss bergantung pada besar jari-jari

dari pusat bola adalah beda potensial antara titik tersebut sama dengan nol)

Karena bentuk medan listrik di dalam dan di luar bola berbeda, maka penurunan fungsi potensial harus dilakukan secara dua tahap yaitu mencari beda potensial antara titik tak hingga

kemudian dilanjutkan dengan beda potensial di dalam bola. Sehingga potensial

Page 115: FISIKA TPB

c. Ditanyakan : Fluks total Jawab : & L \DE BE

& LS\DE9kZ &@^@ \DE9kZ ^@ZU BE Dari hukum Gauss didapatkan

L \DE9kZ &@^@ BE ÙO karena muatan yang

menyebabkan medan listrik tersebut berada di dalam permukaan tertutup tersebut ab Ì. Sementara untuk L \DE9kZ &@^@ BE 0 karena muatan yang menyebabkan medan luar tersebut berada di luar permukaan tertutup ab 0. Sehingga total fluks adalah

& L \DE9kZ &@^@ BE 0 ÙO 3. Diketahui : 2000:

a. Ditanyakan : Arus sesaat 5 setelah saklar ditutup Jawab : Setelah saklar ditutup arus akan mulai mengalir melalui rangkaian. Karakteristik arus yang mengalir dalam rangkaian pada percabangan kapasitor mencapai maksimum pada awalnya dan terus menurun secara eksponensial (kapasitor mulai terisi muatan) hingga akhirnya menjadi nol setelah kapasitor terisi penuh (keadaan tunak) Untuk kasus soal bagian (a), rangkaian sesaat setelah ditutup dapat disederhanakan sebagai berikut (muatan dalam kapasitor masih kosong) Rangkaian tersebut memiliki hambatan pengganti

eJ 0/

« /0 /

« /

c? 3 kΩ Nilai arus 5 yang mengalir dapat diketahui melalui hubungan

5 ÔeJ «

/ 3 mA

b. Ditanyakan : Ë@ saat kapasitor dalam keadaan tunak

Jawab : Pada keadaan tunak, tidak ada arus yang mengalir pada kapasitor sehingga rangkaian dapat disederhanakan sebagai berikut (kapasitor telah terisi penuh)

5 ÔeK «

« 1 mA

Beda tegangan antara titik a dan titik b dapat diketahui dengan menggunakan loop (Hukum Kirchoff) sebagai berikut Ë@ 56 53 Ë Ë@ Ë 56 53

Page 116: FISIKA TPB

Ë@ Ë 0,0016000 0,0013000 3 V

c. Ditanyakan : Energi yang tersimpan dalam kapasitor saat tunak Jawab :

\ Ë

2000 u 10t09 8,1 u 10t J

4. Diketahui : Kawat berupa bujur sangkar dialiri arus a. Ditanyakan : Gaya Lorentz Jawab : Eà HEÉ u DE Gaya pada kawat 1 Eà HEÈ F u DE T 5È H Gaya pada kawat 2 Eà HEÈH u DE T 5È F Gaya pada kawat 3 Eà/ HEÈ F u DE T 5È H Gaya pada kawat 4 Eà HEÈ H u DE T 5È F Gaya totalnya adalah Eà Eà Eà Eà/ Eà 0 b. Ditanyakan : Gaya Lorentz untuk DE F Jawab : Karena medan magnet bekerja dalam arah sumbu-y maka gaya yang bekerja pada kawat 1

dan 3 yang juga berarah dalam sumbu-y adalah nol karena hasil perkalian cross untuk vektor satuan dalam arah yang sama (arah F) adalah nol

Dengan demikian, Eà Eà 0 Gaya pada kawat 2, DE È/2 F Eà HEÈH u È/2 F 5È/2 T Gaya pada kawat 4, DE È/2 F Eà HEÈ H u È/2 F 5È/2 T Sehingga gaya totalnya adalah (](@^ 5È T c. Ditanyakan : Gerak kawat pada kasus (b) Jawab : Kawat akan bergerak lurus dipercepat dalam arah sumbu-z negatif. Hal ini terlihat dari gaya

total yang bekerja pada kawat berarah ke sumbu-z negatif. Kawat tidak akan berotasi karena ∑ Þ 0 5. Diketahui : Spektrometer massa a. Ditanyakan : Gaya yang bekerja pada elektron di K dan M Jawab : Gaya yang bekerja pada titik K adalah gaya listrik dan gaya Lorentz. Misalkan sumbu-z

positif berarah keluar bidang kertas dan sumbu-x positif searah gerak elektron di daerah I

a9(Za \DE À Ô& F À Ô

& F ]Zkb(' E u DE ÀH u STU À F

Page 117: FISIKA TPB

Gaya yang bekerja pada titik M adalah gaya Lorentz yang berarah radial ke pusat lintasan melingkar dengan besar | ]Zkb('| À

b. Ditanyakan : Persamaan kecepatan elektron di L Jawab : Kecepatan di titik L dapat diketahui dengan meninjau gerakan elektron dalam daerah I. Dari

gambar terlihat bahwa elektron bergerak lurus (tidak terbelokkan dalam arah sumbu-y) sehingga dalam arah sumbu-y berlaku hubungan

∑ ' 0 a9(Za ]Zkb(' 0

À Ô& F À F 0

Sehingga didapatkan Ë/B c. Ditanyakan : Jari-jari elektron Jawab : Ketika mulai memasuki daerah II, elektron akan bergerak melingkar ke arah sumbu-y

negatif karena gaya listrik yang awalnya berperan melawan gaya magnet sudah tidak ada. Sehingga dalam daerah II berlaku hubungan

∑ IYZ

IYZ

Sehingga didapatkan C I

kW

Page 118: FISIKA TPB

1. Sebuah dipole listrik terletak di sepanjang sumbuterpisah sejauh L seperti pada gambar

a. Tentukan medan listrik di titik A ( b. Tentukan potensial di titik A c. Jika dipole tersebut ditempatkan dalam pengaruh

medan listrik luar yang homogeny tentukanlah arah dipole tersebut supaya menghasilkan energi potensial minimum

2. (Fisika IIB) Sebuah bola isolator pejal berjari a. Dengan menggunakan hukum Gauss tentukan medan listrik di b. Gambarkan grafik potensial di

dengan nol) 3. (Fisika IIA) Sebuah bola isolator bermuatan dengan jari

seragam, yaitu Í /C, dengan a. Dengan menggunakan hukum Gauss tentukan medan listrik di b. Gambarkan grafik medan listrik terhadap jarak radial c. Tentukan potensial di C d. Berapa besar kerja yang harus dila

posisi C 4; 4. Diketahui rangkaian tertutup

ini a. Tentukan dan / b. Apabila GGL 2 digantikan dengan sebuah kapasitor

berkapasitansi 5: dan setelah kapasitor mencapai keadaan tunak, tentukanlah muatan yang tersimpan pada kapasitor

SOAL UJIAN TENGAH SEMESTER I

FISIKA DASAR IIA

TAHUN 2007/2008

Sebuah dipole listrik terletak di sepanjang sumbu-y dan seperti pada gambar

Tentukan medan listrik di titik A (M, 0) Tentukan potensial di titik A Jika dipole tersebut ditempatkan dalam pengaruh medan listrik luar yang homogeny \DE 3H 4F N/C, tentukanlah arah dipole tersebut supaya menghasilkan energi potensial minimum

(Fisika IIB) Sebuah bola isolator pejal berjari-jari a mempunyai muatan Ì yang tersebar merataDengan menggunakan hukum Gauss tentukan medan listrik di C ; dan Gambarkan grafik potensial di C ; dan C ; (anggap potensial di posisi tak hingga sama

(Fisika IIA) Sebuah bola isolator bermuatan dengan jari-jari a, memiliki rapat muatan yang tidak , dengan adalah konstanta positif dan C yaitu jarak radial dari pusat bola

Dengan menggunakan hukum Gauss tentukan medan listrik di C ; dan grafik medan listrik terhadap jarak radial C ; dan C ; (anggap potensial di titik tak hingga = 0)

Berapa besar kerja yang harus dilakukan untuk memindahkan muatan Diketahui rangkaian tertutup seperti gambar di samping

Apabila GGL 2 digantikan dengan sebuah kapasitor dan diganti menjadi 8 volt,

setelah kapasitor mencapai keadaan tunak, tentukanlah muatan yang tersimpan pada kapasitor

yang tersebar merata dan C ;

gap potensial di posisi tak hingga sama

jari a, memiliki rapat muatan yang tidak yaitu jarak radial dari pusat bola

dan C ;

(anggap potensial di titik tak hingga = 0) dari posisi C 2; ke

Page 119: FISIKA TPB

SOLUSI UJIAN TENGAH SEMESTER I

FISIKA DASAR IIA

TAHUN 2007/2008

1. Diketahui : Dipol listrik sepanjang sumbu-y terpisah sejauh L a. Ditanyakan : Medan listrik di A Jawab :

Misalkan sudut yang dibentuk oleh \ dan \t terhadap sumbu-x adalah 7, maka \DE_ \DE \DEt \DE_ S\DE cos 7 H \DE sin 7 FU \DEt cos 7 H \DEt sin 7 F Karena jarak masing-masing muatan ke titik A sama, maka

\DE \DEt ZY , dari gambar didapatkan C l+à

- Î

Substitusi menghasilkan

\DE_ 2\DE cos 7 H 2 ZY

(OZ H

\DE_ 2 (Oï+%Y-Y(OYð¡/Y H

b. Ditanyakan : Potensial di titik A Jawab :

Ë_ Ë Ë_t Z

Z 0

c. Ditanyakan : Energi potensial dipole Jawab : DDE ÐE · \DE |ÐE|»\DE» cos 7 dengan 7 merupakan sudut antara arah dipole terhadap medan

listrik. Agar minimum maka cos 7 harus bernilai +1 atau 7 0. Sehingga dapat disimpulkan bahwa dipole harus memiliki arah yang sama dengan medan listrik luar

Ð / H

F 2. Diketahui : Bola isolator pejal bermuatan a. Ditanyakan : Medan listrik di C ; dan C ; Jawab : Medan listrik untuk C ; L \ B O

\4mC ÙZO

Untuk C ; besar yang dilingkupi oleh permukaan Gauss bergantung pada besar jari-jari permukaan Gauss yang diambil

ÌC L Í BË ÍË ñ Ùû¡e¡ò

/ mC/ Ùe¡ C/

Page 120: FISIKA TPB

\4mC ÙZO ÙOe¡ C/

\ O

Ùe¡ C

Untuk C ; L \ B O

\4mC ÙO \

OÙZY

b. Ditanyakan : Grafik potensial Jawab : Untuk menggambar grafik potensial, kita harus terlebih dahulu mengetahui fungsi potensial

untuk C ; dan C ; Untuk C ; ËC L \Z BC L +

eY-Z BC

ËC O

ÙZ

Untuk C ; Ë L \Z BC L

OÙZY

e BC L O

Ùe¡

Ze CBC

Ë O

ÙZ

e¡ + c

C- O Ì +

e ZYe¡-

Sehingga diperoleh grafik potensial sebagai berikut

3. Diketahui : Rangkaian tertutup a. Ditanyakan : dan Jawab :

Pada umumnya kita menggunakan loop (hukum

Kirchoff) untuk mencari arus yang mengalir pada masing-masing kawat. Pada soal ini arus yang mengalir diberitahu yaitu ¶ 1A dan ¶ 2A

Karena disusun secara paralel maka berlaku

hubungan Ë@ Ë Ëk& Ambil loop terbuka dari a-b Ë@ Ë ¶c ¶c Ë@ 15 11 13 11V

Page 121: FISIKA TPB

Untuk menentukan ambil loop terbuka f-c Ë Ë ¶ ¶c ¶ ¶c Ë Ë@ 11 1,2 1,1 11 12 11 14V Untuk menentukan / ambil loop terbuka e-d Ëk Ë& ¶c0 / ¶c/ Ëk& Ë@ 11 22 / 21 / 11 22 21 5V b. Ditanyakan : Muatan yang tersimpan dalam kapasitor Jawab :

GGL 2 diganti dengan kapasitor 5: dan / 8Ë Pada keadaan tunak, tidak ada arus yang mengalir pada

kapasitor sehingga tidak terjadi percabangan pada titik f (hanya ada satu nilai arus dalam rangkaian). Gunakan loop untuk mencari arus yang mengalir pada rangkaian

Gunakan loop untuk mencari arus yang mengalir pada

rangkaian Loop a-b-c-d-e-f-a 0 ¶c0 / ¶c/ ¶c ¶c 0 2¶ 8 ¶ 3¶ ¶ 15 7 7¶ # ¶ 1A Ambil loop dari kiri kapasitor-f-e-d-c-kanan kapasitor Ë¥ ¶c ¶c0 / ¶c/ Ë¥ 02 12 8 11 11V Muatan yang tersimpan pada kapasitor Ì Ë@?@9a(]Z , Ì 5:11Ë 55: 4. Medan magnet pada titik yang berjarak M dari pusat cincin merupakan hasil kontribusi dari seluruh

elemen cincin. Oleh sebab itu kita tinjau sebuah elemen kecil cincin setelah itu diintegralkan Medan akibat elemen kecil B4

B fOa

&9EuZEZY fOa

&9ZY cos 7 H sin 7 F

Dari arah medan magnet terlihat bahwa jika diintegralkan untuk seluruh elemen cincin,

komponen medan magnet dalam arah sumbu-y akan saling menghilangkan. Medan magnet hanya bekerja dalam arah sumbu-x, sehingga didapatkan

fOa

²³¨ hZY L B4 H , sin 7 dan C untuk seluruh elemen adalah sama

fOa

ZY +e

Z- 2mcH cos 7 c/C L B4 4 2mc

fOa

eY2YeY¡/Y H

»DE» fOaeY2YeY¡/Y

Dua buah cincin

Page 122: FISIKA TPB

Arah medan magnet cincin 1 Arah medan magnet cincin 2

Untuk M 0 DE(](@^ DE DE

DE(](@^ fOa

eY2YeY¡/Y H fOa

eY

2&YeY¡/Y H Untuk 0 M B DE(](@^ DE DE

DE(](@^ fOa

eY2YeY¡/Y H fOa

eY

&t2YeY¡/Y H Untuk M B DE(](@^ DE DE

DE(](@^ fOa

eY2YeY¡/Y H fOa

eY

2t&YeY¡/Y H Posisi M agar medan magnet B maksimum, yaitu M 0 (pusat cincin 1) atau M B (pusat cincin 2) 5. Jawab : a. Hukum Ampere BÉ :5 2mC :5 # fOa

Z

b. Medan total di titik A DE_ DE DE

DE_ fOaZ F fOa

ZY S cos 7 F sin 7 TU

Dari soal bisa didapatkan C 0,03 m C √3 4 5 cm

sin 7 cos 7 /

Substitusikan seluruh nilai yang diketahui

Page 123: FISIKA TPB

DE_ uæ,/ F uæ

, + / F

T-

DE_ 1,81 u 10tF 6,4 u 10tT c. Gaya magnet per satuan panjang Gaya Lorentz antar kawat

5É u 5É 5É + fOaZó-

^ fOaY

& uæY 2 u 10t N/m

Page 124: FISIKA TPB

SOAL UJIAN TENGAH SEMESTER I

FISIKA DASAR IIA

TAHUN 2008/2009

1. Pada bidang xy, muatan sebesar +) diletakkan pada posisi (0, a) dan muatan sebesar )√3 diletakkan pada posisi (a, 0).

a) Tentukanlah gaya Coulomb yang dijalani oleh muatan / dengan besar +2) yang diletakkan di posisi (a,a) akibat muatan dan .

b) Tentukanlah posisi muatan yang besarnya -2) sedemikian rupa sehingga muatan / yang diletakkan pada posisi (a, a) tidak mengalami gaya Coulomb.

2. Sebuah bola logam pejal A yang mempunyai jejari a dan bermuatan -) disusun sepusat dengan bola logam berongga B bermuatan total +). Jejari dalam dan luar bola berongga tersebut masing-masing adalah 1,5a dan 2a seperti dtunjukkan dalam gambar. a) Tentukan medan listrik di ruang antara kedua bola

logam tersebut (a<r<1,5a) b) Jika ruang antara kedua boala logam tersebut diisi

bahan dielektrik yang memiliki konstanta dielektrik (permitivitas relatif) k, tentukanlah kapasitansi kapasitor bola tersebut.

3. Potongan kawat yang dialiri arus sebesar I dibentuk menjadi loop

tertutup abcda seperti ditujukan dalam gambar. Arah arus I adalah berlawan arah jarum jam seperti ditunjukkan oleh anak panah. Segmen ab berbentuk busur seperempat lingkaran yang titik pusatnya di O dan berjari-jari R, sedangkan ketiga segmen lainnya berbentuk garis lurus. Panjang bc sama dengan panajng da yaitu 2R/3. a) Tentukan medan magnetik di titik O oleh masing-masing

segmen kawat dengan menggunakan hukum Biot-Savart. Bila perlu gunakan bentuk integral tak tentu berikut ini

L &22Y @Y¡ Y⁄ 2

@Y2Y @Y Y⁄ .

b) Tentukan medan magnetik di titik O oleh seluruh kawat. c) Bila sebuah muatan q dengan besar +) memiliki lintasan sedemikian rupa sehingga saat ini ia

berada di titik O kecepatannya sedang berarah dari titik O menuju titik d dengan besar v, tentukanlah gaya megnetik yang dialami oleh muatan q tersebut di titik O.

4. Sebuah loop koduktor segiempat abcdefa yang berbentuk seperti

ditunnjukkan dalam gambar diletakkan dekat kawat panjang lurus sedimikian sehingga bagian bcde berada di sebuah kana kawat lurus tersebut sedangkan bagian efab berada di sebelah kiri kawat tersebut. Kawat panjang lurus itu dialiri arus yang berubah terhadap waktu dan dinyatakan I(t) = 2 + 1 dengan ke arah seperti gambar. a. Tentukanlah fluks magnet total pada loop konduktor abcdefa.

Page 125: FISIKA TPB

b. Tentukanlah besar dan arah ggl induksi pada loop konduktor abcdefa.

5. Suatu rangkain arus bolak-balik yang terdiri dari hambatan R = 30 dan kapasitor C = 250: disusun seri dengan sumber tegangan seperti ditunjukkan dalam gambar di samping. Tegangan sumber mempunyai bentuk Ë9 50 cos100 volt.

a) Tentukanlah impedansi rangkaian tersebut. b) Tentukan tegangan sesaat pada masing-masing komponen dinyatakan dalam bentuk fungsi

cosinus.

Page 126: FISIKA TPB

SOLUSI UJIAN TENGAH SEMESTER I

FISIKA DASAR IIA

TAHUN 2008/2009

1. a) Gaya Coulomb yang dialami muatan / oleh muatan yaitu / adalah / ÙY@Y H,

sedangkan gaya Coulomb yang dialami muatan / oleh muatan yaitu / adalah

/ ÙY√/@Y F, gaya Coulomb total adalah / ÙY

@Y H ÙY√/@Y F

b) Supaya resultan gaya Coulomb menjadi nol maka gaya Coulomb yang dialami muatan / oleh muatan haruslah / ÙY@Y H ÙY√/

@Y F, maka didapat posisi muatan

adalah +0, ; @√/û -

2. a) Berdasarkan hukum Gauss, maka dapat dihitung medan listrik pada ruang kosong di

antara kedua bola tersebut, yaitu \DE. BE O , \ u 4mC tÙO , \ tÙOZY

b) Beda potensial antara kedua kulit bola adalah Ë_W Ë_ ËW ÙOZ Ù

OZ[ ÙZ[tZ

OZZ[ Ù@t@O@Y Ù

O@, sehingga kapasitansi bola tersebut adalah ÙÔ

4mi2;

3. a) Medan magnet oleh segmen kawat lurus <B adalah fO¹

^2l2YY

û, arahnya dapat

dicari dengan aturan tangan kanan, yaitu masuk ke dalam bidang kertas, dimana M adalah

jarak kawat <B terhadap titik O, yang dapat dicari dengan hukum phytagoras, yaitu e0 √2,

dan É adalah panjang kawat <B, yaitu e/ √2. Jadi didapat f¿¹

é¡√

é√léYìYéY*

f¿¹

é¡√,

medan magnet oleh segmen < dan B; adalah nol, sedangkan oleh segmen ; adalah

f¿¹se , arahnya adalah keluar dari bidang kertas

b) Medan magnet total adalah f¿¹se f¿¹

é¡√

c) (](@^ 4. a) Arus pada loop <BÀ adalah berlawanan arah dengan jarum jam. Arah medan magnet

yang dihasilkan oleh arus pada loop <BÀ adalah keluar dari bidang kertas, medan magnet yang dihasilkan oleh kawat < dan BÀ adalah sama besar dan arahnya, yaitu

f¿¹2, sedangkan yang dihasilkan oleh kawat À dan <B adalah f¿¹

',, karena

medan magnet yang dihasilkan oleh semua segmen kawat adalah sama arahnya, maka

untuk mencari medan magnet total tinggal dijumlahkan saja, yaitu 2 +f¿¹2 f¿¹

'-,

fluks magnet total untuk loop <BÀ adalah Φ L B L 2 +f¿¹2 f¿¹

'- BMB 2 +L f¿¹

2 BMB L f¿¹' BMB- 2 +f¿¹

Ì ln È f¿¹ È ln Ì-.

Arus pada loop ;8À adalah berlawanan arah dengan jarum jam. Arah medan magnet yang dihasilkan oleh arus pada loop ;8À adalah keluar dari bidang kertas, medan magnet yang dihasilkan oleh kawat ; dan 8À adalah sama besar dan arahnya, yaitu

f¿¹2, sedangkan yang dihasilkan oleh kawat À dan ;8 adalah f¿¹

',, karena

medan magnet yang dihasilkan oleh semua segmen kawat adalah sama arahnya, maka

Page 127: FISIKA TPB

untuk mencari medan magnet total tinggal dijumlahkan saja, yaitu 2 +f¿¹2 f¿¹

'-,

fluks magnet total untuk loop ;8À adalah Φ L B L 2 +f¿¹2 f¿¹

'- BMB 2 +L f¿¹

2 BMB L f¿¹' BMB- 2 +f¿¹

j ln È f¿¹ È ln j-. Jadi fluks total pada loop

;<BÀ8; adalah Φ 2 +f¿¹ Ì ln È f¿¹

È ln Ì f¿¹ j ln È f¿¹

È ln j-

b) &,&( 2 +f¿(

Ì ln È f¿( È ln Ì f¿(

j ln È f¿( È ln j-, arahnya adalah

searah dengan jarum jam.

5. a) Impedansi rangkaian - lc Î¥, dimana Î¥ 1 ¥⁄ , yaitu

Î¥ 1 100 u 250 u 10t0Â 40 Ω, didapat - √30 40 50Ω

b) Arus total pada rangkaian adalah ¶ Ë - cos100 Tegangan sesaat pada masing-masing komponen diberikan oleh Ëe ¶c 30 cos100=É, Ë¥ ¶Î¥ 40 cos100 =É

Page 128: FISIKA TPB

UJIAN TENGAH SEMESTER II

FISIKA DASAR II

Page 129: FISIKA TPB

SOAL UJIAN TENGAH SEMESTER II

FISIKA DASAR II

TAHUN 2002/2003

1. Dua buah kawat panjang dan lurus terletak sejajar dengan sumbu-z. Kawat pertama terletak pada

posisi M 0 dan 0 dan dialiri arus sebesar 2A ke arah sumbu-z positif, dan kawat kedua terletak pada posisi M 0 dan 3 serta dialiri arus sebesar 3A ke arah sumbu-z negatif

a. Tentukan arah dan medan magnet yang terjadi di titik (4,0,0) b. Tentukan gaya per satuan panjang yang dialami oleh kawat pertama 2. Suatu gelombang merambat pada sebuah tali, yang memiliki massa per satuan panjang : 0,1

kg/m dan tegangan tali adalah 2,5 N. Osilasi gelombang tersebut tiap saat diamati pada titik M 0 dan memenuhi persamaan 5 cos10m m/4, yang pada 04 sedang bergerak ke arah sumbu-y negatif, dinyatakan dalam meter dan dalam detik

a. Tentukan laju rambat gelombang tersebut b. Tuliskan fungsi gelombang jika gelombang tersebut merambat ke sumbu-x positif c. Tentukan kecepatan getar tali pada saat 0,2 detik di M 0,125 meter 3. Tiga celah identik dengan jarak antar celah B 0,03 mm disinari tegak lurus dengan cahaya yang

panjang gelombangnya 6000 angstrom dan pola interferensinya diamati pada layar yang jaraknya 5 meter dari celah

a. Tentukan beda fasa / yang menghasilkan intensitas maksimum dan minimum dengan cara fasor b. Gambarkan distribusi intensitas ¶ terhadap / pola interferensi tersebut c. Gambarkan distribusi intensitas ¶ terhadap / jika celah mempunyai lebar Ñ 0,01 mm 4. Perhatikan rangkaian RLC yang ditunjukkan gambar. Tegangan sumber yang terukur pada

voltmeter adalah 60V, dan diketahui frekuensi dari sumber adalah 50Hz serta fasa awal adalah m/6. Apabila diketahui c 30Ω, È 100/m mH, dan 200/m µF a. Tuliskan tegangan sumber Ë@& dalam fungsi cosinus b. Tentukan persamaan arus total, ¶ pada rangkaian tersebut dalam fungsi cosinus c. Carilah Ë@ dalam fungsi cosinus 5. Dua buah solenoid ideal yang sama panjang É disusun sepusat. Solenoida tersebut masing-masing

mempunyai penampang yang berbentuk lingkaran berjejari c dan c (c c). Solenoida yang berjejari c dialiri ¶ dan mempunyai jumlah lilitan V sementara solenoid yang berjejari c mempunyai lilitan V dan dihubungkan dengan sebuah hambatan C

a. Tentukan fluks magnetik pada solenoid besar (berjejari c) b. Tentukan induktansi bersama sistem ini c. Jika arus pada solenoid besar dinyatakan dengan ¶ 3 2 Ampere, tentukan besar dan

arah arus induksi pada hambatan C

Page 130: FISIKA TPB

SOLUSI UJIAN TENGAH SEMESTER II

1. Diketahui : ¶ 2 A ; ¶ Ditanyakan : Besar dan arah medan magnet pada titik (4,0,0) Jawab : Gambar pada bidang

Jarak dari kawat 1 ke titik (4,0,0) adalah Jarak dari kawat 2 ke titik (4,0,0) adalah

Kuat medan magnet di (4,0,0) oleh kawat 1 adalah

Kuat medan magnet di (4,0,0) oleh kawat 2 adalah

fO

¹Y@Y

Dengan menguraikan 2 0,72 ' 0,96 Sementara untuk didapatkan 2 0 ' 10tß T Sehingga arah dan besar medan magnet di titik (4,0,0) DE DE2H DE' DE 0,72 u 2. Diketahui : : 0,1 kg/m ; Berosilasi pada titik

a. Ditanyakan : Laju rambat gelombang

Jawab : lf l

b. Ditanyakan : Fungsi gelombang tersebut jika merambat ke Jawab : Bentuk umum fungsi gelombang yang merambat ke sumbu M, cos Karena / maka Fungsi gelombang pada tali tersebut adalah M, 5 cos

SOLUSI UJIAN TENGAH SEMESTER II

FISIKA DASAR II

TAHUN 2002/2003

¶ 3 A

Besar dan arah medan magnet pada titik (4,0,0) Gambar pada bidang M (dengan sumbu-z keluar bidang kertas)

Jarak dari kawat 1 ke titik (4,0,0) adalah ; 4 m Jarak dari kawat 2 ke titik (4,0,0) adalah ; √3 4 5 m

Kuat medan magnet di (4,0,0) oleh kawat 1 adalah fO

¹@ 10tß

Kuat medan magnet di (4,0,0) oleh kawat 2 adalah

1,2 u 10tß T

Dengan menguraikan dalam arah sumbu-x dan sumbu-y akan diperoleh72 u 10tß T 96 u 10tß T didapatkan

T Sehingga arah dan besar medan magnet di titik (4,0,0) E'F 0,72 u 10tß5 1 0,96 u 10tß0

u 10tß5 0,04 u 10tß0 kg/m ; 2,5 N

Berosilasi pada titik M 0 # 5 cos10m m/4 Laju rambat gelombang

l,, 5 m/s

Fungsi gelombang tersebut jika merambat ke sumbu-x positif

Bentuk umum fungsi gelombang yang merambat ke sumbu-x positif adalahcos M maka / 2m

Fungsi gelombang pada tali tersebut adalah cos10m 2mM m/4

z keluar bidang kertas)

T

y akan diperoleh

x positif

x positif adalah

Page 131: FISIKA TPB

c. Ditanyakan : Kecepatan getar tali pada saat Jawab :

(Z &'&( 50 sin

50 50 Jadi kecepatan getar tali pada saat

3. Diketahui : B 0,03 mm ;

a. Ditanyakan : Beda fasa Jawab :

Misalkan persamaan gelombang untuk sinarmasing-masing adalah 4: cos 4: cos 4/:/ cosKarena ketiga celah tersebut identik, maka kita dapat menuliskan cos cos / cosDiagram fasor pada saat

Secara geometris, susunan fasor yang akan menghasilkan

Susunan ini diperoleh bila (Catatan : / adalah beda fase antara dua fasor berurutan) Susunan fasor yang akan menghasilkan

Kecepatan getar tali pada saat 0,25 s dan M 0,12510m 2mM m/4

50 sin10m0,2 2m0,125 m/4 50 sin2m 0 Jadi kecepatan getar tali pada saat 0,24 dan M 0,125 m adalah nol

mm ; 6000 A ; È 5 m ; Ñ 0,01 mm

Beda fasa / yang menghasilkan intensitas maksimum dan minimum

Misalkan persamaan gelombang untuk sinar 4, 4, 4/

cos M cos M cos M/ Karena ketiga celah tersebut identik, maka kita dapat

M M / M 2/ Diagram fasor pada saat 0 dan M 0 adalah

Secara geometris, susunan fasor yang akan menghasilkan e terbesar adalah

Susunan ini diperoleh bila / 0, 2m, 4m, 6m,… adalah beda fase antara dua fasor berurutan)

Susunan fasor yang akan menghasilkan e terkecil (minimum) adalah

125 m

m adalah nol

yang menghasilkan intensitas maksimum dan minimum

terbesar adalah e 3

nimum) adalah

Page 132: FISIKA TPB

e minimum diperoleh dari dua konfigurasi yang masing

b. Ditanyakan : Distribusi intensitas terhadap

Jawab :

Maksimum Utama Minimum

Maksimum Sekunder Gambar distribusi intensitas terhadap

c. Ditanyakan : Distribusi intensitas terhadap jika

Jawab :

4. Diketahui : Sumber yang terukur Frekuensi sumber Fase awal c 30 Ω ; È

a. Ditanyakan : Tegangan sumber sebagai fungsi cosinusJawab : Tegangan maksimum sumber Frekuensi sudut 2m8Jadi tegangan sumber adalah 9 cos

b. Ditanyakan : Persamaan arus total

minimum diperoleh dari dua konfigurasi yang masing-masing / /

Distribusi intensitas terhadap /

/ ¶/¶ u 2m 9

u 2m 2m3

0

u 2m m 1

Gambar distribusi intensitas terhadap / adalah

Distribusi intensitas terhadap jika / Ñ 0,01 mm

Sumber yang terukur 9 k 60V Frekuensi sumber 8 50Hz

m/6 È 100/m mH ; 200/m mF

Tegangan sumber sebagai fungsi cosinus

Tegangan maksimum sumber k√2 60√2 m8 100m rad/s tegangan sumber adalah

60√2 cos100m m/6 V

Persamaan arus total

/ dan /

/

Page 133: FISIKA TPB

Jawab :

Impedansi rangkaian - vc Îà Î¥ lc È |¥ 50 Ω

Arus maksimum ¶ ÔÏ2 0√ 0√

A

Diagram fasor

Dari diagram fasor di atas dapat dilihat bahwa arus mendahului tegangan dengan beda fase

tan (%t(¤e

/ # 53] 0,3m rad

Jadi persamaan arus pada rangkaian adalah 5 5 cos

0 √2 cos +100m

0 0,3m- 0 √2 cos100m ,47m A

c. Ditanyakan : Beda potensial pada titik ac

Jawab :

Impedansi pada titik ac, -@ lc Îà √30 10 10√10 Ω

Tegangan maksimum pada titik ac, Ë@ @9 ¶-@ 0 √2 u 10√10 24√5 V

Diagram fasor

Dari diagram fasor di atas dapat dilihat bahwa tegangan mendahului arus dengan beda fase sebesar @

tan @ (%e

/ /

@ ;6t +/- 0,32] 0,1m rad

Jadi persamaan beda potensial antara titik a dan c adalah

@ @ @9 cos @ 24√5 cos100m 0,57m

5. Diketahui : Dua solenoida ideal sepusat a. Ditanyakan : Fluks magnetik pada solenoida besar Jawab : Tinjau solenoid berjejari c 3W L DE · BE L B mc b. Ditanyakan : Induktansi bersama

Jawab : &¹&( dan V &4[

&(

Dari keduanya kita dapatkan hubungan antara induktansi bersama dengan fluks magnetik yaitu

V &4[&(

&¹ &(⁄

Karena arus yang mengalir berubah terhadap waktu, maka fluks magnetiknya pun berubah terhadap waktu

Vmc &¹&(

&¹ &(⁄ Vmc

c. Ditanyakan : Besar dan arus induksi

Page 134: FISIKA TPB

Jawab : Arus yang mengalir pada hambatan C adalah arus yang berasal dari solenoid c. Maka

dengan mengetahui besar tegangan yang dihasilkan oleh induktor, kita dapat mengetahui arus yang mengalir pada hambatan menggunakan hukum Ohm

V &4[&( Vmc &¹

&( 3Vmc

Dengan menggunakan hukum Ohm kita dapat mengetahui besarnya arus yang mengalir pada hambatan C

¶ Z /eYYZ

Page 135: FISIKA TPB

SOAL UJIAN TENGAH SEMESTER II

FISIKA DASAR II

TAHUN 2003/2004

1. Dua buah kawat lurus panjang tak berhingga dialiri arus ¶ yang arahnya berlawanan menembus

bidang M seperti terlihat pada gambar di samping

a. Tentukan besar dan arah medan magnet di titik O (0,0) b. Tentukan besar dan arah medan magnet di titik P (d/2,d/2) 2. Pada gambar di samping dapat dilihat sebuah rangka kawat

berbentuk U terbalik pada bidang vertikal di medan gravitasi dan medan magnet homogen. Besar medan magnet tersebut 2T yang arahnya ditunjukkan pada gambar (masuk bidang kertas). Batang konduktor ; memiliki hambatan c sebesar 1Ω panjangnya 50 cm, dan massanya 10 gram yang dapat meluncur tanpa gesekan pada kawat berbentuk U terbalik

a. Gambarkan gaya-gaya yang bekerja pada batang ; b. Pada saat batang ; bergerak dengan laju tetap, tentukan

besar dan arah arus induksi yang mengalir pada batang ; (ke kiri atau ke kanan) c. Tentukan besar ggl imbas pada batang ; d. Tentukan laju tetap dari batang ; tersebut 3. Sebuah transformator diperlukan oleh sebuah perusahaan untuk mengubah sumber tegangan bolak-

balik dari 20kV menjadi 200V dengan kuat arus sekunder sebesar 50 A a. Tentukan jumlah lilitan pada kumparan primer jika jumlah lilitan pada kumparan sekunder

adalah 100 b. Tentukan kuat arus primer maksimum c. Jika digunakan sumber tegangan searah, tentukan tegangan dan kuat arus pada sekundernya 4. Sumber tegangan bolak-balik dengan frekuensi 50 Hz dihubungkan pada lampu tabung (TL=tube

lamp) diasumsikan sebagai hambatan murni. Jika tegangan efektif sumber yang digunakan 228,5 V dan sumber hambatan dalam C 166,3, kuat arus efektif yang mengalir adalah 0,6A dan tegangan listrik efektif pada lampu adalah 84V

a. Hitung induksi È b. Tentukan beda fasa antar tegangan dan kuat arus c. Hitung daya rata-rata pada tabung d. Jika pada rangkaian di atas dipasang kapasitor secara seri 4,7 µF, berapakah kuat arus efektif

yang mengalir 5. Gelombang sinusoida merambat dalam tali. Ujung tali (M 0) berosilasi menurut persamaan cos , dengan cos m/4, dalam detik

Page 136: FISIKA TPB

a. Hitung besar frekuensi gelombang b. Jika gelombang merambat ke arah sumbu-x positif, berapa panjang gelombang dan laju

rambatnya c. Tentukan fungsi gelombang

Page 137: FISIKA TPB

SOLUSI UJIAN TENGAH SEMESTER II

1. Diketahui : Dua kawat panjang lurus tak berhingga dialiri arus a. Ditanyakan : Medan magnet di titik O Jawab : Perhatikan gambar

masing-masing kawat, yaitu DE] DE] DE]

fO¹& H fO¹

& F f

Besar medan magnet di titik O adalah

v] ] Dengan arah membentuk sudut

dimana

7 ;6t +WW

b. Ditanyakan : Medan magnet di titik P Jawab : Perhatikan gambar berikut

Jarak dari kawat 1 ke titik P sama dengan jarak dari kawat 2 ke titik P yaitu sebesar 1 2Â B√2. Sehingga besar medan magnet yang dihasilkan oleh masing

¼ ¼ Besar medan magnet di titik P adalah

¼ ¼

SOLUSI UJIAN TENGAH SEMESTER II

FISIKA DASAR II

TAHUN 2003/2004

Dua kawat panjang lurus tak berhingga dialiri arus ¶ dengan arah berlawanan

Medan magnet di titik O Perhatikan gambar

Karena besar arus yang mengalir pada kawat 1 dan 2 adalah sama dan jarak dari kawat 1 ke titik O sama dengan jarak dari kawat 2 ke titik O maka besar yang dihasilkan oleh kedua kawat tersebut pada titik O adalah sama, yaitu sebesar

] ] fO¹&

Arah kedua vektor medan magnet tersebut dapat digambarkan sebagai Vektor medan magnet di titik O adalah resultan dari vector medan-medan magnet yang dihasilkan oleh

masing kawat, yaitu

fO¹& H F

Besar medan magnet di titik O adalah fO¹

& √2

Dengan arah membentuk sudut 7 terhadap sumbu-x positif,

+W¿W¿Y- ;6t1 45]

Medan magnet di titik P (B/2, B/2)

Perhatikan gambar berikut

Jarak dari kawat 1 ke titik P sama dengan jarak dari kawat 2 ke titik P yaitu sebesar

besar medan magnet yang dihasilkan oleh masing-masing kawat adalah

fO¹

Y&√ fO¹&√ fO¹

& √2

Besar medan magnet di titik P adalah

¼ fO¹& √2

dengan arah berlawanan

Karena besar arus yang mengalir pada kawat 1 dan 2 adalah sama dan jarak dari kawat 1 ke titik O sama dengan jarak dari kawat 2 ke titik O maka besar medan magnet yang dihasilkan oleh kedua kawat tersebut pada titik O

Arah kedua vektor medan magnet tersebut dapat

Vektor medan magnet di titik O adalah resultan dari medan magnet yang dihasilkan oleh

Jarak dari kawat 1 ke titik P sama dengan jarak dari kawat 2 ke titik P yaitu sebesar masing kawat adalah

Page 138: FISIKA TPB

Dengan arah membentuk sudut 45] terhadap sumbu-x positif

2. Diketahui : 2 T ; c 1 Ω ; È 0,5 m ; 10g 0,01kg ; 10 m/s2

a. Ditanyakan : Gaya-gaya yang bekerja pada batang ; Jawab : Gaya-gaya yang bekerja pada batang ; adalah gaya gravitasi dan gaya Lorentz. Gaya-gaya pada batang ; diperlihatkan pada gambar di samping. Total gaya yang bekerja pada batang ; adalah nol

b. Ditanyakan : Besar dan arah arus induksi yang mengalir pada batang ; Jawab : Ketika batang ; bergerak dengan laju tetap maka berlaku ∑ 0 # à 0 ¶È 0 # ¶

Sehingga arus yang mengalir pada kawat ; adalah

¶ Wà ,

, 0,1A

Dengan menggunakan aturan tangan kanan dapat disimpulkan bahwa arah yang mengalir pada kawat ; adalah ke kanan

c. Ditanyakan : Besar ggl imbas pada batang ; Jawab : ¶c 0,11 0,1 V

d. Ditanyakan : Laju batang ; Jawab :

Untuk menghitung laju batang ; kita dapat menggunakan hukum Faraday. Perhatikan gambar berikut

&4&( &

&( · &W&( &_

&(

Karena medan magnet konstan maka &W&( 0

sehingga &_&(

Dari gambar dapat disimpulkan bahwa B È BM sehingga

&_&( È &2

&( È

Wà ,

, 0,1 m/s

3. Diketahui : Ë? 20Ë ; Ë9 200Ë ; ¶9 50 A ; V9 100

a. Ditanyakan : Jumlah lilitan pada kumparan primer Jawab :

ÔJÔ§ J

§ # V? ÔJÔ§ V9

b. Ditanyakan : Kuat arus primer maksimum

Jawab :

Page 139: FISIKA TPB

Dengan menganggap transformator ideal maka daya pada kumparan primer sama dengan kumparan sekunder j? j9 Ë?¶? Ë9¶9

¶? Ô§ÔJ ¶9

50 0,5 A

c. Ditanyakan : Tegangan dan kuat arus sekunder

Jawab : Transformator bekerja berdasarkan hukum Faraday yaitu ggl induksi yang muncul karena adanya perubahan fluks. Jika tegangan yang digunakan adalah tegangan searah yang tidak berubah terhadap waktu maka tidak akan terjadi perubahan fluks dan tidak akan dihasilkan ggl induksi pada kumparan sekunder. Jadi jika digunakan tegangan searah maka tegangan dan kuat arus pada kumparan sekunder adalah nol

4. Diketahui : 8 50 Hz ; Ë9 k 228,5 V ; C 166,3 Ω ; ¶k 0,6 A ; Ëe k 84 V a. Ditanyakan : Induksi È Jawab :

Ë9 k ¶9 k- # Ë9 k ¶9 kvC c È

Ë9 k ¶9 k5ïC Ôé áÓÓ¹§ áÓÓ ð 2m8È

ïÔ§ áÓÓ¹§ áÓÓ ð ïC Ôé áÓÓ

¹§ áÓÓ ð 2m8È

2m8È ïÔ§ áÓÓ¹§ áÓÓ ð ïC Ôé áÓÓ

¹§ áÓÓ ð

È 5ïÔ§ áÓÓ

¹§ áÓÓ ð ïC Ôé áÓÓ¹§ áÓÓ ð

È l+s,

,0 - +166,3 s,0- 0,72 H

b. Ditanyakan : Beda fase antara tegangan dan arus Jawab :

tan |àeZ ,ß

/0,/ 0,74

arctan0,74 0,636 C;B 50,4] c. Ditanyakan : Daya rata-rata pada lampu tabung Jawab : j Ëe k¶9 k 840,6 50,4 W

d. Ditanyakan : Kuat arus Jawab : Jika kapasitor 4,7: dipasang seri, maka impedansi total

- lc C +È |¥- l306,3 +2m5072

æ¡-

- 545,7 Ω Arus efektif yang mengalir

¶9 k Ô§ áÓÓ2 s,,ß 0,42 A

Page 140: FISIKA TPB

5. Diketahui : pada M 0 ; cos pada M 0,05 m ; cos m/4 0,1 m ; 20m rad/s

a. Ditanyakan : Frekuensi gelombang Jawab :

Dari hubungan 2m8 akan diperoleh 8 |

10 Hz

b. Ditanyakan : Panjang gelombang dan laju rambat

Jawab : Bentuk umum persamaan gelombang yang merambat ke arah sumbu-x positif adalah cos M. Dengan membandingkan persamaan gelombang pada titik M 0,05 m akan diperoleh 0,05 cos 0,05 cos m/4 Maka dapat disimpulkan bahwa 0,05 m/4 sehingga

, 5m m-1

c. Ditanyakan : Fungsi gelombang

Jawab : Dengan mensubstitusikan nilai-nilai yang telah didapatkan ke bentuk umum, persamaan gelombang akan diperoleh cos M 0,1 cos20m 5mM

Page 141: FISIKA TPB

1. Suatu kawat dilekukkan membentuk

Setengah lingkaran pertama memiliki jaripada bidang 6 sedangkan setengah lingkaran kedua berjariC 5 cm dan terletak pada bidang tersebut berada pada pusat koordinat. Jika kawat diairi arus dalam arah seperti ditunjukkan pada gambar

a. Tentukan besar dan arah medan magnet di pusat koordinat yang dihasilkan oleh masing-

b. Tentukan besar dan arah medan magnet resultan di pusat koordinat

2. Dua rel logam berbentuk U yang bukaannya menghadap ke atas ditempatkan dalam ruang yang memiliki dengan arah masuk tegak lurus bidang gambar. Batang logam panjangnya 2 m dan massanya 0,03 kg muladilepaskan tanpa kecepatan awal. Ketika tercapai keadaan seimbang, tentukan

a. Gaya-gaya yang bekerja pada batang b. Besar dan arah induksi yang mengalir pada logam 3. Ujung tali digetarkan dengan periode 0,4s. Simpangan maksimum yang dibentuk adalah 10 cm

pada saat 0,24, ujung tali (yaitu merambat ke arah x positif dengan laju 4 m/s. Jika dianggap belum ada gelombang terpantulkan tentukan

a. Panjang gelombang dan frekuensi sudut b. Persamaan gelombang dalam fungsi cosinus c. Simpangan pada posisi 5. Cahaya sejajar dari sumbu kromatik dijatuhkan secara tegak lurus pada celah interferensi atau

difraksi a. Jika dijatuhkan pada celah interferensi 4 celah, gambarkan pola in

(sampai pada orde dua). Diketahui jarak antar celah adalah b. Jika dijatuhkan pada suatu celah difraksi dengan lebar

sebagai fungsi sin 7 (sampai orde 3) c. Jika dijatuhkan pada em

gambarkan pola interferensi /difraksi sebagai fungsi Diketahui B 4Ñ

d. Pada pertanyaan (c), maksimum ke berapa dari pola interferensi yang hildifraksi?

6. Jawablah pertanyaan berikut ini a. Gambarkan model atom Rutherford, dan berikan penjelasan kualitatif tentang model atom ini

tidak stabil b. Mengapa model atom tidak stabil? c. Apa usul Bohr untuk menstabilkan model atom d. Turunkan persamaan kuantisasi energi elektron model Bohr untuk atom hidrogen jika diketahui,

massa elektron k 9` 6,6 u 10t/ Js

SOAL UJIAN TENGAH SEMESTER II

FISIKA DASAR II

TAHUN 2004/2005

Suatu kawat dilekukkan membentuk dua buah setengah lingkaran seperti terlihat pada gambar. Setengah lingkaran pertama memiliki jari-jari C 10 cm dan terletak

sedangkan setengah lingkaran kedua berjari-jari cm dan terletak pada bidang M . Titik pusat lengkungan

tersebut berada pada pusat koordinat. Jika kawat diairi arus ¶ 0,1A dalam arah seperti ditunjukkan pada gambar

arah medan magnet di pusat koordinat yang -masing lengkungan kawat

Tentukan besar dan arah medan magnet resultan di pusat koordinat

Dua rel logam berbentuk U yang bukaannya menghadap ke atas ditempatkan dalam ruang yang memiliki medan magnet 0,6 T dengan arah masuk tegak lurus bidang gambar. Batang logam ; yang panjangnya 2 m dan massanya 0,03 kg mula-mula diam, kemudian

an tanpa kecepatan awal. Ketika tercapai keadaan seimbang,

gaya yang bekerja pada batang ; Besar dan arah induksi yang mengalir pada logam

Ujung tali digetarkan dengan periode 0,4s. Simpangan maksimum yang dibentuk adalah 10 cm, ujung tali (yaitu M 0) mengalami simpangan dan gelombang yang terbentuk

merambat ke arah x positif dengan laju 4 m/s. Jika dianggap belum ada gelombang terpantulkan

Panjang gelombang dan frekuensi sudut mbang dalam fungsi cosinus

Simpangan pada posisi M 0,4 pada saat 0,34

Cahaya sejajar dari sumbu kromatik dijatuhkan secara tegak lurus pada celah interferensi atau

Jika dijatuhkan pada celah interferensi 4 celah, gambarkan pola interferensi sebagai fungsi (sampai pada orde dua). Diketahui jarak antar celah adalah B Jika dijatuhkan pada suatu celah difraksi dengan lebar Ñ, gambarkan pola intensitas difraksi

(sampai orde 3) pada empat buah celah difraksi dengan lebar celah Ñ, dan jarak antar celah

gambarkan pola interferensi /difraksi sebagai fungsi sin 7 (dari orde ke 0 sampai orde 5).

Pada pertanyaan (c), maksimum ke berapa dari pola interferensi yang hil

Jawablah pertanyaan berikut ini Gambarkan model atom Rutherford, dan berikan penjelasan kualitatif tentang model atom ini

Mengapa model atom tidak stabil? Apa usul Bohr untuk menstabilkan model atom ini? Turunkan persamaan kuantisasi energi elektron model Bohr untuk atom hidrogen jika diketahui, 9,1 u 10t/ kg dan muatannya À 1,6 u 10t« C serta konstanta Planck

dua buah setengah lingkaran seperti terlihat pada gambar.

Ujung tali digetarkan dengan periode 0,4s. Simpangan maksimum yang dibentuk adalah 10 cm ) mengalami simpangan dan gelombang yang terbentuk

merambat ke arah x positif dengan laju 4 m/s. Jika dianggap belum ada gelombang terpantulkan

Cahaya sejajar dari sumbu kromatik dijatuhkan secara tegak lurus pada celah interferensi atau

terferensi sebagai fungsi sin 7

, gambarkan pola intensitas difraksi

, dan jarak antar celah B, (dari orde ke 0 sampai orde 5).

Pada pertanyaan (c), maksimum ke berapa dari pola interferensi yang hilang akibat adanya

Gambarkan model atom Rutherford, dan berikan penjelasan kualitatif tentang model atom ini

Turunkan persamaan kuantisasi energi elektron model Bohr untuk atom hidrogen jika diketahui, C serta konstanta Planck

Page 142: FISIKA TPB

SOLUSI UJIAN TENGAH SEMESTER II

1. Diketahui : C 10 cm ;

a. Ditanyakan : Besar dan arah medan magnet di titik pusat koordinatJawab : Perhatikan gambar berikut

Besar B, v

b. Ditanyakan : Besar dan arah medan magnet resultanJawab :

Besar v Arah : bidang M dengan membentuk sudut terhadap sumbu

7 arctan +WW

2. Diketahui : É 2 a. Ditanyakan : Gaya yang bekerja pada batang Jawab : Gaya yang bekerja pada kawat adalah gaya Lorentz ke atas

bawah d b. Ditanyakan : Besar dan arah Induksi yang mengalir Jawab : Dalam keadaan setimbang, kedua gayanya sama besar, sehingga arus induksi adalah

¶ ^W ,/

Arah arus : Penjelasan 1 : Karena gerak turun kawat memperkecil luas daerah yang dilingkupi batang ; dan rel maka terjadi penguranga

menghasilkan medan magnet yang melawan perubahan fluks. Karena itu arah medan magnet di dalam loop yang dihasilkan arus induksi harus dari depan ke belakang tegak lurus bidang juga. Untuk menghasilkan medadari ; ke kemudian turun ke sisi kanan rel

SOLUSI UJIAN TENGAH SEMESTER II

FISIKA DASAR II

TAHUN 2004/2005

C 5 cm ; ¶ 0,1 A

Besar dan arah medan magnet di titik pusat koordinat

Perhatikan gambar berikut i. Medan magnet yang dihasilkan oleh berada pada bidang 6

Besar :

fO¹Z uæ,

, 3,14Arah : Sumbu-x negatif ii. Medan magnet total Lekukan kawat pada bidang M

fO¹Z uæ,

, 6,28 uArah : Sumbu-z negatif

v3,14 u 10tß 6,28 u 10tß 7,02sar dan arah medan magnet resultan

v3,14 u 10tß 6,28 u 10tß 7,02 udengan membentuk sudut terhadap sumbu-z negatif

+WWY- arctan +/,uæ

0,suæ- arctan +- 26,6]

2 m ; 0,03 kg ; 0,6 T

Gaya yang bekerja pada batang ;

bekerja pada kawat adalah gaya Lorentz ke atas W

Besar dan arah Induksi yang mengalir

Dalam keadaan setimbang, kedua gayanya sama besar, sehingga arus induksi adalah /,0 0,25 A

: Karena gerak turun kawat memperkecil luas daerah yang dilingkupi batang dan rel maka terjadi pengurangan fluks. Menurut hukum Lorentz, arus induksi harus

menghasilkan medan magnet yang melawan perubahan fluks. Karena itu arah medan magnet di yang dihasilkan arus induksi harus dari depan ke belakang tegak lurus bidang juga.

Untuk menghasilkan medan magnet arah tegak lurus ke belakang, maka kemudian turun ke sisi kanan rel

dan magnet yang dihasilkan oleh lekukan kawat yang

14 u 10tß T

u 10tß T

02 u 10tß T

10tß T z negatif

¶È dan gaya berat ke

Dalam keadaan setimbang, kedua gayanya sama besar, sehingga arus induksi adalah

: Karena gerak turun kawat memperkecil luas daerah yang dilingkupi batang n fluks. Menurut hukum Lorentz, arus induksi harus

menghasilkan medan magnet yang melawan perubahan fluks. Karena itu arah medan magnet di yang dihasilkan arus induksi harus dari depan ke belakang tegak lurus bidang juga.

n magnet arah tegak lurus ke belakang, maka arah arus induksi adalah

Page 143: FISIKA TPB

Penjelasan 2 : Karena gerak turun batang ;, maka

muatan dalam batang ; memiliki kecepatan ke bawah. Karena ada medan magnet ke arah belakang, maka muatan tersebut mengalami gaya Lorentz yang arahnya ke ujung untuk muatan positif dan ujung ; untuk muatan negatif. Karena arah arus sesuai dengan arah aliran muatan positif, maka arah arus adalah dari ; ke kemudian turun di sisi kanan rel

3. Diketahui : 3 0,4 s ; 4 m/s ; 0,1 m

a. Ditanyakan : Panjang gelombang dan frekuensi sudut Jawab : Frekuensi sudut 2m/3 5m rad/s Panjang gelombang 40,4 1,6 m Bilangan gelombang 2m/ 2m/1,6 5m/4

b. Ditanyakan : Persamaan gelombang Jawab :

M, cos M 0,1 cos +5m M -

Pada saat 0,2 s, simpangan titik M 0 adalah maksimum. Jadi 0,1 m. Sehingga

0,1 0,1 cos +5m0,2 0 - # m 0 # m

Sehingga bentuk lengkap persamaan gelombang

M, 0,1 cos +5m M m- m

c. Ditanyakan : Simpangan di titik M 0,4 m saat 0,3 s

Jawab :

0,4 ; 0,3 0,1 cos +5m0,3 0,4 m- 0,1 m

4. a. Hambatan total : c c c 30 50 80 Ω Reaktansi induktif : Îà È 104 u 10t/ 40 Ω Reaktansi kapasitif : Î¥ 1/ 1/1010t0 100 Ω

Impedansi total : -@& vc Îà Î¥

Fase : 3 arctan +ts - arctan + /

- 0,64 rad atau 37]

Potensial maksimum antara titik ; dan Ë ¶-@ 0,02100 2 V Karena Î¥ Îà maka rangkaian bersifat kapasitif. Oleh karena itu arus mendahului tegangan. Kebergantungan tegangan antara ; dan terhadap waktu adalah Ë@ 2 cos m/6 0,64 2 cos 0,12 V

c. j@ ¶c

0,0280 0,016 W

5. Diketahui : Cahaya sejajar monokromatik dijatuhkan tegak lurus pada celah interferensi atau

difraksi

Page 144: FISIKA TPB

a.

b.

c.

d. Maksimum keempat pola interferensi hilang karena tepat berhimpit dengan minimum pertama

pola difraksi 6. Jawab : a. Gambaran model atom i. Atom terdiri dari inti dimana hampir

elektron mengitari inti dalam orbit lingkaran ii. Jari-jari orbit elektron jauh lebih besar dari ukuran inti sehingga hampir semua volume atom

merupakan ruang kosong iii. Elektron tetap berada pada orbitnya karena adanya gaya Coulomb antara elektron dengan

inti b. Masalah kestabilan atom yang dihadapi model atom Rutherford. Berdasarkan teori

elektrodinamika klasik : i. Muatan yang bergerak dengan perc ii. Elektron adalah partikel bermuatan

iii. Karena bergerak melingkar maka elektron memiliki percepatan sentripetal, karena itu elektron selalu memancarkan gelombang EM

iv. Akibat energi yang dimiliki elektron semakin kecil hingga v. Pada akhirnya elektron jatuh ke inti, yang berarti atom tidak stabil

Maksimum keempat pola interferensi hilang karena tepat berhimpit dengan minimum pertama

Gambaran model atom Rutherford Atom terdiri dari inti dimana hampir seluruh massa atom terkonsentrasi dan elektronelektron mengitari inti dalam orbit lingkaran

jari orbit elektron jauh lebih besar dari ukuran inti sehingga hampir semua volume atom uang kosong

Elektron tetap berada pada orbitnya karena adanya gaya Coulomb antara elektron dengan

Masalah kestabilan atom yang dihadapi model atom Rutherford. Berdasarkan teori elektrodinamika klasik :

Muatan yang bergerak dengan percepatan memancarkan gelombang EMElektron adalah partikel bermuatan Karena bergerak melingkar maka elektron memiliki percepatan sentripetal, karena itu elektron selalu memancarkan gelombang EM Akibat energi yang dimiliki elektron semakin kecil hingga jarak elektron ke inti makin kecilPada akhirnya elektron jatuh ke inti, yang berarti atom tidak stabil

Maksimum keempat pola interferensi hilang karena tepat berhimpit dengan minimum pertama

seluruh massa atom terkonsentrasi dan elektron-

jari orbit elektron jauh lebih besar dari ukuran inti sehingga hampir semua volume atom

Elektron tetap berada pada orbitnya karena adanya gaya Coulomb antara elektron dengan

Masalah kestabilan atom yang dihadapi model atom Rutherford. Berdasarkan teori

epatan memancarkan gelombang EM

Karena bergerak melingkar maka elektron memiliki percepatan sentripetal, karena itu

jarak elektron ke inti makin kecil

Page 145: FISIKA TPB

c. Bohr memperkenalkan konsep kuantisasi momentum sudut elektron. Meskipun memiliki

percepatan saat mengitari inti, elektron tidak memancarkan gelombang EM jika momentum sudutnya, È, memenuhi

È 6 `/2m dengan 1,2,3,… d. Gaya sentripetal elektron selama mengitari inti sama dengan gaya Coulomb, sehingga

OkYZY IY

Z

Dari (i) kita dapat menuliskan energi kinetik elektron

i

sOkYZ

O

kYZ

\ i sO

kYZ

OkYZ

\ sO

kYZ

\b kûsOYAY

bY /,0

bY eV

Page 146: FISIKA TPB

SOLUSI UJIAN TENGAH SEMESTER II

1. Dua kawat yang sangat panjang

sumbu-z menembus bidang titik (0, 0, 0) dan di titik (0, 1, 0) m. Kawat arus ¶ 1A dan kawat ¶arah masing-masing seperti terlihat pada gambar di bawah a. Dengan menggunakan hukum amper

medan magnet (arah dan besar)arus pada kawat ¶

b. Tentukan gaya per satuan panjang yang dialami oleh kawat ¶ karena arus pada kawat

2. Di dalam medan magnet bidang gambar, sebuah batang konduktor dibentuk seperti “<” dengan sudut. Sebuah batang konduktor di atas batang konduktor “<” tersebut dan digerakkan mulai dari titik A dengan kecepatan tetap Ketika batang konduktor Édan 80 cm serta membentuk segitiga sama kaki seperti terlihat pada gambar di bawah, tentukanlah

a. Besar fluks magnetik dalam loop segitiga yang terbentuk b. Besar ggl induksi yang dihasilkan c. Arah arus induksi yang terjadi pada loop segitig 3. Suatu rangkaian RLC seri seperti terlihat pada gambar di bawah ini,

terdiri c 40 Ω, 10seri tersebut dihubungkan dengan sumber tegangan bolakË9 5 cosS1000 m

a. Reaktansi kapasitansi total rangkaian

b. Arus 5 yang mengalir pada rangkaian itu c. Tegangan Ë pada masingËe, Ëà, dan Ë¥ 4. Gambar di bawah menunjukkan dua buah grafik yang melukiskan gelombang transversal pada tali.

Gelombang merambat ke arah sumbukedua grafik tersebut

a. Tentukan panjang gelombang, periode gelombang, dan laju ram b. Tuliskan fungsi gelombang secara lengkap c. Tentukan kecepatan osilasi elemen tali pada

SOLUSI UJIAN TENGAH SEMESTER II

FISIKA DASAR II

TAHUN 2005/2006

Dua kawat yang sangat panjang ¶ dan ¶ sejajar z menembus bidang M masing-masing di

titik (0, 0, 0) dan di titik (0, 1, 0) m. Kawat ¶ dialiri dialiri arus ¶ 2A dengan

masing seperti terlihat pada gambar di

Dengan menggunakan hukum ampere tentukan medan magnet (arah dan besar) pada kawat ¶ oleh

Tentukan gaya per satuan panjang yang dialami oleh karena arus pada kawat ¶

50 mT dengan arah keluar , sebuah batang konduktor dibentuk seperti

“<” dengan sudut. Sebuah batang konduktor É disentuhkan di atas batang konduktor “<” tersebut dan digerakkan mulai dari titik A dengan kecepatan tetap 0,1 m/s. É berada pada posisi È 50 cm,

cm serta membentuk segitiga sama kaki seperti terlihat pada gambar di bawah, tentukanlah

Besar fluks magnetik dalam loop segitiga yang terbentuk Besar ggl induksi yang dihasilkan Arah arus induksi yang terjadi pada loop segitiga

seri seperti terlihat pada gambar di bawah ini, 10 :, dan È 70 mH. Jika rangkaian RLC seri tersebut dihubungkan dengan sumber tegangan bolak-balik m/3U V, tentukanlah Î¥, reaktansi induktif Îà, dan impedansi

yang mengalir pada rangkaian itu pada masing-masing komponen c, È, dan , yaitu

ah menunjukkan dua buah grafik yang melukiskan gelombang transversal pada tali. Gelombang merambat ke arah sumbu-x positif. Dengan menggunakan informasi yang ada di dalam

Tentukan panjang gelombang, periode gelombang, dan laju rambat gelombang dalam taliTuliskan fungsi gelombang secara lengkap Tentukan kecepatan osilasi elemen tali pada M 0 dan 0,25 s

ah menunjukkan dua buah grafik yang melukiskan gelombang transversal pada tali. x positif. Dengan menggunakan informasi yang ada di dalam

bat gelombang dalam tali

Page 147: FISIKA TPB

5. Seberkas cahaya merah 625 nm dilewatkan pada sistem kisi 3 celah dengan jarak antar kisi

sama dengan 0,5 nm. Kisi tersebut berjarak 2 meter dari layar. (Pakailah aproksimasi bahwa sudut pandang sangat kecil) a. Tentukan dengan menggunakan diagram fasor, beda yang menghasil interferensi dengan

intensitas maksimum (baik utama maupun sekunder) dan intensitas minimum b. Tentukan jarak antara maksimum utama pertama dengan maksimum utama kedua c. Jika jarak antar celah adalah 3 kali lebar celah, tentukan pola intensitas yang terbentuk di layar

6. Diketahui suatu elektron di dalam atom hidrogen bertransisi dari tingkat energi 6 4 ke tingkat

energi 6 2. Jika tetapan Planck 6,63 u 10t/ Js dan tingkat energi dasar pada atom hidrogen \ 13,6 eV, tentukan a. Energi yang dipancarkan elektron b. Frekuensi dan panjang gelombang foton dipancarkan oleh gelombang elektromagnetik

tersebut

Page 148: FISIKA TPB

SOLUSI UJIAN TENGAH SEMESTER II

FISIKA DASAR II

TAHUN 2005/2006

1. Diketahui : ¶ 1 A ; ¶ 2 A ; B 1 m

a. Ditanyakan : Medan magnet pada kawat ¶ oleh arus ¶ Jawab : Medan magnet pada kawat ¶ oleh arus pada kawat ¶

Dengan menggunakan hukum Ampere akan diperoleh DE BÉE :¶ab L BÉ :¶ 2mB :¶

fO¹Y&

Dengan memasukkan nilai-nilai yang diberikan pada soal, akan diperoleh besar medan magnet pada kawat ¶ oleh arus pada kawat ¶, yaitu

fO¹Y& uæ

4 u 10tß T

Arah medan magnet tersebut adalah masuk bidang kertas (dapat ditentukan dengan menggunakan aturan tangan kanan)

b. Ditanyakan : Gaya per satuan panjang pada kawat ¶ karena arus pada kawat ¶ Jawab : Gaya Lorentz yang bekerja pada kawat adalah à ¶É Jadi gaya per satuan panjang kawat adalah

% ¶ 4 u 10tß N/m dengan arah ke kiri

2. Diketahui : 50 mT 0,05 T ; 0,1 m/s ; È 50 cm 0,5 m ; 0,8 m

a. Ditanyakan : Fluks magnetik Jawab : Dari gambar akan diperoleh

3 È

3 È tan7/2 0,050,54/5 0,01 Tm2 b. Ditanyakan : Besar ggl induksi Jawab : Dari hukum Faraday akan diperoleh

|ab&| &4&( 2È tan7/2 &à

&( 2È tan7/2 0,004 V

c. Ditanyakan : Arah arus induksi Jawab : Dengan menggunakan hukum Lenz dapat kita simpulkan bahwa arah arus yang mengalir

pada batang konduktor É adalah dari atas ke bawah. Arah arus pada loop segitiga dapat digambarkan sebagai

Page 149: FISIKA TPB

3. Diketahui : c 40 Ω ; 10: ; È 70 mH ; Ë9 5 cosS1000 m/3U a. Ditanyakan : Reaktansi kapasitif Î¥, reaktansi induktif Îà, dan impedansi - Jawab :

Î¥ |¥

uæ 100 Ω

Îà È 100070 u 10t/ 70 Ω

- vc Îà Î¥ v40 70 100 50 Ω b. Ditanyakan : Arus yang mengalir pada rangkaian Jawab : Arus maksimum yang mengalir pada rangkaian adalah

¶ ÔÏ2 0,1 A

Untuk menentukan beda fase antara arus dan tegangan, perhatikan diagram fasor berikut

tan (¤t(%e /

;6 t3/4 37] Dari diagram fasor di samping dapat disimpulkan bahwa tegangan tertinggal 37o dari arus, atau dapat dikatakan bahwa pada rangkaian, arus mendahului tegangan dengan beda fase 37o Persamaan arus yang mengalir pada rangkaian adalah 5 0,1 cos1000 m/3 37] A

c. Ditanyakan : Tegangan pada masing-masing komponen

Pada resistor Tegangan maksimum : Ëe @9 ¶c 0,140 4 V Beda fase antara arus dan tegangan sama dengan nol Jadi tegangan pada resistor dapat dituliskan sebagai Ëe 4 cos1000 m/3 37] V Pada kapasitor Tegangan maksimum : Ë¥ @9 ¶Î¥ 0,1100 10 V Beda fase antara arus dan tegangan adalah 90o (arus mendahului tegangan) Jadi tegangan pada lapasitor dapat dituliskan sebagai Ëe 10 cos1000 m/3 37] 90] V

Ëe 10 cos1000 m/3 53] V

Pada induktor Tegangan maksimum : Ëà @9 ¶Îà 0,170 7 V Beda fase antara arus dan tegangan adalah 90o (tegangan mendahului arus) Jadi tegangan pada induktor dapat dituliskan sebagai

Page 150: FISIKA TPB

Ëe 7 cos1000 Ëe 7 cos1000 4. Diketahui : Tali merambat sepanjang sumbu a. Ditanyakan : Panjang gelombang, periode gelombang, laju rambat gelombang Jawab : Panjang gelombang Periode 3 0,2 s

Laju rambat gelombang

b. Ditanyakan : Fungsi gelombang Jawab : Bentuk umum fungsi gelombang Dengan memasukkan nilai

gelombang, maka dapat disimpulkan bahwa fungsi gelombang untuk soal ini adalah 0,01 sin50mM 10m c. Ditanyakan : Kecepatan osilasi pada Jawab :

Kecepatan osilasi &'&(

Kecepatan osilasi ketika

&'&( 0,1m cos

5. Diketahui : 625 nm ;

a. Ditanyakan : Beda fase yang menghasilkan interferensi maksimum, minimum, dan maksimum sekunder

Jawab : Misalkan persamaan gelombang untuk sinar

masing adalah 4 8 cos 4 8 cos 4/ 8 / cos Karena ketiga celah tersebut identik, maka kita dapat menuliskan cos M cos M / cos M Diagram fasor pada saat

Secara geometris, susunan fasor yang akan menghasilkan

1000 m/3 37] 90] V 1000 m/3 127] V

merambat sepanjang sumbu-x positif

Panjang gelombang, periode gelombang, laju rambat gelombang

Panjang gelombang 0,04 m

Laju rambat gelombang ,

, 0,2 m/s

Fungsi gelombang

Bentuk umum fungsi gelombang : sinM Dengan memasukkan nilai-nilai yang diberikan pada gambar ke bentuk umum fungsi gelombang, maka dapat disimpulkan bahwa fungsi gelombang untuk soal ini adalah m

Kecepatan osilasi pada M 0 dan 0,25 s

&'&( 0,1m cos50mM 10m

Kecepatan osilasi ketika M 0 dan 0,25 s

cos2,5m 0

nm ; 6 3 ; B 0,5 mm

Beda fase yang menghasilkan interferensi maksimum, minimum, dan

Misalkan persamaan gelombang untuk sinar 4, 4, 4/ masing-

M M M/ Karena ketiga celah tersebut identik, maka kita dapat menuliskan M M / M 2/ Diagram fasor pada saat 0 dan M 0 adalah

Secara geometris, susunan fasor yang akan menghasilkan e terbesar adalah

Panjang gelombang, periode gelombang, laju rambat gelombang

nilai yang diberikan pada gambar ke bentuk umum fungsi gelombang, maka dapat disimpulkan bahwa fungsi gelombang untuk soal ini adalah

Beda fase yang menghasilkan interferensi maksimum, minimum, dan

terbesar adalah

Page 151: FISIKA TPB

Susunan ini diperoleh bila (Catatan : / adalah beda sudut fase antara dua fasor berurutan) Susunan fasor yang akan menghasilkan

e minimum diperoleh dari dua konfigurasi yang masing

Susunan fasor yang akan menghasilkan maksimum sekunder adalah

Beda fase untuk susunan ini adalah Jadi dapat disimpulkan

Maksimum utama Minimum

Maksimum Sekunder Catatan : 0,1,2,3,…

b. Ditanyakan : Jarak antara maksimum utama pertama dengan maksimum utama keduaJawab :

Dengan menggunakan

/ B sin 7 Sehingga untuk maksimum utama pertama 0 Sedangkan untuk maksimum utama kedua

2m &'à #

Jadi jarak antara maksimum utama pertama dan

à& S0

c. Ditanyakan : Pola intensitas yang terbentuk di layar jika

Susunan ini diperoleh bila / 0, 2m, 4m, 6m,…

adalah beda sudut fase antara dua fasor berurutan) Susunan fasor yang akan menghasilkan e terkecil (minimum) adalah

minimum diperoleh dari dua konfigurasi yang masing-masing / /

Susunan fasor yang akan menghasilkan maksimum sekunder adalah

Beda fase untuk susunan ini adalah / m

/ ¶/¶ u 2m 9

u 2m 2m3

0

u 2m m 1 …

Jarak antara maksimum utama pertama dengan maksimum utama kedua

Dengan menggunakan aproksimasi sudut kecil dapat diperoleh

&'à

Sehingga untuk maksimum utama pertama / 0 akan diperoleh

Sedangkan untuk maksimum utama kedua / 2m akan diperoleh

# 2m &'à # à

& Jadi jarak antara maksimum utama pertama dan kedua adalah

S0uæ*U,uæ¡ 2,5 mm

ola intensitas yang terbentuk di layar jika B 3Ñ

/ dan /

/

Jarak antara maksimum utama pertama dengan maksimum utama kedua

Page 152: FISIKA TPB

6. Diketahui : 6 4 ; 6 a. Ditanyakan : Energi yang dipancarkan elektron Jawab : Transisi dari 6 4 ke 6 Energi yang dipancarkan pada transisi tersebut adalah

∆\ \ \ Tanda negatif menunjukkan pemancaran energi b. Ditanyakan : Frekuensi dan panjang gelombang yang dipancarkan Jawab : Energi yang dipancarkan pada 2,25ÀË 4 Frekuensi dari gelombang yang dipancarkan adalah

∆\ `8 # Panjang gelombangnya adalah

2 ; \ 13,6 eV

Energi yang dipancarkan elektron

6 2 Energi yang dipancarkan pada transisi tersebut adalah

\ t/,0Y +t/,0

Y - 2,25 eV

Tanda negatif menunjukkan pemancaran energi

Frekuensi dan panjang gelombang yang dipancarkan

Energi yang dipancarkan pada transisi tersebut adalah 4,08 u 10t« J Frekuensi dari gelombang yang dipancarkan adalah

8 ∆þA ,suæ*

0,0/uæ¡û 6,15 u 10 Hz

Panjang gelombangnya adalah /uì

0,uû 4,88 u 10tß m

Page 153: FISIKA TPB

SOAL UJIAN TENGAH SEMESTER II

FISIKA DASAR II

TAHUN 2006/2007

1. a. Sebutkan kelemahan model atom Thomson (model atom kismis) b. Sebutkan perbedaan model atom Rutherford dan model atom Bohr c. Dengan deret Balmer pada atom Bohr, hitunglah bilangan kuantum (tingkat energi) awal dari

spectrum cahaya tampak atom hidrogen dengan 496 nm Diketahui konstanta Rydberg, c 1,097 u 10ß m-1 2. Suatu rangkaian RLC seri (lihat gambar) memiliki c 10 Ω, i 200/m H, dan 1000/m :. Rangkaian tersebut dialiri 5 fungsi cosinus yang memiliki frekuensi 8 50 Hz, fasa awal arus m/2 rad, dan arus 5Z9 2 A

a. Tentukan persamaan arus 5 b. Tentukan Îà , Î¥ , dan impedansi total - c. Tentukan fungsi tegangan Ë@, Ë, dan Ë& d. Tentukan beda fasa antara arus dengan tegangan sumber 3. Seutas tali diberi gangguan yang merambat dengan fungsi

sebagai berikut M, cos9m2M 1/2: ( dan M dalam cm, dan dalam detik) a. Tentukan amplitudo, panjang gelombang, dan frekuensi gelombang b. Tentukan simpangan pada M 1 cm, dan 5 detik c. Tentukan jarak antara dua titik terdekat dalam arah rambat yang mempunyai beda fasa m pada

saat tertentu d. Tentukan nilai maksimum laju getar partikel tali 4. Dua buah celah sempit berjarak 0,85 mm disinari cahaya dengan panjang gelombang 600 nm. Pola

interferensi diamati pada layar yang berjarak 2,80 m dari celah (Gunakan pendekatan sudut kecil) a. Tentukan beda fase dari hasil interferensi dua gelombang pada titik di layar yang berjarak 2,50

mm dari pusat pola interferensi b. Tentukan rasio intensitas antara pola interferensi pada titik berjarak seperti soal (a) dari pusat

pola interferensi terhadap intensitas pola interferensi di pusat 5. Kawat lurus panjang dialiri arus yang berubah terhadap waktu 5 2 5 u 10t/ A, dimana dalam detik. Di sampingnya

diletakkan kawat berbentuk loop segiempat yang memiliki hambatan c 2,0 Ω, seperti pada gambar. Diketahui È 20 cm, 10 cm, dan ; 5 cm

a. Tentukan besar dan arah medan magnetik pada titik yang

berjarak C dari kawat lurus panjang b. Berapa fluks magnetik total yang melewati loop kawat? c. Ke mana arah arus induksi dalam loop kawat? Berikan

alasannya

Page 154: FISIKA TPB

SOLUSI UJIAN TENGAH SEMESTER II

FISIKA DASAR II

TAHUN 2006/2007

1. Jawab : a. Kelemahan model atom Thomson (model roti kismis) Tidak dapat menjelaskan mengapa dalam percobaan hamburan partikel alfa oleh foil emas

sebagian kecil partikel dipantulkan dengn sudut yang sangat besar. Selain itu ada sebagian kecil partikel alfa yang dipantulkan dalam arah yang hampir berlawanan dengan sinar datang

b. Perbedaan model atom Rutherford dan model atom Bohr Pada model atom Ruherford, elektron mengelilingi inti dengan orbit yang tidak stasioner

dan spectrum emisi atom bersifat kontinu, sementara pada model atom Bohr elektron mengelilingi inti dengan orbit stasioner dan spectrum emisi atom bersifat diskrit.

c. Bilangan kuantum (tingkat energi) awal dari spektrum cahaya tampak (deret Balmer) atom

hidrogen dengan 496 nm

c +

Y Y-

s0uæ 1,097 u 10ß +

Y-

Sehingga didapatkan 16 atau 4 2. Diketahui : c 10 Ω ; È 200/m H ; 1000/m µF ; 8 50 Hz a. Ditanyakan : Persamaan arus 5 Jawab : 5 5 cos 5 2√2 cos100m m/2 A b. Ditanyakan : Îà , Î¥ , dan impedansi total - Jawab : Îà È 2m8È

Îà 2m50 + u 10t/- 20 Ω

Î¥ |¥

¥

Î¥ +OOO" uæ- 10 Ω

- vc Îà Î¥ v10 20 10 10√2 Ω c. Ditanyakan : Fungsi tegangan Ë@, Ë, dan Ë& Jawab :

Ë@ 5c

+2√2 cos +100m -- 10

20√2 cos +100m -

Untuk mencari tegangan Ë, ingatlah kembali

diagram fasor untuk c, Îà , dan Î¥. Beda fasa antara c dan Îà adalah sebesar m/2, sedangkan beda fasa antara c

Page 155: FISIKA TPB

dengan Î¥ sebesar –m/2. Ë 5Îà

+2√2 cos +100m

-- 20

40√2 cos100m m Ë& 5Î¥

+2√2 cos +100m

-- 10

20√2 cos100m d. Ditanyakan : Beda fase Jawab :

Perhatikan kembali gambar diagram fasor di atas

tan ' (%t(¤e , dengan ' adalah beda fasa

Sehingga ' arctan +(%t(¤e - arctan +t

- arctan 1 45]

3. Diketahui : M, cos ;m +2M -<

a. Ditanyakan : Amplitudo, panjang gelombang, dan frekuensi gelombang Jawab : Lihat panjang gelombang kembali

M, cos ;m +2M -<

Secara umum, persamaan gelombang dituliskan sebagai M, cosM Dari perumusan tersebut terlihat bahwa Ampltudo gelombang, 1 cm Panjang gelombang, 2m/ 2m/2m 1 cm Frekuensi gelombang, 2m8 m # 8 1/2 Hz b. Ditanyakan : Simpangan pada M 1 cm dan 5 detik adalah Jawab :

M 1, 5 cos ;m +21 5 -<

cos2m 5m m/2

cos + m- 0

c. Ditanyakan : Jarak antara dua titik terdekat Jawab : ∆ ∆M

mS2∆MU m # ∆M cm

d. Ditanyakan : Laju getar maksimum Jawab :

&'&( m sin m +2M

- m/s

Laju maksimum terjadi saat nilai sinus sama dengan satu

sin m +2M - 1 # @9 m

4. Diketahui : 600 nm ; B 0,85 mm ; È 2,8 m a. Ditanyakan : Beda fase Jawab :

Page 156: FISIKA TPB

Menghitung beda fase 6 Pada soal ini diketahui jarak sebuah titik

terhadap pusat interferensi. Berdasarkan gambar, beda fase interferensi gelombang pada titik yang berjarak interferensi harus memenuhi

Jika dianggap sudut

Ð/È, sehingga persamaan di atas akan menjadi

6 ?&à S,uæ¡U0u

Atau bila dinyatakan dalam sudut, beda fasenya adalah 1,26 b. Ditanyakan : Rasio intensitas

Jawab : ¹¹O ,0_OY

_OY 5. Diketahui : ¶ 2 a. Ditanyakan : Medan magnet di titik yang berjarak Jawab : Gunakan hukum Ampere L DE BÉE :¶ 2mC : fO¹(

Z fO(uZ

bidang kertas b. Ditanyakan : Fluks magnetik total yang melewati loop kawat Jawab : Tinjau elemen luas yang diarsir setebal

luas tersebut adalah B3 B BÈC B3 ÈBC Fluks total yang melewati loop kawat adalah

3 L B3 L@

@ +fO

+fO

Masukkan nilai È 3 +fO(

0,1 0,035 3 0,4396 c. Ditanyakan : Besar ggl induksi dalam loop kawat

Jawab : &4&(

0,8792 u d. Ditanyakan : Arah arus induksi Jawab : Arah arus induksi harus berlawanan dengan arah arus jarum jam

6 Pada soal ini diketahui jarak sebuah titik Ð 2,5 mm

terhadap pusat interferensi. Berdasarkan gambar, beda fase 6 dari interferensi gelombang pada titik yang berjarak Ð dari pusat interferensi harus memenuhi B sin 7 6

Jika dianggap sudut 7 cukup kecil, maka sin 7 tan 7 , sehingga persamaan di atas akan menjadi

?&à 6

US,suæ¡Uæ*,s 1,26

Atau bila dinyatakan dalam sudut, beda fasenya adalah 1,26

Rasio intensitas Y

1,26

5 u 10t/A; c 2Ω ; È 20cm ; 10cm; ;Medan magnet di titik yang berjarak C

Gunakan hukum Ampere ¶

æ¡ T dengan arah masuk menembus

Fluks magnetik total yang melewati loop kawat Tinjau elemen luas yang diarsir setebal BC. Perubahan fluks yang melewati elemen

B3 DEBE. Perhatikan gambar # B ÈBC (L konstan)

Fluks total yang melewati loop kawat adalah

L È@@ BC

L +fO(uæ¡Z -@ ÈBC

+ O(uæ¡ - È L

Z@

@ BC

+ O(uæ¡ - È ln +@

@ -

20 cm ; 10 cm ; dan ; 5 cm uæ¡

- 0,2 ln +,,, -

1 +fO(uæ¡ - ln 3

0354m u 10tß2 5 u 10t/ 2 5 u 10t Wb

Besar ggl induksi dalam loop kawat

&&( 0,43962 5 u 10t

u 10t V

Arah arus induksi Arah arus induksi harus berlawanan dengan arah arus jarum jam

; 5cm

. Perubahan fluks yang melewati elemen

Arah arus induksi harus berlawanan dengan arah arus jarum jam

Page 157: FISIKA TPB

Menurut hukum Lenz, arah arus listrik yang terjadi menyebabkan fluks magnetik induksi yang melawan perubahan fluks magnetik asal

Menurut hukum Lenz, arah arus listrik yang terjadi menyebabkan fluks magnetik induksi yang fluks magnetik asal

Menurut hukum Lenz, arah arus listrik yang terjadi menyebabkan fluks magnetik induksi yang

Page 158: FISIKA TPB

SOAL UJIAN TENGAH SEMESTER II

FISIKA DASAR II

TAHUN 2007/2008

Tambahan data konstanta: Konstanta Planck, ` 6,63 u 10t/ m2 kg/s; laju rambat cahaya dalam vacuum, < 3 u 10s m/s

1. Gambar di atas menunjukkan dua buah kurva simpangan gelombang pada tali yang merambat ke

arah sumbu-x positif. Tentukanlah : a. Amplitudo, panjang gelombang, dan periode gelombang b. Fungsi gelombang yang merambat pada tali dinyatakan dalam fungsi cos c. Besar kecepatan getar maksimum dan percepatan getar maksimum tali 2. Perhatikan gambar di samping. Bila dimisalkan arus sesaat yang

mengalir pada rangkaian tersebut memiliki bentuk ¶ ¶ sin , berturut-turut dalam satuan ampere, rad/s dan sekon. Tentukanlah

a. Impedansi total rangkaian dan beda fase tegangan sumber terhadap arus

b. Arus maksimum dan tegangan sesaat pada sumber c. Tegangan sesaat pada resistor, kapasitor, dan induktor 3. Diketahui bahwa fungsi kerja (work function, M) atom Na adalah 2,35 eV. Sebuah foton dengan

panjang gelombang 342 nm menumbuk sebuah elektron yang terikat pada atom tersebut a. Berapakah momentum foton dan energi foton b. Hitunglah energi kinetik maksimum elektron setelah terlepas dari atom Na c. Apakah yang terjadi jika foton dengan panjang gelombang yang sama menumbuk elektron pada

atom Fe (dengan fungsi kerja 4,31 eV) 4. Sebuah batang logam, tidak berhambatan dan memiliki

panjang 0,15 m, diletakkan pada rel konduktor tak berhambatan sehingga kedua ujung logam bebas bergerak. Sebuah hambatan c 1 Ω dihubungkan pada kedua ujung rel. Rangkaian tersebut diletakkan dalam medan magnetik 500 mT seperti tampak pada gambar. Jika batang digerakkan horizontal ke kiri dengan laju 2 m/s, tentukanlah

a. Besar dan arah ggl induksi pada batang logam b. Besar dan arah arus induksi yang mengalir hambatan c 5. (Fisika IIB) Pada sebuah percobaan difraksi celah tunggal, digunakan seberkas sinar dengan

panjang gelombang 500 nm dengan jarak antara celah dan layar sejauh 1,2 m. Jika lebar celah adalah 2,5 mm tentukanlah

a. Posisi intensitas minimum pertama b. Lebar daerah terang utama

Page 159: FISIKA TPB

c. Perbandingan intensitas maksimum sekunder terhadap intensitas maksimum utama 6. (Fisika IIA) Sumber cahaya monokromatik (panjang gelombang ) dilewatkan pada tiga buah celah

sempit. Jarak antar dua celah berdekatan B. a. Tentukan beda fase berapa saja interferensi akan maksimum dengan cara fasor b. Tentukan beda fase berapa saja interferensi akan minimum dengan cara fasor c. Gambarkan pola interferensi (intensitas terhadap sin 7) pada layar d. Jika lebar celah Ñ 0,25B, gambarkan pola interferensi-difraksi (intensitas terhadap sin 7)

pada layar

Page 160: FISIKA TPB

SOLUSI UJIAN TENGAH SEMESTER II

FISIKA DASAR II

TAHUN 2007/2008

1. Diketahui : Dua kurva simpangan gelombang ke arah sumbu-x positif a. Ditanyakan : Amplitudo, panjang gelombang, dan periode Jawab : Amplitudo 0,01 m Panjang gelombang 0,02 m Periode 3 0,4 s b. Ditanyakan : Fungsi gelombang dalam cosinus Jawab : Karena gelombang merambat dalam sumbu-x positif maka persamaan umumnya cos M atau cosM Misalkan kita gunakan persamaan yang pertama, substitusikan nilai , , dan 3 yang telah

didapat 0,01 cos5m 100M Untuk mendapatkan , kita gunakan grafik terhadap M atau terhadap (pilih salah

satu). Misal kita gunakan grafik terhadap M. Substitusikan nilai 0 pada 0 dan M 0 sehingga didapatkan

0 0,01 cos0 0 cos Sehingga nilai yang memenuhi adalah m/2 atau 3m/2 m/2 Kita harus tentukan nilai yang sesuai dengan grafik. Untuk itu kita periksa turunan kedua

(gradien) dari grafik terhadap M +='=(-

Catatan : Jika grafik yang digunakan terhadap maka gradiennya ='=(

='=2 1 sin5m 100M

Pada 0 dan M 0 gradien grafik positif

='=2 sin 0 agar sesuai kita gunakan nilai m/2 sehingga persamaan akhirnya

0,01 +5m 100M -

Dengan cara yang sama, jika kita gunakan persamaan yang kedua maka akan didapatkan

hasil akhir

0,01 +100M 5m -

Dengan menggunakan hubungan cosinus dapat dibuktikan bahwa kedua persamaan itu sama c. Ditanyakan : Besar kecepatan dan percepatan maksimum Jawab : Persamaan kecepatan getar

='=( 0,05m sin +5m 100M

-, maka kecepatan maksimum adalah 0,05m

Persamaan percepatan

=Y'=(Y 0,25m sin +5m 100M

-, maka percepatan maksimum adalah 0,25m

2. Diketahui : ¶ ¶ sin

Page 161: FISIKA TPB

a. Ditanyakan : Impedansi total Jawab :

- vc Îà Î¥ v40 10 40 50 Ω Beda fase tegangan terhadap sumber arus Îà Î¥

tan Ëà Ë¥Ëe Îà Î¥c 34

arctan + /- 37] 0,2m

b. Ditanyakan : Arus maksimum Jawab :

¶@2 ÔÏKr2N¿NK 2 A

Karena persamaan yang diberikan adalah persamaan sesaat arus 5, maka seluruh persamaan terhadap fungsi waktu mengacu pada persamaan arus yang diberikan. Sehingga untuk mencari tegangan sumber sesaat adalah

Ë9kZ Ë@2 sin ∆ ∆ adalah beda fase tegangan sumber relatif terhadap arus (karena jadi acuan). Pemilihan

fungsi sinus menyesuaikan dengan persamaan arus yang juga dalam fungsi sinus (bukan cosinus)

Dari grafik fasor, terlihat bahwa tegangan tertinggal (tegangan berada di bawah arus) sebesar 0,2m maka ∆ 0,2m sehingga didapatkan persamaan Ë9kZ 100 sin25 0,2m

c. Ditanyakan : Tegangan sesaat resistor, kapasitor, dan inductor Jawab : Tegangan resistor Ëe ¶@2c sin25 ∆ e Dari grafik fasor terlihat bahwa arus searah dengan tegangan resistor sehingga ∆ e 0 Ëe 80 sin25 Tegangan kapasitor Ë¥ ¶@2Î¥ sin25 ∆ ¥ Dari grafik fasor terlihat bahwa tegangan kapsitor tertinggal sebesar 0,5m Ë¥ 80 sin25 0,5m Tegangan induktor Ëà ¶@2Îà sin25 ∆ à Dari grafik fasor terlihat bahwa tegangan kapsitor mendahului sebesar 0,5m Ë¥ 20 sin25 0,5m 3.Diketahui : fungsi kerja Na 2,35 eV ; 342 nm a. Ditanyakan : Momentum dan energi foton Jawab : Momentum foton

Ð A 0,0/uæ¡û/uæ* 1,94 u 10tß Ns

Energi foton

\ A Ð< 5,82 u 10t« J 3,64 eV

b. Ditanyakan : Energi kinetik maksimum elektron

Page 162: FISIKA TPB

Jawab : Elektron mendapatkan energi 3,64 eV dari foton. Energi tersebut akan digunakan untuk lepas dari atom Na. Sisa energinya merupakan energi kinetik elektron

\](]b 3 \ \ 3,64 2,35 1,29 eV c. Ditanyakan : Yang terjadi jika foton menumbuk elektron atom Fe Jawab : Karena energi yang diberikan foton \ 3,64 ÀË lebih kecil dari work function Fe 3 4,31 ÀË maka elektron tidak tereksitasi tetapi tetap berada dalam atom tersebut 4. Diketahui : É 0,15 m ; c 1 Ω ; 500 mT ; 2 m/s a. Ditanyakan : GGL induksi Jawab :

|| &4&( &W_

&( &_&( &à2

&( È

|| È 500 u 10t/0,152 0,15 V b. Ditanyakan : Arus induksi

Jawab : 5 e ,

0,15 A

Arah induksi harus menghasilkan medan magnet berarah ke luar bidang kertas untuk

melawan pertambahan fluks akibat gerakan batang. Oleh sebab itu, berdasarkan aturan tangan kanan, arus induksi akan memiliki arah berlawanan arah jarum jam (juga untuk GGL induksi)

5. (Fisika IIB) Diketahui : 500 nm ; È 1,2 m ; Ñ 2,5 mm a. Ditanyakan : Posisi intensitas minimum pertama Jawab : Persamaan titik minimum difraksi Ñ sin 7 6 Ñ '

à 6 , minimum pertama 6 1

à Suæ*U,

,uæ¡ 2,4 u 10t m

b. Daerah terang utama dibatasi oleh minimum pertama di kedua sisinya. Maka lebar celah utama

akan sama dengan 2 kali posisi minimum pertama dari pusat ` 2 u 2,4 u 10t 4,8 u 10t m c. Intensitas maksimum sekunder ¶ dibandingkan dengan intensitas maksimum utama ¶

(maksimum utama memiliki intensitas sama dengan intensitas sumber) yaitu

¹¹O 9abY?/

?/Y dengan / Ñ sin 7

5. (Fisika IIA) Diketahui : Cahaya monokromatik dilewatkan pada tiga buah celah sempit a. Ditanyakan : Beda fase untuk interferensi maksimum Jawab : Susunan yang menyebabkan interferensi maksimum (cara fasor)

Page 163: FISIKA TPB

Maksimum primer Maksimum sekunder b. Ditanyakan : Beda fase untuk interferensi minimum Jawab : Susunan yang

Dari susunan di atas terlihat bahwa interferensi minimum terjadi ketika terdapat beda fase sebesar / 2m/3 dan /

c. Ditanyakan : Pola intensitas terhadap Jawab :

Karena lebar celah cukup besar dibandingkan dengan jarak antar celah, maka akan teramati

pula pola difraksi sehingga pola yang terbentuk pada layar merupakan kombinasi antara keduanya sebagai berikut

d. Ditanyakan : Pola interferensi Jawab :

Besar intensitas maksimum menjadi lebih kecil dari pita pusat diakibatkan oleh teramatinya

gejala difraksi. Yang jadi poin paling penting pada grafik di atas adalah minimum pertama dari pola difraksi (pada kasus ini ketika

Hubungan tersebut dapat diturunkan sebagai berikut Persamaan difraksi minimum Ñ sin 7 6 sin 7 /Ñ Substitusi hubungan

sin 7 &

diamati pita terang (dari pola interferensi, silakan bandingkan dengan grafik sebelumnya).

Beda fase / 0 atau kelipatan 2m aksimum sekunder Beda fase / m

Beda fase untuk interferensi minimum Susunan yang menyebabkan interferensi mnimum (cara fasor)

Dari susunan di atas terlihat bahwa interferensi minimum terjadi ketika terdapat beda fase / 4m/3

Pola intensitas terhadap sin 7

cukup besar dibandingkan dengan jarak antar celah, maka akan teramati pula pola difraksi sehingga pola yang terbentuk pada layar merupakan kombinasi antara keduanya

Pola interferensi-difraksi

maksimum menjadi lebih kecil dari pita pusat diakibatkan oleh teramatinya gejala difraksi. Yang jadi poin paling penting pada grafik di atas adalah minimum pertama dari pola difraksi (pada kasus ini ketika sin 7 4 /B

tersebut dapat diturunkan sebagai berikut Persamaan difraksi minimum 6 minimum pertama ketika 6 1

sin 7 ketika minimum pertama terjadiSubstitusi hubungan Ñ 0,25B sehingga didapatkan

terlihat posisi jatuhnya minimum pertama, sehingga pada grafik tidak

diamati pita terang (dari pola interferensi, silakan bandingkan dengan grafik sebelumnya).

menyebabkan interferensi mnimum (cara fasor)

Dari susunan di atas terlihat bahwa interferensi minimum terjadi ketika terdapat beda fase

cukup besar dibandingkan dengan jarak antar celah, maka akan teramati pula pola difraksi sehingga pola yang terbentuk pada layar merupakan kombinasi antara keduanya

maksimum menjadi lebih kecil dari pita pusat diakibatkan oleh teramatinya gejala difraksi. Yang jadi poin paling penting pada grafik di atas adalah posisi jatuhnya B)

ketika minimum pertama terjadi

posisi jatuhnya minimum pertama, sehingga pada grafik tidak

diamati pita terang (dari pola interferensi, silakan bandingkan dengan grafik sebelumnya).

Page 164: FISIKA TPB

SOAL UJIAN TENGAH SEMESTER II

FISIKA DASAR IIA

TAHUN 2008/2009

1. Medan magnetik dari gelombang elektromagnetik (EM) dinyatakan sebagai berikut:DE

70 cos0,1266 3,77 M 10ß H 63; 6 B;6 dalam SI. Tentukan a) Panjang gelombang dan frekuensi dari gelombang EM tersebut.

b) Fungsi gelombang medan listrik \DE c) Intensitas dan arah perambatan energi gelombang EM tersebut

2. Cahaya monokromatik dengan panjang gelombang 400 nm dikenakan pada suatu permukaan logam yang menghasilkan efek fotolistrik dengan energi kinetik elektron maskimum1,1 eV. a) Hitung fungsi kerja logam tersebut. b) Jelaskan apakah efek fotolistrik terjadi jika cahaya yang digunakan mempunyai panjang 600

nm? c) Hitung panjang gelombang de Broglie elektron dengan enenrgi kinetik sebagaimana

disebutkan di atas. 3. a) Sebuah cahaya tak terpolarisasi dengan intensitas ¶ diarahkan ke polarisator, menghasilkan

cahaya dengan intensitas I, yang kemudian dilewatkan ke analisator. Cahaya yang keluar dari analisator ternyata mempunyai intensitas 30% dari intensitas awalnya ¶. Hitunglah intensitas I dan besarnya sudut antara sumbu transmisi( sumbu mudah) polarisator dan analisator. b) Apabila sinar datang dari medium dengan indeks bias n = 1,2 dijatuhkan pada sebuah kristal, ternyata pada sudut 30 sinar pantulannya terpolarisasi sempurna. Hitunglah indeks bias kristal tersebut.

4. Dua belah celah sempit dengan jarak antar celah 9 disinari cahaya koheren dengan panjang gelombang 600nm.Cahay jatuh pada celah secara tegak lurus dan jarak celah ke layar adalah 2m. a) Tentukan jarak anatara dua minimum yang berurutan. b) Andaikan celah mempunyai lebar 3 , tentukanlah orde interferensi maksimum yanng

hilang(pada kasus soal a). c) Buat sketsa intensitas sebagai fungsi jarak dari pusat terang utama pada layar dengan

memperhatikan efek lebar celah. 5. Dua pesawat ruang angkasa A dan B diluncurkan bersamaan dari sebuah tempat peluncuran

dengan kecepatan tetap dan searah, berturut-turut sebesar Ë@ 0,5 < dan Ë 0,8 < (relatif terhadap tempat peluncuran ). Pesawat B membawa sebuah batang bermassa 2 kg dengan panjang 2 m (memanjang searah gerak pesawat). Massa dan panjang tersebut diukur oleh penumpang pesawat B dengan alat ukur yang berada di dalam pesawat tersebut. Tentukan: a) Laju pesawat B menurut pengamat yang berada di pesawat A. b) Panjang batang menurut pengamat yang berada di pesawat A. c) Massa dan energi kinetik batang menurut pengamat yang berada di tempat peluncuran.

Page 165: FISIKA TPB

SOLUSI UJIAN TENGAH SEMESTER II

FISIKA DASAR IIA

TAHUN 2008/2009

1. a) 0.126 2m  , 2m 0.126 49.84

3,77 u 10ß 2m8, 8 3,77 u 10ß 2m 6,003 u 100 `ÀC6

b) \@9 <@9 70<, dengan demikian maka fungsi gelombang medan listrik \DE adalah \DE 70< cos0.1266 3,77 u 10ß F c) Intensitas adalah ¶ þÏKg§WÏKg§

f ßußf «

f , arah perambatan gelombang adalah

\DE u DE F u H T, berarti arah perambatan gelombang adalah ke arah sumbu y positif

2. a) i@9 A d, d A i@9, d 0,00uæ¡ûu/uìuæ 1.1ÀË 4,9695 u

10t«Ä 1.1ÀË 3.1ÀË 1.1ÀË 2ÀË

b) Jika A d, maka terjadi efek fotolistrik, jadi tinggal kita hitung saja

A , A

0,00uæ¡ûu/uì0uæ 3,313 u 10t«Ä 2,07ÀË, karena

A d, maka terjadi efek

fotolistrik

c)i@9 Aá, 1,1 ÀË Aá, 1,7622 u 10t« 0,00uæ¡ûu/uìá ,

k 0,00uæ¡ûu/uì,ß0uæ* 1.13:

3. a) ¶ 0,3 ¶, ¶ ¶cos3, 0,3 cos3, cos3 0.548, 3 56,79 b) tan 7? bÕ

bK, tan 30 bÕ,, 6 0,693

4. a) B sin 7 6 , B 2à 1 , 9 u 10t0 2

6 u 10tß, M u0uæ«uæ 0.13

b) Minimum difraksi ditandai oleh bilangan bulat & 1, 2,… sedangkan maksimum interferensi ditandai dengan bilangan bulat a 0, 1, 2,…, persamaan untuk interferensi adalah B sin 7 a , dan persamaan untuk difraksi adalah d sin 7 , akhirnya dengan perbandingan akan didapat bahwa maksimum interferensi akan berimpit dengan minimum difraksi pada a 3, 6, 9 c)

Page 166: FISIKA TPB

5. a) It

tI YÂ ,/t, 0,5<

b) É Él1 IYY 2v1 0,25 1,732

c) O5têY

ëY 2,31 , i Ê 1 < 0,156 u 2 u < 0,312<

Page 167: FISIKA TPB

UJIAN AKHIR SEMESTER II

FISIKA DASAR II

(Untuk Mahasiswa Dengan nilai di bawah C

setelah UTS 1 dan UTS 2)

Page 168: FISIKA TPB

SOAL UJIAN AKHIR SEMESTER II

FISIKA DASAR II

TAHUN 2003/2004

1. Tiga buah partikel bermuatan sama besar yaitu masing-

masing 15: terletak pada titik-titik seperti pada gambar di samping ini :

a. Tentukan potensial listrik di titik P b. Hitung kerja yang harus dilakukan untuk memindahkan

muatan Ì 20 : dari tempat tak hingga ke titik P c. Hitung energi potensial listrik sistem empat partikel

bermuatan tersebut 2. Diketahui suatu rangkaian arus searah dilukiskan oleh gambar di bawah ini. Dimana 20 V ; 20 V ; c 6 Ω ; c 2 Ω ; c/ 4 Ω a. Tentukan arus yang mengalir pada c, c, c/ b. Tentukan daya disipasi pada masing-masing resistor c, c, c/ c. Tentukan Ë@

3. Batang logam yang panjangnya È diletakkan pada daerah yang memiliki medan magnet homogen. Batang logam diputar dengan kecepatan sudut tetap pada arah berlawanan dengan arah jarum jam (poros putar di titik O)

a. Tentukan besar dan arah gaya yang dialami elektron yang terletak sejauh CE dari poros O

b. Tentukan ggl imbas yang terjadi

4. Sebuah kawat berarus yang berbentuk seperti pada gambar di samping, dialiri arus 5 yang arahnya ditunjukkan oleh panah. Tentukan besar dan arah medan magnetik di titik O a. Oleh kawat setengah lingkaran CD dan EA b. Oleh kawat lurus c. Oleh seluruh kawat (total)

5. Cahaya merah dengan 633 nm jatuh pada dua celah yang

jarak antar celahnya 0,1 mm. Pola interferensi ditangkap pada layar yang jaraknya 6,55 meter dari celah tersebut

a. Gunakan cara fasor untuk menentukan posisi-posisi intensitas maksimum dan minimum pada layar. Nyatakan dalam sin 7. Gunakan asumsi 7 kecil

b. Berapa perbandingan intensitas maksimum terhadap intensitas masing-masing sumber

c. Berapa jarak antar dua maksimum berurutan di layar

Page 169: FISIKA TPB

SOLUSI UJIAN AKHIR SEMESTER II

FISIKA DASAR II

TAHUN 2003/2004

1. Diketahui : Tiga buah muatan 15 µC a. Ditanyakan : Potensial listrik di titik P Jawab :

˼ + ÙZ¾ ÙY

Z¾Y Ù¡Z¾¡- 9 u 10«15 u 10t0 +

√ 1 1-

˼ 3,65 u 10 V b. Ditanyakan : Kerja yang dibutuhkan Jawab : d Ì˼ 20 u 10t03,65 u 10 7,3 J c. Ditanyakan : Energi potensial sistem empat partikel Jawab :

+ÙÙYZY ÙÙ¡

Z¡ ÙÙûZû ÙYÙ¡

ZY¡ ÙYÙûZYû Ù¡Ùû

Z¡û -

9 · 10«15 · 10t0 +uæ uæ

uæ√ uæ

√ uæ uæ

-

12,79 J 2. Diketahui : 20 V ; 20 V ; c 6 Ω ; c 2 Ω ; c/ 4 Ω a. Ditanyakan : Tentukan arus yang mengalir c, c, c/ Jawab : Loop 1 : ∑ 5c ∑ 5c 5c 5 5 5/ Substitusi menghasilkan 5c c 5/c 85 65/ 20 Loop 2 : ∑ 5c ∑ 5c 5/c/ 25 45/ 16 Dari dua persamaan tersebut dihasilkan 5/ 2 A

5 s S20 62U 4 A

5 5 5/ 4 2 A b. Ditanyakan : Daya disipasi pada masing-masing resistor Jawab : Daya disipasi pada c adalah jc ¶ c 46 24 W Daya disipasi pada c adalah jc ¶ c 162 32 W Daya disipasi pada c/ adalah jc/ ¶/ c/ 44 16 W

Page 170: FISIKA TPB

c. Ditanyakan : Ë@ Jawab : Ë@ Ë@ Ë 5c 3. Diketahui : Batang dalam pengaruh medan magnet diputar a. Ditanyakan : Gaya yang dialami elektron Jawab : E E u DE sin b. Ditanyakan : GGL imbas yang terjadi Jawab : GGL yang ditimbulkan oleh perputaran batang

&@&(

Maka besarnya GGL yang timbul pada batang

4. Diketahui : Kawat berbentuk seperti gambar dialiri arus a. Ditanyakan : Medan magnet di O akibat kawat CD dan EA Jawab : Hukum Biot-Savart

fO L a &EuZ

ZY

Sepanjang CD C tegak lurus

DE¥£ fO L a & E

eY£

¥ f

Untuk kawat EA, dengan cara yang sama seperti di atas didapatkan

DEþ_ fO L a &E

eY_

þ f

b. Ditanyakan : Medan magnet di O akibat kawat lurus Jawab : Untuk kawat AC

DE_¥ fO L a &EuZ

ZY¥

_

Karena C sejajar BÉE, perkalian cross menghasilkan nol DE_¥ 0 Hasil yang sama berlaku pula pada kawat DE, sehingga c. Medan magnet di O akibat seluruh kawat Jawab : Total adalah DE DE_¥ DE 0 fOa

e 5. Diketahui : 633 nm ; a. Ditanyakan : Posisi intensitas maksimum dan minimum Jawab : Untuk 7 kecil (layar jauh dari celah), berkas

dari celah dapat dianggap sejajar sehingga beda lintasan antar celah tersebut adalah

∆M B sin 7

Ë ∑ 5c ∑ c 42 20 12 V

Batang dalam pengaruh medan magnet diputar

Gaya yang dialami elektron E sin 7 , tegak lurus maka 7 90] C dengan arah seperti pada gambar

GGL imbas yang terjadi GGL yang ditimbulkan oleh perputaran batang

&_&(

È &h&(

È

Maka besarnya GGL yang timbul pada batang È adalah È

Kawat berbentuk seperti gambar dialiri arus

Medan magnet di O akibat kawat CD dan EA

tegak lurus BÉE, sehingga dapat dituliskan fOaeY L BÉE£

¥ fOaeY mc fOa

e (dengan arah masuk bidang gambar

, dengan cara yang sama seperti di atas didapatkan Oa

eY L BÉE_þ fOa

eY 2mc fOae (dengan arah masuk bidang gambar)

Medan magnet di O akibat kawat lurus

, perkalian cross menghasilkan nol

Hasil yang sama berlaku pula pada kawat DE, sehingga DE£þ 0

Medan magnet di O akibat seluruh kawat

_¥ DE¥£ DE£þ DEþ_

0 fOae /fOa

e (dengan arah masuk bidang gambar)

nm ; B 0,1 mm ; É 6,55 m

Posisi intensitas maksimum dan minimum kecil (layar jauh dari celah), berkas-berkas

dari celah dapat dianggap sejajar sehingga beda lintasan antar

engan arah masuk bidang gambar)

suk bidang gambar)

(dengan arah masuk bidang gambar)

Page 171: FISIKA TPB

Beda fasa ∆ ∆M B sin 7 Intensitas maksimum pada layar : Beda fasa ∆ 26m 2 B sin 7 26m

B sin 7 26m # sin 7 6

&

Intensitas minimum pada layar : Beda fasa ∆ 26 1m 0 B sin 7 26 1m

B sin 7 26 1m # sin 7 6 b

&

b. Ditanyakan : Perbandingan intensitas maksimum terhadap intensitas sumber

Jawab : ¹ÏKr

¹O _OY_OY 4

c. Ditanyakan : Jarak antara dua maksimum berurutan Jawab : 7 kecil maka sin 7 tan 7 '

^

Intensitas maksimum : sin 7 6 /B

'^ 6

& # 6 à&

Jadi jarak antara dua maksimum berurutan adalah (∆6 1)

∆ ∆6 à& 0,//uæ0,

,uæ¡ 41 u 10t/ m = 4,1 cm

Page 172: FISIKA TPB

SOAL UJIAN AKHIR SEMESTER II

FISIKA DASAR II

TAHUN 2007/2008

1. Suatu rangkaian RLC disusun secara seri dengan È 0,1 H ; 1,014 : ; dan c 5Ω.

Rangkaian ini berada dalam keadaan resonansi, dan sumber tegangan yang digunakan memiliki Ë@9 50 V a. Hitunglah reaktansi induktif, reaktansi kapasitif, dan impedansi rangkaian dalam keadaan ini b. Hitung juga beda fase antara 5 dan serta besarnya arus maksimum yang mengalir c. Hitunglah tegangan maksimum pada resistor, induktor, dan kapasitor rangkaian ini 2. Suatu kapasitor 100 : dihubungkan dengan sumber tegangan 12 volt. Setelah keadaan tunak

tercapai, kapasitor ini dilepaskan dari sumber tegangan dan dihubungkan secara parallel dengan kapasitor (tidak bermuatan) yang memiliki dimensi persis sama dengan , namun mengandung bahan dielektrik Z 3,0

a. Hitung muatan dan energi yang tersimpan pada masing-masing kapasitor pada keadaan akhir b. Berapakah tegangan masing-masing kapasitor pada keadaan akhir 3. Suatu kawat penghantar (lihat gambar) dialiri arus searah sebesar ¶ 2 A. Panjang kawat 4 m, garis tengah setengah

lingkaran 6 m, kawat 3 m, dan kawat \ 1 m. j adalah titik pusat lengkung

a. Dengan menggunakan hukum Biot-Savart, turunkanlah besar dan arah medan magnet di titik j akibat arus pada segmen kawat

b. Dengan hukum yang sama, turunkanlah besar dan arah medan magnet di titik j akibat arus pada kawat lengkung

c. Dengan memakai perumusan pada soal (a) dan (b), tentukanlah besar dan arah medan magnet di titik j yang ditimbulkan oleh arus pada seluruh kawat penghantar

4. Diketahui gelombang transversal sinusoidal pada tali memiliki perioda 3 25 ms dan merambat

dalam arah x positif dengan laju 30 m/s. Pada 0, elemen tali di M 0 memiliki simpangan 2 cm dan bergerak ke bawah dengan laju 2 m/s. Tentukan

a. Amplitudo dan fasa awal dari gelombang b. Laju transversal (laju osilasi) maksimum pada tali c. Fungsi gelombang untuk gelombang tersebut 5. Dua buah cincin konsentrik dengan jari-jari cincin dalam adalah c dan

cincin luar c 4c, seperti tampak pada gambar di samping. Titik j terdapat pada sumbu putar kedua cincin tersebut yang berjarak 2c dari titik pusat cincin. Cincin dalam bermuatan Ì

a. Turunkan besar dan arah listrik di titik j karena muatan listrik pada

cincin dalam b. Tentukan muatan pada cincin luar agar medan listrik di titik j sama

dengan nol

Page 173: FISIKA TPB

SOLUSI UJIAN AKHIR SEMESTER II

FISIKA DASAR II

TAHUN 2007/2008

1. Diketahui : È 0,1 H ; 1,014 : ; c 5Ω ; Ë@9 50 V a. Ditanyakan : Reaktansi induktif, kapasitif, dan impedansi rangkaian Jawab : Pada keadaan resonansi terpenuhi hubungan Îà Î¥ # È 1/ l

ॠl ,,uæ 3140 1000m

Maka didapatkan Reaktansi induktif Îà È 100m Ω Reaktansi kapasitif Î¥ Îà 100m Ω Impedansi rangkaian

- vc Îà Î¥ c 5 Ω b. Ditanyakan : Beda fase antara dan 5 Jawab : Diagram fasor secara umum

Secara umum, beda fase antara arus dan tegangan

total adalah 3 dan memiliki besar

tan (%t(¤e

Dalam keadaan resonansi, Îà Î¥ 0 sehingga tan 0 dan beda fase 0 Arus maksimum

¶@2 ÔÏKr2N¿N 10 A

c. Ditanyakan : Tegangan maksimum pada masing-masing komponen Jawab : Ëe @2 ¶@2c 50 V Tegangan maksimum resistor Ëà @2 ¶@2Îà 5000m V Tegangan maksimum induktor Ë¥ @2 ¶@2Î¥ 5000m V Tegangan maksimum kapasitor 2. Diketahui : 100 µF ; Ë 12 V ; Z 3,0 a. Ditanyakan : Muatan dan energi pada kapasitor Jawab : Untuk mencari muatan pada masing-masing kapasitor kita harus mengetahui terlebih

dahulu tegangan masing-masing kapasitor. Setelah itu kita dapat menggunakan hubungan Ì Ë untuk menentukan besar muatan dalam kapasitor Ketika disambungkan, muatan dalam kapasitor 1 akan mengisi kapasitor 2 Jumlah muatan dalam sistem tetap yaitu Ì Ë sehingga berlaku hubungan Ë Ë Ë Perpindahan muatan akan berhenti ketika beda potensial antara kedua kapasitor sama, atau Ë Ë Ë. Dengan substitusi ke persamaan di atas didapatkan

Ë ¥ÔO¥¥Y Dari soal diketahui hubungan Z

Catatan : Kapasitor yang mengandung bahan dielektrik memiliki kapasitas Z kali lebih besar

Page 174: FISIKA TPB

Ë ff/

Muatan Ì Ë 100 Energi

Ë

100 Muatan Ì Ë 300 Energi

Ë

300 b. Ditanyakan : Tegangan masing Jawab : Ë Ë 4 3. Diketahui : ¶ 2 A ; a. Ditanyakan : Medan magnet di titik Jawab : Medan magnet di titik

satu elemen kemudian diintegralkan

B fOa

&2EuZZY fO

B fOa

A

2YAY¡/Y L fOa

A

2YAY¡/Y fOa

` + 2AY√2YAY

Substitusikan nilai-

Didapatkan b. Ditanyakan : Medan magnet di titik Jawab : Medan listrik di titik

Oleh sebab itu kita tinjau satu elemen kemudian diintegralkan

B fOa

&2EuZZY fO

sin 7 1 Seluruh nilai C konstan untuk seluruh elemen yaitu

fOa

eY L B4

Substitusi nilai yang telah diketahui menghasilkan

uæ/

c. Ditanyakan : Medan magnet di titik Jawab : Medan di titik j merupakan akumulasi dari seluruh bagian kawat ¼ _W Medan ¥£

¥£ fOa Alà

/f 4 V

u 10t04 4 u 10t C

100 u 10t04 8 u 10t J

300 u 10t04 1,2 u 10t/ C

300 u 10t04 2,4 u 10t/ J

Tegangan masing-masing kapasitor 4 V

4 m ; 6 m ; 3 m ; \ 1 m

Medan magnet di titik j akibat kawat AB

Medan magnet di titik j merupakan kontribusi seluruh elemen. Oleh sebab itu kita tinjau satu elemen kemudian diintegralkan

Oa

&2 ¨©ª hZY fOa

&2ZY +A

Z-

BM

Y BM fOa ` L

2YAY¡/Y

tà BM

Y-tà fOa

à

AÈYAY

-nilai yang telah diketahui uæ

/√Y/Y 1,07 u 10tß T dengan arah masuk bidang kertas

Medan magnet di titik j akibat kawat BC Medan listrik di titik j merupakan kontribusi seluruh elemen.

Oleh sebab itu kita tinjau satu elemen kemudian diintegralkan Oa

&9ZY , B4E tegak lurus terhadap C sehingga

konstan untuk seluruh elemen yaitu C c sehingga

L B4 fOa

eY mc fOa

e

Substitusi nilai yang telah diketahui menghasilkan

2,09 u 10tß T

Medan magnet di titik j akibat seluruh kawat

merupakan akumulasi dari seluruh bagian kawat W¥ ¥£ £þ

ढlढYAY

//√/Y/Y 4,71 u 10ts T

merupakan kontribusi seluruh elemen. Oleh sebab itu kita tinjau

T dengan arah masuk bidang kertas

merupakan kontribusi seluruh elemen.

sehingga

Page 175: FISIKA TPB

Medan £þ Medan akibat kawat AB, BC, dan CD dapat

sebelumnya. Untuk kawat DE, penurunannya sama akan tetapi berbeda pada batas integralnya

fOa ` +AY

Substitusi nilai-nilai yang dik

£þ uæ

Sehingga didapatkan medan total di titik ¼ _W ¼ 1,07 u ¼ 3,52 u 4. Diketahui : 3 25 ms ; a. Ditanyakan : Amplitudo dan fase awal Jawab : Misal fungsi simpangan

Maka fungsi kecepatan

Atau I| sinM

Untuk mendapatkan amplitudo, kita gunakan sedikit trik matematika /

l +I|-

Untuk mencari fase awal, bagi tan tan b. Ditanyakan : Laju transversal maksimum Jawab : Laju osilasi maksimum didapatkan dari amplitudo persamaan kecepatan yaitu 80m0 c. Ditanyakan : Fungsi Jawab : Kita tentukan terlebih dahulu nilai bilangan gelombang

Z@@( |

Sehingga fungsi gelombang menjadi

0,12 cos 5. Diketahui : c 4c a. Ditanyakan : Medan listrik di titik Jawab : Medan listrik di titik

cincin. Oleh karena itu kita tinjau kontribusi satu elemen kemudian integralkan untuk seluruh cincin

Medan akibat kawat AB, BC, dan CD dapat ditentukan dari penurunan pada bagian sebelumnya. Untuk kawat DE, penurunannya sama akan tetapi berbeda pada batas integralnya

+ 2Y√2YAY-e

eà¢A fOa +

ढ-B elढYeY

eà¢Aleà¢A

nilai yang diketahui menghasilkan æ +

/- ï /√/Y/Y /

v/Y/Yð 6,19 u 10t« T

Sehingga didapatkan medan total di titik j adalah W¥ ¥£ £þ 10tß 2,09 u 10tß 4,71 u 10ts 6,19 u 1010tß T

30 m/s ; 2 cm

Amplitudo dan fase awal

Misal fungsi simpangan cosM

Maka fungsi kecepatan ='=( sinM

M dengan 80m rad/s

Untuk mendapatkan amplitudo, kita gunakan sedikit trik matematikasinM cosM - l0,02 + /

s- 0,12 m

fase awal, bagi dengan / Substitusikan nilai ketika

s, # 0,378 0,12m

Laju transversal maksimum

Laju osilasi maksimum didapatkan dari amplitudo persamaan kecepatan yaitu0,02 5,03 m/s

Fungsi gelombang

Kita tentukan terlebih dahulu nilai bilangan gelombang dari hubungan

# |IôKÏÕKN s

/ 8m/3

Sehingga fungsi gelombang menjadi

cos +s/ mM 80m 0,12m- m

Medan listrik di titik j akibat muatan cincin dalam

Medan listrik di titik j merupakan hasil kontribusi seluruh elemen cincin. Oleh karena itu kita tinjau kontribusi satu elemen kemudian integralkan untuk seluruh cincin

ditentukan dari penurunan pada bagian sebelumnya. Untuk kawat DE, penurunannya sama akan tetapi berbeda pada batas integralnya

¢AYढYC

10t«

Untuk mendapatkan amplitudo, kita gunakan sedikit trik matematika

0

Laju osilasi maksimum didapatkan dari amplitudo persamaan kecepatan yaitu

merupakan hasil kontribusi seluruh elemen cincin. Oleh karena itu kita tinjau kontribusi satu elemen kemudian

Page 176: FISIKA TPB

Elemen dalam arah horizontal (bidang M) akan saling menghilangkan satu sama lain, sehingga medan listrik total akan berarah ke sumbu-z

B\(](@^ B\'

B\(](@^ &ZY cos 7 &

ZY£Z

Nilai C, 7, dan adalah konstan untuk seluruh elemen sehingga

\(](@^ ZY

£Z L B Ù

\(](@^ ÙeeY£Y¡/Y dengan arah ke sumbu-z positif

b. Ditanyakan : Muatan cincin luar agar medan di j nol Jawab : Misalkan muatan cincin luar Ì maka agar nol harus terpenuhi \^@Z \&@^@

Ù£eY£Y¡/Y tÙ£

eY£Y¡/Y

Ì Ì +0eYeYeYeY -// Ì +0eYeY

eYeY -// 8Ì